Download as pdf or txt
Download as pdf or txt
You are on page 1of 186

Solutions to

Mathematics
PULLOUT WORKSHEETS
FOR CLASS X
Second Terms
By
Surender Verma
M.Sc. (Mathematics), B.Ed.
Delhi Public School,
Dwarka, New Delhi

Since 1950
SARASWATI HOUSE PVT. LTD.
(An ISO 9001:2008 Company)
EDUCATIONAL PUBLISHERS
9, Daryaganj, Near Telephone Office, New Delhi-110002
Ph: 43556600 (100 lines), 23281022 • Fax: 43556688
E-mail: delhi@saraswatihouse.com
Website: www.saraswatihouse.com
Branches
Bengaluru: (080) 26619880 • Chandigarh: (0172) 2624882 • Chennai: (044) 24343740
Jaipur: 9672987282 • Kochi: (0484) 3925288 • Lucknow: (0522) 4062517
Mumbai: (022) 28343022 • Patna: (0612) 2570403
Second Term

1. Quadratic Equations

V Worksheets (1 to 20) ................................................................................................ 135

• Assessment Sheets (1 and 2) .................................................................................. 157


• Chapter Test ............................................................................................................ 160

2. Arithmetic Progressions

V Worksheets (24 to 37) .............................................................................................. 162

• Assessment Sheets (3 and 4) .................................................................................. 175


• Chapter Test ............................................................................................................ 178

3. Circles

V Worksheets (41 to 46) .............................................................................................. 180

• Assessment Sheets (5 and 6) .................................................................................. 186


• Chapter Test ............................................................................................................ 189

4. Constructions

V Worksheets (50 to 54) .............................................................................................. 191

• Assessment Sheets (7 and 8) .................................................................................. 203


• Chapter Test ............................................................................................................ 209

–3–
5. Some Applications of Trigonometry

V Worksheets (57 to 62) ............................................................................................... 211

• Assessment Sheets (9 and 10) ................................................................................ 219


• Chapter Test ............................................................................................................ 223

6. Probability

V Worksheets (66 to 68) .............................................................................................. 225

• Assessment Sheets (11 and 12) ............................................................................... 228


• Chapter Test ............................................................................................................ 231

7. Coordinate Geometry

V Worksheets (72 to 82) .............................................................................................. 233

• Assessment Sheets (13 and 14) .............................................................................. 247


• Chapter Test ............................................................................................................ 250

8. Areas Related to Circles

V Worksheets (85 to 89) .............................................................................................. 252

• Assessment Sheets (15 and 16) .............................................................................. 257


• Chapter Test ............................................................................................................ 260

9. Surface Areas and Volumes

V Worksheets (93 to 98) .............................................................................................. 262

• Assessment Sheets (17 and 18) .............................................................................. 269


• Chapter Test ............................................................................................................ 273

PRACTICE PAPERS (1 to 5) ................................................................................... 275

–4–
Solutions to
PULLOUT WORKSHEETS
AND
PRACTICE PAPERS
[Summative Assessments]
[SECOND TERM]
Chapter

1 QUADRATIC EQUATIONS

WORKSHEET– 1 ⇒ (x + 2 ) ( )
2x + 5 = 0
1. (B) For real and distinct roots, D > 0 ⇒ x + 2 = 0 or 2x + 5 = 0
⇒ b2 – 4ac > 0 ⇒ 16 – 4p > 0
⇒ 4 > p ∴ p < 4. −5
⇒ x = − 2 or x = .
2
2. (C) p(1)2 + p(1) + 3 = 0
7. 2x 2 − 5x + 3 = 0
3
⇒ 2p = – 3 ⇒ p = –
2 5 3
⇒ x2 − x + = 0
and (1)2 +1+q=0 ⇒ q=–2 2 2
5 3
 3 ⇒ x2 − x = −
∴ pq =  −  (– 2) = 3. 2 2
 2
2
(− 5) 2 Add both side  5 
3. (A) + 2( − 5 − k ) = 0 4
−5
2 2
5  5 3 5
⇒ –10 – 2k = 5 ⇒ x2 − x +  = − +  
2  4  2 4
⇒ – 2k = 15
2
−15  5 −24 + 25 1
⇒ x −  = =
⇒ k= .  4 16 16
2
4. Given equation: 4x2 – 3x – 5 = 0 5 1
⇒ x− = ±
Divide throughout by coefficient of x2, i.e., 4 4
4 to get 5 1 5 1
⇒ x = + or −
3x 5 4 4 4 4
x2 – − =0
4 4 3
⇒ x= or 1.
1 3 2
of coefficient of x = −
2 8 8. x = 2 or 1
3 Hint: The given eqn. can be written as:
and square of it =
64 x −7 − x − 4 11
3 =
Therefore, required constant is . ( x + 4 )( x − 7 ) 30
64
5. True ⇒ x2 − 3x − 28 = − 30
Reason: The value of t for which given ⇒ x2 − 3x + 2 = 0.
equation has real and equal roots are 9. Let speed of stream = x km/h
± 2 21 which are irrational. ∴ upstream speed = (18 − x) km/h
6. 2x 2 + 7 x + 5 2 = 0 downstream speed = (18 + x) km/h
Time taken to cover upstream distance of
⇒ 2x 2 + 5x + 2x + 5 2 = 0
24
⇒ x ( 2x + 5 + 2) ( )
2x + 5 = 0
24 km =
18 − x

Q U A D R A T I C E Q U A T I O N S 135
Time taken to cover downstream distance of 5. Yes.
24 ∵ (x – 2 )(x + 3 ) = 0
24 km =
18 + x
According to question, ⇒ x2 + ( 3 − 2 ) x – 6 = 0.

24 24
− = 1 3 ± 19
18 − x 18 + x 6. x =
5
 18 + x − 18 + x  Hint: 5x2 − 6x − 2 = 0
⇒ 24  = 1
 324 − x 2  6 2
⇒ x2 − x =
⇒ 48x = 324 − x2 5 5
⇒ 2
x + 48x − 324 = 0 2
2
x + 54x − 6x − 324 = 0 Add both side  3  and use
 5
⇒ x(x + 54) − 6(x + 54) = 0
⇒ (x − 6) (x + 54) = 0 a2 − 2ab + b2 = (a − b)2.
⇒ x = 6 or x = − 54 (Reject) 1 1
7. D = 0; real and equal roots, x = or
∴ x = 6 km/h. 3 3
OR Hint: Discriminant: D = b2 − 4ac.
Let size of square be x
x
∴ No. of students in square = x2 8. Let y=
2x + 1
∴ According to question,
∴ Given equation becomes
Case I. Total students = x2 + 24
Case II. Also total students = (x + 1)2 – 25 1 29
y+ =
∴ x2 + 24 = (x + 1)2 – 25 y 10
⇒ x2 + 24 = x2 + 2x + 1 – 25
y2 + 1 29
⇒ 2x = 48 ⇒ x = 24 ⇒ =
y 10
Number of students = (24)2 + 24
⇒ 10y2 − 29y + 10 = 0
= 576 + 24 = 600.
2
⇒ 10y − 25y − 4y + 10 = 0
WORKSHEET – 2 ⇒ 5y(2y − 5) − 2(2y − 5) = 0
⇒ (5y − 2) (2y − 5) = 0
1. (A) D = (− 2 2)2 – 4 × 2 × 1 = 8 – 8 = 0.
2 5
⇒ y= or y =
2. (C) Two equal and real roots 5 2
⇒ D= 0 2 x 2
⇒ 2
k –4×2×3= 0 If y = , then = gives x = 2
5 2x + 1 5
⇒ k2 = 4 × 6
5 x 5 5
⇒ k= ± 2 6 . If y = , then = gives x = − .
2 2x + 1 2 8
3. (A) x = a must satisfy the given equation 9. Yes, length = 40 m and breadth = 20 m
⇒ a2 – (a + b)a + k = 0 ⇒ k = ab. Hint: The situation will be possible if D ≥ 0.
4. For real and distinct roots,
OR
D >0
Let shorter side = x m
⇒ b2 − 4ac > 0 ∴ longer side = 30 + x
⇒ 16 − 4k > 0
∴ diagonal = x 2 + (30 + x )2
⇒ 16 > 4k ⇒ k < 4.

136 M A T H E M A T I C S – X
According to question, 5
6. x = ,2
2
x + (30 + x ) 2 = x + 60 3
Hint: See Worksheet − 1, Sol. 7 for comple-
ting square method.
7
7. p = 7; k =
4
Hint: Put x = – 5 in first equation, which
Squaring both sides gives p = 7
x2 + (30 + x)2 = (x + 60)2 And p(x2 + x) + k = 0 to have equal roots:
⇒ x2 + 900 + x2 + 60x = x2 + 3600 + 120x D =0
⇒ x2 − 60x − 2700 = 0 ⇒ (p)2 − 4 (p)(k) = 0
⇒ x2 − 90x + 30x − 2700 = 0 ⇒ (7)2 − 28k = 0
⇒ x(x − 90) + 30 (x − 90) = 0 7
⇒ k= .
⇒ (x + 30) (x − 90) = 0 4
⇒ x= − 30 (Rejected) 8. x = 0; 2(a + b)
or x = 90 Hint: Given equation is:
∴ Shorter side = 90 m x2 – x (2a + 2b) + 4ab = 4ab
Longer side = 120 m. ⇒ x2 – 2x (a + b) = 0.
9. 10, 5
WORKSHEET – 3 Hint: Quadratic equation is:

1. (A) For two distinct real roots, 1 1 3


+ =
D > 0 ⇒ b2 – 4ac > 0 ⇒ b2 > 4ac. x 15 − x 10
2. (C) Given equation is: 15 3
2 − 5x + 2x2 = 0 ⇒ 2
=
15x − x 10
⇒ 2x2 − 5x + 2 = 0
⇒ 150 = 45x − 3x2
D = (5)2 − 4 (2)(2)
⇒ 3x2 − 45x + 150 = 0
D = 9.
⇒ x2 − 15x + 50 = 0.
2
3. (D) Since and − 3 are roots of equation OR
3
2 −7 Let Ist part = x (larger)
Sum of roots = −3= ⇒m=3 ∴ 2nd part = 16 − x (smaller)
3 m
2 n According to question,
Product of roots = (− 3) = ⇒ n = − 6. (16 − x)2 + 164 = 2x2
3 m
256 + x2 – 32x + 164 = 2x2
4. No, as given equation is:
⇒ x2 + 32x − 420 = 0
x2 + x + 8 = x2 − 4
⇒ x − 12 = 0 ⇒ x2 + 42x − 10x − 420 = 0
∴ It is a linear equation. ⇒ x(x + 42) −10 (x + 42) = 0
⇒ (x − 10) (x + 42) = 0
5. False.
⇒ x = 10 or x = – 42 (Reject)
∵ x2 – 3x + 1 = 0
∴ Larger part = 10
is an equation with integral coefficients but
its roots are not integers. Smaller part = 6.

Q U A D R A T I C E Q U A T I O N S 137
WORKSHEET – 4 6. a = 3, b = – 6

1. (D) x = 1 must satisfy both the equations coefficient of x


Hint: Use sum of roots = –
⇒ a + a + 3 = 0 and 1 + 1 + b = 0 coefficient of x 2
constant term
3 Product of roots =
⇒ a= – and b = – 2 coefficient of x 2
2
⇒ ab = 3. 7. k = 14
Hint: For real and equal roots:
2. (B) For real and equal roots, D = 0
D= 0
⇒ (4k)2 – 4 .1. (k2 – k + 2) = 0
⇒ 2
b − 4ac = 0
⇒ 12k2 + 4k – 8 = 0
Use a = k − 12, b = 2(k − 12), c = 2.
⇒ 3k2 + k – 2 = 0
⇒ (3k – 2)(k + 1) = 0 9 ± 57
8. x =
⇒ 3k – 2 = 0 or k + 1 = 0 4
2 Hint: Given equation can be written as
⇒ k= or – 1.
3 2x2 – 9x + 3 = 0
∴ Quadratic formula is
2
3. (A) Putting x = in the given equation,
3 − b ± b 2 − 4 ac
we get x = .
2a
2
2 2
3   + p   + 4 = 0 9. Let unit's digit = x
3 3 and ten's digit = y
⇒ 4 + 2p + 12 = 0 ∴ Original number = 10y + x
⇒ p = – 8. Now xy = 16
4. Yes. As 16x2 − 24x − 1 = 0 16
⇒ y= ...(i)
x
−b ± D Also, according to question
⇒ x=
2a ⇒ 10y + x − 54 = 10x + y
⇒ 9y − 9x = 54
24 ± (24)2 + 4 ×16
= ⇒ y − x =6 ...(ii)
2 ×16
16
Substituting y = from (i) to (ii), we get
24 ± 576 + 64 x
= 16
32 − x =6
x
24 ± 640 3 ± 10 ⇒ 16 − x2 = 6x
= = .
32 4 ⇒ x2 + 6x − 16 = 0
⇒ 2
x + 8x − 2x − 16 = 0
1
5. − ±
33
⇒ (x + 8) (x − 2) = 0
4 ⇒ x = 2 or x = − 8 (Reject)
Hint: Divide the equation by 2 and then 16
∴ y= =8
1 2
add on both sides of equation.
4 ∴ Number is 82.

138 M A T H E M A T I C S – X
OR 2
 3 9
40 km/h ⇒  2 x +  – +5= 0
4 16
Hint: Let speed of train = x km/h
2
360 360  3 −71
∴ − =1 ⇒  2 x +  = <0
x x+5 4 16
2
WORKSHEET – 5 But  2 x + 3  cannot be negative for any
 4
1. (A) For real and equal roots, real value of x. So there is no real value of
4 x satisfying the given equation. Therefore,
D = 0 ⇒ 9k2 – 4 × 4 × 1 = 0 ⇒ k = ± . the given equation has no real roots.
3
2. (D) 2(x2 – x) = 3 ⇒ 2x2 – 2x – 3 = 0 7. Let another root be α.
Here, D = 4 + 4 × 2 × 3 = 28 > 0. −6 3
⇒ So roots are real and distinct. Product of roots = 2α = ⇒ α= −
2 2
3. (A) Given equation is 2x2 – 14x – 1 = 0
k 3 k
∴ D = b2 – 4ac Sum of roots = − ⇒ − +2= −
2 2 2
D = (– 14)2 – 4(2) (–1)
⇒ k= –1
= 196 + 8 = 204
3
14 ± 204 Thus, k = – 1 and another root = − .
∴ x= 2
4
7 ± 51 x −1
= . 8. Let y =
2 2x − 1
∴ Given equation can be written as:
4. No.
1 5
We have, x2 – 2x + 8 = 0 ⇒ y+ =
y 2
Put x = – 2, we get
⇒ (– 2)2 – 2(– 2) + 8 = 0 ⇒ 2y2 + 2 = 5y
16 = 0 which is wrong. ⇒ 2y2 – 5y + 2 = 0
Therefore, x = – 2 does not satisfy given ⇒ 2y2 – 4y – y + 2 = 0
equation. ⇒ 2y(y – 2) – 1 (y – 2) = 0
5. True. ⇒ (2y – 1) (y – 2) = 0
D = b2 – 4ac 1
⇒ y= or y = 2
Put b = 0 and a = 1 2
D = – 4c x−1 1 x−1
As c < 0 ⇒ – 4c > 0 ⇒ D > 0 ⇒ = or =2
2x − 1 2 2x − 1
⇒ Roots are real
⇒ 2x − 2 = 2x − 1 Not possible
b
Also, sum of roots = – a = 0 or x − 1 = 4x − 2
⇒ Roots are numerically equal and ⇒ 1 = 3x
opposite in sign. 1
⇒ x= .
6. Quadratic equation written as: 3
9. 15 hrs or 25 hrs
2 2
3
(2x)2 + 2 × (2x) × +  3  −  3  + 5 = 0 Hint: Let smaller tap takes x hrs to fill the
4  4  4 tank itself.

Q U A D R A T I C E Q U A T I O N S 139
∴ Larger tap will take (x − 10) hrs to fill As p(x2 + x) + k = 0, i.e., px2 + px + k = 0 has
the tank itself. equal roots,
∴ Given situation can be expressed as: D=0 ⇒ p2 – 4pk = 0
1 1 8 But p = 7, ∴ (7)2 – 4(7) k = 0.
+ =
x − 10 x 75 7
⇒ 4 × 7k = 7 × 7 ⇒ k= .
4
⇒ 4x2 − 115x + 375 = 0
5. 3x 2 − 6 x − 6 x + 2 = 0
15
⇒ x= (cannot be taken) or 25.
4 ⇒ 3x ( 3x − 2 − 2 ) ( )
3x − 2 = 0
OR
Let one number be x so another number be
⇒ ( 3x − 2) ( 3x − 2) = 0

15 – x. ⇒ 3x − 2 = 0 or 3x − 2 = 0
According to question, 2 2 6 6
⇒ x= ; ⇒ x= ; .
1 1 3 3 3 3 3
+ =
x 15 – x 10 6. 2 2x 2 + 15 x + 2 = 0
x + 15 – x 3 15 1
⇒ = ⇒ x2 + x+ = 0
x(15 – x ) 10 2 2 2
3x(15 – x) = 150 15 1
⇒ 15x – x2 = 50 ⇒ x2 + x = − .
2 2 2
⇒ 2
x – 15x + 50 = 0 2
2  15  to both sides,
Add 
⇒ x – 10x – 5x + 50 = 0
 4 2 
⇒ x(x – 10) – 5(x – 10) = 0
2 2
⇒ (x – 5)(x – 10) = 0 15  15  1  15 
⇒ x +2 = − +
⇒ x = 5 or x = 10 x +   
2 2 4 2  2 4 2 
When x = 5, 15 – x = 10;
2
when x = 10, 15 – x = 5  15  1 15
Hence, the numbers are 5 and 10. ⇒ x+  = − +
 4 2 2 32
WORKSHEET – 6 −16 + 15
=
32
1. (A) x2 – 4x + p = 0
2
For real roots, D ≥ 0  15  1
∴ x+  = −
⇒ (– 4)2 – 4 × 1 × p ≥ 0  4 2 32
⇒ 16 – 4p ≥ 0 ⇒ 4p ≤ 16 ⇒ p ≤ 4. which is not possible as square of any real
2. (C) For equal roots, D = 0 number can't be negative.
⇒ (6k)2 – 4 × 9 × 4 = 0 ∴ No real roots possible.
⇒ 36k2 = 4 × 36 ⇒ k = ± 2. 1 4
7. x = or
2 3
3. (B) D = ( 4 3 ) – 4 × 4 × 3 = 48 – 48 = 0
2
2x − 3 1 x −1
Hint: Let y = ∴ =
Since, the discriminant is zero, therefore, x −1 y 2 x−3
the given equation has real and equal roots.
4
4. – 5 must satisfy 2x2 + px – 15 = 0, Given equation becomes: y − =3
y
i.e., 2 × 25 – 5p – 15 = 0 ⇒ p = 7 Now solve.

140 M A T H E M A T I C S – X
8. Let the required number has x as ten’s digit 64
of the number. ⇒ 25m2 < 64 ⇒ m2 <
25
Given: Product of the digit = 8 2
8 8 8
8 ⇒ m2 <   ⇒ – <m< .
∴ Unit’s digit = 5 5 5
x
8 5. 3x2 + 9x + 2x + 6 3 = 0
∴ Number = 10x +
x
If 63 is subtracted from the number the
⇒ ( ) (
3x x + 3 3 + 2 x + 3 3 = 0 )
digit interchange their places. ⇒ ( 3x + 2 ) ( x + 3 3 ) = 0
8 8 −2
∴ 10x + – 63 = 10 × +x ⇒ x= or x = − 3 3 .
x x 3
8 80
⇒ 10x + – 63 = +x 6. 5 x2 + 9x + 4 5 = 0
x x
72 Dividing both sides by 5 to get
⇒ 9x – – 63 = 0
x 9
x2 + x+4= 0
⇒ 9x2 – 63x – 72 = 0 5
⇒ x2 – 7x – 8 = 0 9 
2
Adding   to both sides to get
⇒ (x + 1)(x – 8) = 0
2 5
⇒ x + 1 = 0, x – 8 = 0 2 2
9 9   9 
∴ x = – 1, x = 8 x2 + x + 4 +   =  
Reject x = – 1 ∴ x = 8 5 2 5 2 5
8 2
∴ Required number = 10 × 8 + = 81. 9 9  81
8 ⇒ x2 + x +   = 20 – 4
9. 60 km/hr 5 2 5
2 2
Hint: Let speed of express train = x km/h  9   1 
⇒ x +  = 
∴ Speed of another train = (x − 12) km/h  2 5 2 5
∴ According to question, 2 2
 9   1 
240

240
=1
⇒ x +  −   =0
 2 5  2 5
x − 12 x
Now solve.  9 1  9 1 
⇒ x + +   x+ −  =0
 2 5 2 5  2 5 2 5
WORKSHEET – 7
4
⇒x= − 5,– .
1. (D) x(px + 6) = – 1 ⇒ px2 + 6x + 1 = 0 5
For real and distinct roots, D > 0 7. 13, 15
⇒ 62 – 4 × p × 1 > 0 ⇒ p < 9. Hint: Two consecutive odd numbers are of
4 3 type 2x + 1, 2x + 3
2. (C) D = (– 2)2 – 4     = 4 – 4 = 0.
 3  4 ∴ According to question,
3. (D) For equal roots, D = 0 (2x + 1)2 + (2x + 3)2 = 394.
⇒ k2 – 4 × 4 × 9 = 0 ⇒ k = ± 12. Now solve.
4. For no real roots, D < 0 x
8. Let y =
∴ (5m)2 – 4 × 1 × 16 < 0 x+1

Q U A D R A T I C E Q U A T I O N S 141
∴ Given equation becomes According to question,
2y2 − 5y + 2 = 2 80 80
− =1
∴ D = b2 − 4ac x x+4
⇒ D = 25 − 4 (2) (2)
x+ 4− x
= 25 − 16 = 9 ⇒ 80  =1
 x( x + 4) 
−b ± D ⇒ 320 = x2 + 4x
∴y=
2a ⇒ x2 + 4x − 320 = 0
⇒ 2
x + 20x − 16x − 320 = 0
5± 9 5± 3 1
⇒y= = = 2,
4 4 2 ⇒ x (x + 20) − 16 (x + 20) = 0
⇒ (x − 16) (x + 20) = 0
x x 1
∴ = 2 or = ⇒ x = 16 or x = − 20 (Reject)
x+1 x+1 2
∴ Number of books = 16.
⇒ x = 2x + 2 or 2x = x + 1
⇒x=−2 or x = 1. WORKSHEET – 8
9. Let the present ages (in years) of father 1. (C)
and son be x and y respectively.
Hint: Discriminant is: D = b2 − 4ac.
∴ x = y2 ... (i)
2. (B) For equal and real roots, D = 0.
1 year ago, father’s age = (x – 1) years and
5
son’s age = (y – 1) years. ∴ (– 5)2 – 4 . k . k = 0 ⇒ 4k2 = 25 ⇒ k = ± .
2
According to the question, 3. (D) – 4 must satisfy x2 + px – 4 = 0
x – 1 = 8(y – 1) ⇒ p =3 ... (i)
⇒ x – 8y + 7 = 0 ... (ii) 2
x + px + q = 0 has equal roots
Substituting the value of x from equation (i) ⇒ p2 – 4q = 0 ... (ii)
in equation (ii), we have 9
y2 – 8y + 7 = 0 From (i) and (ii), we have p = 3, q = .
4
(y – 1)(y – 7) = 0 4. No.
⇒ y = 1 or 7 LHS = 4(3)2 − 14(3) + 16
Case I. If y =1 year
1 year ago, son’s age = 1 – 1 = 0 = 36 − 42 + 16 = 10
So, 1 year ago the father could not 8 times RHS = 9 − 12 + 3 + 9 − 9 = 0
as old as his son. As LHS ≠ RHS ⇒ x = 3 is not a root.
Case II. If y = 7 years
5. 5z 2 − 5z − 4 z + 4 5 = 0
1 year ago, son’s age = 7 – 1 = 6 years
So, 1 year ago, the father was 8 × 6 = 48 ⇒ ( ) (
5z z − 5 − 4 z − 5 = 0 )
years old.
∴ x = y2 = 72 = 49
⇒ ( 5z − 4 ) ( z − 5 ) = 0
Hence, present ages of the father and the 4
⇒ z= or z = 5.
son are 49 years and 7 years. 5
OR 6. 4x2 + 4 3 x + 3 = 0
Let number of books purchased = x
Divide both sides by 4.
80 3
∴ Cost of 1 book = x2 + 3x + = 0
x 4
142 M A T H E M A T I C S – X
2 2 Area (A1) of 1st square = x2
3  3  3
Area (A2) of 2nd square = y 2
⇒ x2 + 3x + +   –   =0
4  2  2  Perimeter of 1st square = 4x
 2 Perimeter of 2nd square = 4y
 3    3 3
⇒ 2
x + 3x +  + − =0 According to question,
  2    4 4 
x2 + y2 = 468; 4x − 4y = 24 ...(i)
2
     x=6+y ...(ii)
⇒ x+ 3 = 0 ⇒ x+ 3 x+ 3 = 0
 2   2  2  ∴ Use (ii) in (i)
(6 + y)2 + y2 = 468
3 3
⇒x=– ,− . ⇒ 36 + y2 + 12y + y2 = 468
2 2
⇒ 2y2 + 12y − 432 = 0
5 3 ⇒ y2 − 6y − 216 = 0
7. x = – ,
2 2 ⇒ 2
y + 18y − 12y − 216 = 0
x
Hint: Let y = . ⇒ y(y + 18) − 12(y + 18) = 0
x+1
⇒ (y – 12)(y + 18) = 0
8. 6 km/hr
⇒ y = −18 or y = 12
Hint: Let the speed of stream = x km/hr
According to question, Reject y = − 18 ∴ y = 12
∴ x = 18 m; y = 12 m.
24 24
− = 1 ⇒ x = 6.
18 – x 18 + x WORKSHEET– 9
9. Let the usual speed of the plane be
u km/h. Let the usual time of flight be 1. (D) For equal roots, D = 0
t hours. b2
∴ b2 – 4ac = 0 ⇒ c = .
Distance = Time × Speed 4a
1600 = t × u ... (i) −l
2. (C) Sum of roots = − ⇒ l+m=l
 40  1
1600 =  t −  × (u + 400)
 60  ⇒ m = 0 and l can take any real value,
2 800 e.g., m = 0, l = – 2.
⇒ 1600 = tu + 400t – u – ... (ii)
3 3 m
From equations (i) and (ii), we have Product of roots = ⇒ lm = m
1
1600 2 800 ⇒ m(l – 1) = 0 ⇒ m = 0, l = 1.
400 × − u− =0
u 3 3 3. (C) For real roots: D ≥0
⇒ u2 + 400u – 960000 = 0 ⇒ b2 − 4ac ≥0
⇒ u2 + 1200u – 800u – 960000 = 0 ⇒ k2 − 4(5)(5) ≥0
⇒ (u + 1200)(u – 800) = 0 ⇒ k2 − 100 ≥0
⇒ u = – 1200 or u = 800 ⇒ (k − 10) (k + 10) ≥0
Since, speed in negative sign is not possible ⇒ k ≤ − 10 or k ≥ 10.
∴ Speed = 800 km/h. 4. Yes, (x + 2)3 = x(x2 – 1)
OR ⇒ x + 2 + 3(x + 2)2x = x3 – x
3 3

Let side of 1st square = x ⇒ x3 + 8 + 6x2 + 12x = x3 – x


and side of 2nd square = y ⇒ 6x2 + 13x + 8 = 0
Let x > y which is a quadratic equation.

Q U A D R A T I C E Q U A T I O N S 143
According to question,
5. 3x 2 + 10 x + 7 3 = 0
10 1
x. (24 − x) = x + 9
⇒ x2 + x + 7= 0 4
3 Now solve.
2
 5  25 OR
⇒ x +  − 3 + 7= 0
 3 Let the usual speed and time of journey be
u km/h and t hours respectively
2 2
 5   2  Distance
⇒ x +  −  =0 Time =
 3  3 Speed
5 2  5 2 
⇒  x + +  x + –  =0 360
 3 3  3 3 t= ... (i)
u
7 But new speed = (u + 5) km/h and
⇒ x= − ,− 3.
3 time = (t – 1) hours

6. D= (8ab)2 − 4(3a2)(4b2) 360


Then, t–1= ... (ii)
= 64a2b2 − 48a2b2 u+5
= 16a2b2 Subtracting equation (ii) from (i) to get
= (4ab)2 ≥ 0 1 1 
1 = 360  − 
−b ± D u u + 5
∴x =
2a ⇒ u(u + 5) = 5 × 360
⇒ u2 + 5u – 1800 = 0
− 8 ab ± 4 ab
=
2 × 3 a2 −5 ± 25 + 4 × 1 × 1800
⇒u=
− 12 ab − 4 ab 2×1
= or
6 a2 6 a2 − 5 ± 85
⇒u= ⇒ u = – 45 or u = 40
− 2b − 2b 2
⇒x= or x = .
a 3a Neglecting u = – 45 due to negative value,
hence u = 40 km/h.
7. p = 14
Hint: For real and equal roots WORKSHEET– 10
D= 0
∴ Take a = p − 12 1. (A) For no real roots D < 0
b = 2(p − 12) ⇒ b2 − 4ac < 0 ⇒ 25p2 − 64 < 0
c=2 8
b2 − 4ac = 0. ⇒ 25p2 < 64 ⇒ p < ±
5
8. x = – 2 or 1
−8 8
Hint: Given equation is: ∴ <p< .
5 5
(4 − 3x) (2x + 3) = 5x
⇒ 8x + 12 − 6x2 − 9x = 5x 2. (B) For equal roots, D = 0
⇒ 6x2 + 6x − 12 = 0 ∴ 22(k – 12)2 – 4(k – 12) × 2 = 0
⇒ x2 + x − 2 = 0. ⇒ 4(k – 12)(k – 12 – 2) = 0
9. Son = 2 years; Father = 22 years ⇒ k = 12 or k = 14
Hint: Let boy's present age = x But k – 12 ≠ 0 as it is the coefficient of x2.
∴ Father's present age = 24 − x Hence, k = 14.

144 M A T H E M A T I C S – X
3. (B) For perfect square, D= 0 8. D= b2 − 4ac
⇒ [4(α − 3)]2 − 4(α − 3)(4) = 0 = [− 3(a2 + b2)]2 − 4(9)(a2 b2)
⇒ 16(α − 3)2 −16(α − 3) = 0 = 9(a2 + b2)2 − 36a2b2
⇒ 16(α − 3) (α − 3 − 1)] = 0 = 9a4 + 9b4 + 18a2b2 − 36a2b2
⇒ α = 3 (Reject) or α = 4 = 9a4 + 9b4 − 18a2b2
∴ Required value is α = 4. = (3a2 − 3b2)2 ≥ 0
4. Given equation is: 2x2 – 4x + 3 = 0
−b ± D 3(a2 + b2 ) ± 3(a2 − b2 )
Here a = 2, b = – 4, c = 3 ∴x= =
2a 2× 9
∴ D = b2 – 4ac = (– 4)2 – 4.2.3
= 16 – 24 = – 8 < 0. 6 a2 6b 2
⇒x= or x =
As D < 0, so the given equation has no real 18 18
root. a2 b2
⇒x= or x = .
5. Yes. 3 3
Given equation is
9. Let the breadth of the rectangular park be
(2x + 5)(5x – 3) = 16x2 – 3
b metres.
⇒ 10x – 6x + 25x – 15 = 16x2 – 3
2
Then its length = (b + 3) metres
⇒ 6x2 – 19x + 12 = 0 Area of the rectangular park = b(b + 3) sq. m
Therefore, given equation is quadratic Area of the triangular park
equation. 1
6. Let the Shefali’s marks in Mathematics be = × base × altitude
2
x and marks of English be (30 – x)
1
According to question, = × b × 12
2
(x + 2)(30 – x – 3) = 210
= 6b
⇒ 27x – x2 + 54 – 2x = 210
Now,
⇒ x2 – 25x + 156 = 0
2
area of rectangular park – area of triangular
⇒ x – 12x – 13x + 156 = 0 park = 4
⇒ x(x – 12) – 13(x – 12) = 0 b(b + 3) – 6b = 4
⇒ (x – 12)(x – 13) = 0 ⇒ b2 + 3b – 6b – 4 = 0
⇒ (x – 12) = 0, (x – 13) = 0 ⇒ b2 – 3b – 4 = 0
∴ x = 12, x = 13 ⇒ (b – 4)(b + 1) = 0 ⇒ b = –1, 4
⇒ 30 – x = 18, 17. Reject b = –1 as breadth is not possible in
Thus, marks in Mathematics and in English negative.
are 12 and 18 or 13 and 17 respectively. ∴ b = 4 m and b + 3 = 7 m
1 1 x−2−x Hence, length = 7 m and breadth = 4 m.
7. − =3 ⇒ =3
x x−2 x(x − 2) OR
⇒ 3x(x – 2) = – 2 ⇒ 3x2 – 6x = – 2 Let first number be x and second number
be x + 4.
⇒ 3x2 – 6x + 2 = 0
According to question,
6 ± 36 − 4 × 3 × 2 6 ± 12 1 1 4
⇒x= ⇒x= − =
2×3 2×3 x x+4 21

6±2 3 3± 3 x+ 4−x 4
⇒x= ⇒ x= . ⇒ =
2×3 3 x ( x + 4) 21

Q U A D R A T I C E Q U A T I O N S 145

4
=
4 ⇒ (x − 2)2 − ( 3 )2 = 0
2
x + 4x 21 ⇒ x2 + 4 − 4x − 3 = 0
1 1 ⇒ x2 − 4x + 1 = 0.
⇒ 2
=
x + 4x 21
x − 1 + 2x − 4 6
⇒ x2
+ 4x – 21 = 0 6. =
( x − 2)( x − 1) x
⇒ x2 + 7x – 3x – 21 = 0 ⇒ (3x − 5)x = 6 (x2 − 3x + 2)
⇒ x(x + 7) – 3(x + 7) = 0
⇒ 3x2 − 5x = 6x2 − 18x + 12
⇒ x = 3 or x = – 7. 2
⇒ 3x − 13x + 12 = 0
Reject x = –7 2
⇒ 3x − 9x − 4x + 12 = 0
∴ First number = 3
⇒ 3x(x − 3) − 4(x − 3) = 0
Second number = 7.
⇒ (3x − 4) (x − 3) = 0
WORKSHEET – 11 4
⇒ x= or x = 3.
3
1. (A) We have, kx2 − 2kx + 6 = 0
7. Discriminant for 3 x2 + 10x – 8 3 = 0 is
For real, equal roots D=0
given by
⇒ 4k2 − 24k = 0
⇒ 4k(k − 6) = 0 D = 102 – 4 × 3 × (– 8 3 )
⇒ k = 0 or k = 6 = 100 + 96 = 196
∴ k = 6. ⇒ D>0
2. (D) As D > 0, the given equation has real roots.
Hint: D = b2 − 4ac. −10 ± 196 −10 ± 14
3. (B) If x = – 2 is a root of the equation, then Now, x= =
2 3 2 3
k(– 2)2 + 5(– 2) – 3k = 0 ⇒ k – 10 = 0
⇒ k = 10. 2
⇒ x= −4 3,
4. Yes. 3

2 2 Thus, the given equation has real roots


 2
At x = , 9x2 – 3x – 2 = 9  2  – 3   – 2 2
3  3  3
which are − 4 3 and .
=4 – 4 = 0 3
2 8. Let P's obtained marks in Mathematics
1 1  1
At x = – , 9x2 – 3x – 2 = 9  −  – 3  −  – 2 be x then obtained marks in Science be
3  3   3
(28 – x).
=2 – 2 = 0
Consider the question,
2 1 P’s new marks in Mathematics = (x + 3)
Clearly, both the values of x =  , − 
3 3 and P’s new marks in Science = (28 – x – 4)
satisfy the equation 9x2 – 3x – 2 = 0, so, = (24 – x)
2 1 According to question,
x=  ,−  are the roots of it.
3 3 Product of new marks = 180
5. Consider, α+β=4 ⇒ (x + 3)(24 – x) = 180
⇒ 24x – x2 + 72 – 3x = 180
⇒ 2− 3 +β=4
⇒ x2 – 21x + 108 = 0
⇒ β=2+ 3
⇒ x2 – 12x – 9x + 108 = 0
∴ Equation is: (x − 2 + 3 ) (x − 2 − 3)=0 ⇒ x(x – 12) – 9 (x – 12) = 0

146 M A T H E M A T I C S – X
⇒ (x – 9)(x – 12) = 0 ⇒ x2 = 12 + x (Squaring)
⇒ x – 9 = 0, x – 12 = 0 ⇒ x2 – x – 12 = 0
∴ x = 9, x = 12 ⇒ x = 4, – 3 (Reject) ∴ x = 4.
If x = 9; 28 – x = 19, so, marks obtained in 2. (B) For equal roots, D = 0
Mathematics and Science are 9 and 19
∴ { – 2b(a + c)}2 – 4(a2 + b2)(b2 + c2) = 0
respectively.
⇒ 4a2b2 + 4b2c2 + 8ab2c – 4a2b2 – 4c2a2
If x = 12; 28 – x = 16, so, marks obtained in
Mathematics and Science are 12 and 16 – 4b4 – 4b2c2 = 0
respectively. ⇒ b – 2ab c + c a = 0 ⇒ (b2 – ca)2 = 0
4 2 2 2

OR ⇒ b2 = ac.
Let the sides of the two squares be a and b.
Now, sum of areas = 640 3. (Α) α – β = (α + β)2 − 4 αβ
⇒ a2 + b2 = 640 ... (i)
Difference of perimeters = 64 = ( 2 )2 − 4 (1) = 0.
⇒ 4a – 4b = 64
⇒ a – b = 16 4. p ≤ − 2 6 or p ≥ 2 6
⇒ b = a – 16 ... (ii) Hint: Use D ≥ 0
From equations (i) and (ii), we have i.e., b2 − 4ac ≥ 0.
a2 + (a – 16)2 = 640 5. D = p2 – 4 × 1 × 2q = p2 – 8q
⇒ 2a2 – 32a – 384 = 0
∵ q < 0 ∴ 8q < 0 ⇒ – 8q > 0 ...(i)
⇒ a2 – 16a – 192 = 0
⇒ 2
a – 24a + 8a – 192 = 0 Further p2 ≥ 0 ...(ii)
⇒ (a – 24)(a + 8) = 0 Adding equation (i) and (ii) to get
⇒ a = 24 or a = – 8 p2 – 8q > 0 ⇒ D > 0 ⇒ Real and distinct
Rejecting a = – 8 (negative length), roots.
... a = 24 m 6. 4x2 + 4bx – (a2 – b2) = 0
Using equation (ii), b = 8 m.
1
Hence, sides of the two squares are 24 m ⇒ x2 + bx − (a2 – b2) = 0
4
and 8 m.
9. 3 hr 30 min. b2 b2 1 2 2
⇒ x2 + bx + − − (a – b ) = 0
Hint: Let average speed = x km/h 4 4 4
2 2
∴ Distance = 2800 km ⇒  b  a
2800  x +  –   = 0
2 2
∴ Original time (duration) =
x  b a  b a
2800 ⇒  x + +   x + −  = 0
∴ New time = 2 2 2 2
x − 100
[∵ A2 – B2 = (A + B)(A – B)]
2800 2800 1
∴ − =  a + b  a − b
x − 100 x 2 ⇒  x +  x −  = 0
Now solve. 2  2 
a+b a−b
⇒ x=– , .
WORKSHEET–12 2 2
7. Hint: D= 0
... 2
b – 4ac = 0
1. (A) Let x = 12 + 12 + 12 + .... ...(i)
(c – a)2 – 4(b – c) (a – b) = 0
⇒ x = 12 + x ⇒ (c + a – 2b)2 = 0
[Using equation (i)] ⇒ c+a= 2b

Q U A D R A T I C E Q U A T I O N S 147
a+c 200 200
⇒ b= . ∴ − = 2 (Accordingly question)
2 l l+5
x + 2 + 2x + 2 4 l+5−l
8. = ⇒ 200 × = 2 ⇒ l2 + 5l – 500 = 0
( x + 1)( x + 2) x+4 l(l + 5)
⇒ (3x + 4)(x + 4) = 4 (x2 + 3x + 2) ⇒ l2 + 25l – 20l – 500 = 0
⇒ 3x2 + 12x + 4x + 16 = 4x2 + 12x + 8 ⇒ (l – 20)(l + 25) = 0
⇒ x2 − 4x − 8 = 0 ⇒ l = 20 (Rejecting l = – 25)
4 ± 16 + 32 200
∴ x= Rate = = ` 10.
2 20
Hence, the original length of the piece is
4 ± 48 20 m and its original rate is ` 10 per metre.
x=
2
WORKSHEET – 13
= 2 ± 2 3.
9. Let 1st pipe fill cistern in x min 1. (D) For real roots, D ≥ 0
and 2nd pipe fill cistern in (x + 3) min. ∴ (– 3p)2 – 4 × 4 × 9 ≥ 0 ⇒ 9p2 ≥ 4 × 4 × 9
The part of cistern filled by 1st pipe in ⇒ p ≥ 4 or p ≤ – 4.

1 5
1 min = . 2. (A) x = must satisfy 2x2 – 8x – m = 0
x 2
2
The part of cistern filled by 2nd pipe in 5 5 15
∴ 2   − 8   – m = 0 ⇒ m = − .
1 2 2 2
1 min = .
x+3
3. (A) Let y = 6 + 6 + 6 + 6 + .... .
According to question,
1 1 13 ∴ y = 6+y ⇒ y2 − y − 6 = 0
+ =
x x+3 40 ⇒ (y − 3) (y + 2) = 0
x+3+x 13 ⇒ y = 3 or − 2 (Reject).
=
2
x + 3x 40 4. 5x2 + (3p + 2)x + 15 = 0
Putting x = 5, we have
⇒ 80x + 120 = 13x2 + 39x
5 × 25 + (3p + 2) × 5 + 15 = 0
⇒ 13x2
− 41x − 120 = 0
⇒ 125 + 15p + 10 + 15 = 0
⇒ 13x2 − 65x + 24x − 120 = 0
⇒ 15p = – 150
⇒ 13x (x − 5) + 24(x − 5) = 0 ⇒ p = – 10.
⇒ (13x + 24) (x − 5) = 0
3
− 24 5. 4x2 + 4 3x + 3 = 0 ⇒ x2 + 3x + =0
⇒ x= (Rejected), x = 5 4
13 2 2
 3  3 3
∴ Required time = 5 min, 8 min. ⇒ x2 + 3x +   –   + =0
 2   2  4
OR 2
Let the original length of the piece be  3 = 0 3
⇒  x +  ⇒ x+ =0
l metres. 2  2
Cost 3
∵ = Rate ⇒ x=– .
Length 2

148 M A T H E M A T I C S – X
6. D = b2 − 4ac Case II. If y = 3
= ( 3 + 1)2 − 4(1)( 3 ) x2 + 2
= 3 ⇒ x2 + 2 = 9x2 – 18
= 3+ 1 + 2 3 −4 3 x2 − 2
=4− 2 3 5
⇒ 8x2 = 20 ⇒ x = ±
= ( 3 − 1 )2 > 0 2

∴ x=
−b± D
⇒ x=
3 +1 ± ( 3 −1 ) Hence, x= ±
10 5
.
, ±
2a 2 3 2
x= 3, 1. 9. 25 min and 20 min.
7. 2x2 + 5x − 4 = 0 1 1 9
Hint: Use: + = .
−5 x x + 5 100
⇒ α + β= ; α .β = − 2
2 OR
2 750 km/hr
α β
+ =
α 2 + β2 (α + β ) − 2α.β
(a) = 1500 1500 1
β α αβ αβ Hint: Use: = − =
x x + 250 2
2
 − 5 where x = usual speed.
 2  + 4 − 41
= = . WORKSHEET – 14
−2 8
(b) α3 + β3 = (α + β)3 − 3 αβ (α + β) 1. (B) For equal roots, D = 0
∴ 4(a2c2 + b2d2 + 2abcd) – 4(a2c2 + a2d2 + b2c2
− 125  −5
= +6   + b2d2) = 0
8  2  2 2 2 2
⇒ 8abcd = 4(a d + b c )
− 125 − 120 − 245 ⇒ a2d2 + b2c2 – 2abcd = 0
= = .
8 8 ⇒ (ad – bc)2 = 0 ⇒ ad = bc.
8. Given equation is 2. (D) For no real roots, D < 0
x +22
x −2 2 ∴ k2 – 4 × 1 × 1 < 0 ⇒ k2 – 22 < 0
+6 2 =5 ... (i)
2
x −2 x +2 ⇒ (k – 2)(k + 2) < 0 ⇒ – 2 < k < 2.
3. (A) Let us find the discriminant of equation
x 2 + 2 so that 1 x2 − 2
Putting y = = , x2 – 4x + 3 2 = 0.
x2 − 2 y x2 + 2
D = (– 4)2 – 4 × 1 × 3 2 = 16 – 12 2
equation (i) reduces to
= 16 – 16.97
6
y+ = 5 ⇒ y2 – 5y + 6 = 0 ⇒ D < 0.
y
⇒ (y – 3)(y – 2) = 0 ⇒ y = 2 or 3 Therefore, x2 – 4x + 3 2 has no real roots.
Case I. If y = 2 4. Given equation is:
x2 + ax – 4 = 0
x2 + 2 D = b2 – 4ac
= 2 ⇒ x2 + 2 = 4x2 – 8
x2 − 2 = a2 – 4(– 4)
⇒ 3x2 = 10 ⇒ x = ± 10 = a2 + 16 > 0
3 As D > 0, two real and distinct roots exist.

Q U A D R A T I C E Q U A T I O N S 149
5. Let the required whole number be x. 8. Let the tap of larger diameter takes x hours
1 to fill the tank. Therefore, the other tap will
∴ x – 20 = 69 × take (x + 10) hours to fill the same tank.
x
⇒ x2 – 20x = 69 The tap of larger diameter will fill the tank
⇒ x2 – 20x – 69 = 0 1
part in one hour and the other one will
x
⇒ x2 – (23 – 3)x – 23 × 3 = 0
⇒ (x – 23)(x + 3) = 0 1
fill part in the same time.
⇒ x = 23 or x = – 3 x + 10
But – 3 is not a whole number According to the question,
∴ x = 23.
1 1 1
2 a + b a + 2b + =
6. x = , x x + 10 3
3 3 9
8
Hint: See solved example 5(ii). 2 x + 10 8
⇒ =
25 x( x + 10) 75
7. (x − 5) (x − 6) =
(24)2 ⇒ 4x2 – 35x – 375 = 0
2
⇒ 4x – 60x + 25x – 375 = 0
25
⇒ x2 − 11x = − 30 ⇒ 4x(x – 15) + 25(x – 15) = 0
(24)2
25
2 ⇒ x = 15, −
Add  11 to both sides. 4
 2
25
2 2
Rejecting x = − hours due to negative
11 25 11 4
⇒ x2 − 11x +   = − 30 +   time, we have
 2 (24)2  2
x = 15 hours and x + 10 = 25 hours.
2 Hence the tap of larger diameter and of
 11  25 121
⇒ x−  = 2
− 30 + smaller diameter can separately fill the tank
 2 (24) 4 in 15 hrs and 25 hrs respectively.
25 1 OR
= 2
+
(24) 4 7 m, 4 m
Hint: See Worksheet − 10, Sol. 9.
25 + 144  13  2
= =   1 1 1 1
576  24  9. − = +
a+b+x x a b
11 13 x− a−b− x b+a
⇒ x− = ± . ⇒ =
2 24 x( a + b + x ) ab
11 13
⇒ x = ± ⇒ − ab = x (a + b) + x2
2 24
⇒ x2 + x (a + b) + ab = 0
132 ± 13 ⇒ (x + a) (x + b) = 0
⇒ x =
24
⇒ x = − a or − b.
145 119 OR
⇒ x = ;
24 24 Hint: D = 4a(a3
+ b3 + c3 – 3abc)
1 23 ∴ D=0
⇒ x = 6 ; 4 .
24 24 a = 0 or a3 + b3 + c3 = 3abc.

150 M A T H E M A T I C S – X
WORKSHEET – 15 ∴ a = α – 3, b = 0,
c = 4 (α – 3) – 4 = 4 (α – 4)
1. (C) For real roots, D ≥ 0 ∴ D = 0 – 4(α – 3) × 4 (α – 4) = 0
∴ (– k)2 – 4 × 5 × 1 ≥ 0 ⇒ k2 ≥ 20 ⇒ α = 3 or α = 4
⇒ k ≤ – 20 or k ≥ 20 . But α ≠ 3, i.e., α – 3 ≠ 0, as (α – 3) is the
constant of the leading term.
2. (A) 3(2)2 – 2p(2) + 2q = 0
Hence, α = 4.
and 3(3)2 – 2p(3) + 2q = 0
8. Yes; 5 m and 12 m
⇒ 4p – 2q = 12 and 6p – 2q = 27
Hint: Let distance of pole P from gate B be
15 x m and from A, (x + 7) m.
⇒p= , q = 9.
2 Therefore, x2 + (x + 7)2 = 132
3. (B) D = ( 4 3 ) – 4 × 3 × 4.
2
Now solve.
= 48 – 48 = 0
⇒ Two roots are real and equal.
4. False.
There can be quadratic equation which have
no real roots e.g. x2 + 2x + 7 = 0; This equation
has no real roots because D = – 24 < 0.
5. No. OR
Let their ages be x years and y years.
Then x + y = 20 ... (i)
And (x – 4)(y – 4) = 48 ... (ii)
Consider equation (ii).
xy = 112 ... (iii )
From equations (i) and (iii), we have
x2 – 20x + 112 = 0
Here, D<0
Hence, the given situation is not possible.
6. D= b2 − 4ac
Let the snake is caught at a distance of x m
= 16a4 − 4(4) (a4 − b4)
from the pillar base
= 16a4 − 16a4 + 16b4
∴ From figure, AC2 = 92 + x2
= (4b2)2 ≥ 0
(Using Pythagoras Theorem)
−b ± D 4 a2 ± 4b 2 and CD = 27 − x.
∴ x= =
2a 2× 4 Since their speed are same so,
a2 + b 2 a2 – b 2 AC = CD (∵ Distance covered will be
⇒ x= ; . equal in equal time)
2 2
⇒ AC2 = CD2
7. (α – 3) x2 + 4 (α – 3) = 4
⇒ 81 + x2 = (27 − x)2
⇒ (α – 3) x2 + 4(α – 3) – 4 = 0 ... (i)
81 + x2 = 729 + x2 – 54x
Since equation (i) has real and equal roots,
∴ Discriminant (D) = 0 54x = 648
⇒ b2 – 4ac = 0 ∴ x = 12 m.

Q U A D R A T I C E Q U A T I O N S 151
a a+b 4. Yes.
9. x2 + x + x+1 = 0
a+b a (x – 1)3 = x3 – 2x + 1
a  a+b  a  ⇒ x3 – 1 + 3x(–1)(x – 1) = x3 – 2x + 1

⇒ xx+ + x + =0
 a + b  a  a + b ⇒ x3 – 1 – 3x2 + 3x = x3 – 2x + 1
⇒ 3x2 – 5x + 2 = 0
 a + b  a  That is a quadratic equation.
⇒  x +   x + a + b  = 0
a   5. (2x – 1) (x + 7) = 9
−a a+b ⇒ 2x2
+ 14x – x – 7 = 9
∴ x= or −
a+b a ⇒ 2x2 + 13x – 16 = 0
OR ⇒ D = 169 – 4 (2) (–16)
4x2 + 4bx = a2 − b2 = 169 + 128
a2 − b 2 = 297
⇒ x2 + bx =
4
−13 ± 297
∴ x = .
b
2
a2 − b 2 b2 4
⇒ x2 + bx +   = +
 2 4 4 6. 4x2 – 2(a2 + b2) + a2b2 = 0
Here, A = 4, B = – 2(a2 + b2) and C = a2b2
 b
2
a2
⇒  x +  = Now,
 2 4

b a −B ± B 2 − 4AC
⇒ x+ = ± x=
2 2 2A

−b ± a 2( a 2 + b 2 ) ± 4( a 2 + b 2 )2 − 4 × 4 × a 2 b 2
x= . =
2 2×4

WORKSHEET – 16 2( a 2 + b 2 ) ± 2 a 4 + b 4 + 2 a 2 b 2 − 4 a 2 b 2
=
1. (C) For real and equal roots, D = 0. 2×4
∴ (4k)2 – 4 × 12 × 3 =0
⇒ 16(k2 – 32) =0 a 2 + b 2 ± ( a 2 − b 2 )2
=
⇒ (k – 3)(k + 3) =0 4
⇒ k = ± 3. a 2 + b 2 + a 2 − b 2 a2 + b2 − a2 + b 2
= ,
2 4 4
2. (B) α + β = and αβ = – 3
1 a2 b 2
⇒ (α + 2) + (β + 2) = 2 + 4 ∴ x= , .
2 2
and (α + 2)(β + 2) = αβ + 2(α + β) + 4
10
= – 3 + 2 (2) + 4 7. k = − or k = 2
9
⇒ S = 6 and P = 5
Required equation: x2 – Sx + P = 0, Hint: Put D = b2 − 4ac = 0
i.e., x2 – 6x + 5 = 0. Take a = 1, b = −2(1 + 3k)
3. (B) (b)2 – (a + b) b + p = 0 c = 7(3 + 2k).
⇒ b2 – ab – b2 + p = 0 8. 81
⇒ p = ab. Hint: See Worksheet – 6, Sol. 8.

152 M A T H E M A T I C S – X
OR
2 +1+ 2 −1 2 +1− 2 +1
25 students = ,
2 2
Hint: Let the number of students attended
picnic = x 2 2 2
= , ∴ x= 2, 1
500 2 2
∴ Per head contribution =
x Hence, the required roots are 2 and 1.
According to question,
500 500 WORKSHEET – 17
− =5
x–5 x
1. (A) Let roots be α and β such that
x − x + 5
∴ 500   =5 3+ 2 3− 2
 x( x − 5)  α= and β = .
3 3
⇒ 500 = x2 – 5x
Required equation would be
⇒ 2
x – 5x – 500 = 0.
x2 – (α + β)x + αβ = 0
2x 1 3x + 9 7
9. + + =0 ⇒ x2 – 2x + = 0
x − 3 2 x + 3 ( x − 3)(2 x + 3) 9
⇒ 2
9x – 18x + 7 = 0.
2x(2 x + 3) + x − 3 + (3 x + 9)
⇒ =0
( x − 3)(2x + 3) 2. (B) α2 + β2 – 4αβ = (α + β)2 – 6αβ
2
4 x 2 + 10 x + 6 − 5 25 73
⇒ =0 =   – 6 (– 2) = + 12 = .
( x − 3)(2 x + 3)  2  4 4
⇒ 2x2 + 5x + 3 = 0 2x + 1 + x − 1
3. (D) Given equation is: =3
⇒ 2x2 + 3x + 2x + 3 = 0 ( x − 1) (2x + 1)
⇒ (2x + 3)(x + 1) = 0
⇒ x = 2x2 − x −1
3 2
⇒ x = – 1, − ⇒ 2x − 2x − 1 = 0
2
∴ D = (– 2)2 – 4 × 2 × (– 1) = 12.
3
But x ≠ − . Therefore, x = – 1. 4. – 12 < k < 12
2
OR Hint: D < 0
The given quadratic equation is ⇒ k2 – 144 < 0
⇒ (k – 12) (k + 12) < 0
x2 – ( 2 + 1) x + 2 =0
⇒ − 12 < k < 12.
Here, a = 1, b = – ( 2 + 1) , c = 2 5. D = 0
2
−b ± b − 4 ac ⇒ (11 + m)2 − 4(3m + 1)(9) = 0
Now, x = ⇒ 121 + m2 + 22m − 108m − 36 = 0
2a
⇒ m2 − 86m + 85 = 0
⇒ 2
m − 85m − m + 85 = 0
2 + 1 ± 2 + 1+ 2 2 − 4 2
= ⇒ m(m − 85) − 1(m − 85) = 0
2
⇒ m = 1 or m = 85.
2 +1± 2−2 2 +1 4b 2 − 3a2
= 6. ,
2 a2 b2
2 +1 ± ( 2 − 1)
2
− B ± B 2 − 4AC
= Hint: Use x =
2 2A

Q U A D R A T I C E Q U A T I O N S 153
7. Let the triangle be ABC. Number of soldiers in each row = x
According to the question, and number of rows = x – 2
AC = 50 cm ...(i) Therefore, total number of soldiers
AB + BC + AC = x(x – 2) ... (i)
When total number of rows are doubled,
= 112 cm ...(ii) number of soldiers in each row
From equations (i) and (ii), we have = x–7
AB + BC = 62 cm ... (iii) and number of rows = 2(x – 2)
Using Pythagoras theorem in ∆ABC, Therefore, total number of soldiers
we have = 2(x – 2)(x – 7) ... (ii)
AC2 = AB2 + BC2 ... (iv) According to the question, we arrive
From equations (i), (iii) and (iv), we have equation (ii) – equation (i) = 160
(50)2 = AB2 + (62 – AB)2 ⇒ 2(x – 2)(x – 7) – x(x – 2) = 160
⇒ 2AB2 – 124AB + 3844 – 2500 = 0 ⇒ x2 – 16x – 132 = 0
⇒ AB2 – 62AB + 672 = 0 ⇒ x2 – 22x + 6x – 132 = 0
⇒ AB2 – 48AB – 14AB + 672 = 0 ⇒ (x + 6)(x – 22) = 0
⇒ x = – 6, 22
⇒ (AB – 48)(AB – 14) = 0
Negative value of x cannot be taken, so
⇒ AB = 48 cm or AB = 14 cm
required number of soliders is 22 in each
⇒ BC = 14 cm or BC = 48 cm row.
[Using (iii)] OR
1 2
Now, ar(∆ABC) = × 14 × 48 .
2 5
= 336 cm2. x 2 x + 1 29
Hint: + =
2x + 1 x 10
8. 4 x − 3 = 6 − 2x + 3
Squaring both sides, ⇒ 8x2 – 11x – 10 = 0

4x − 3 = 36 + 2x + 3 − 12 2x + 3 −5
⇒ x = 2 or .
8
⇒ 2x − 42 = − 12 2x + 3
⇒ x − 21 = − 6 2 x + 3 WORKSHEET – 18
Again, squaring both sides, we get, 1. (Β) − 5 is a root of 2x2 + px − 15 = 0
x2 + 441 − 42x = 36(2x + 3) ∴ 2(−5)2 +p (−5) −15 = 0
⇒ x2 + 441 – 42x = 72x + 108 ⇒ p= 7
⇒ 2
x − 114x + 333 = 0 ∴ p(x2 + x) + k = 0
⇒ 2
x − 111x − 3x + 333 = 0 ⇒ 7x2 + 7x + k = 0
⇒ x(x − 111) − 3(x − 111) = 0 For equal roots, D= 0
⇒ (x – 3)(x – 111) = 0 ⇒ 49 − 28k = 0
∴ x = 3 or x = 111 (Reject). 7
⇒ k= .
9. Let required number of soldiers in each 4
row be x. 2. (A) D = b2 – 4ac
When the number of soldiers in each row = (– 6)2 – 4 × 7 × (– 13 7 )
is 2 more than the total number of rows, = 36 + 364 = 400.

154 M A T H E M A T I C S – X
3. (A) For no real roots, D < 0. 8. x = 3, – 1, 1 ± 2
2 Hint:
∴ 25k2 – 64 < 0 ⇒ k2 –  8  < 0
 5 Let y = x2 − 2x
8 8 ∴ Given equation becomes y2 − 4y + 3 = 0.
⇒ − <k< .
5 5 5
9.
4. The given equation is a perfect square, if 8
D = 0. Hint: Let numerator = x
⇒ (2p + 4)2 – 4(4 – p)(8p + 1) = 0 ∴ Denominator = x + 3
⇒ 4p + 16p + 16 + 32p2 – 124p – 16 = 0
2 x
∴ Fraction =
⇒ 36p2 – 108p = 0 x+3
⇒ p(p – 3) = 0 ⇒ p = 0 or 3. According to question,
x+1 x 1
5. No. − = .
x + 4 x + 3 24
If x = − 3 is a solution of x2 + 2 2x + 3 = 0
OR
x = − 3 satisfies it. 84
So, LHS = − 3 ( ) 2
+ 2 2 (− 3 ) + 3 Hint: Let unit's place = x
ten's place = y
=3–2 6 +3=6– 2 6 ≠0 ∴ Original number = 10y + x
Hence, x = − 3 is not a solution of the According to question,
given equation. 10y + x = 7(y + x) ... (i)
and 10y + x = 3(xy) − 12 ... (ii)
q2
6. x = , –1 Substitute the value of y from (i) to (ii) and
p2 solve.
Hint: D = b2 − 4ac
= (p2 – q2)2 – 4 × p2 × –(– q)2 WORKSHEET – 19
= (p2 + q2)2 1. (C) S = 8 + 2 ⇒ 10 = − a ⇒ a = − 10
−b ± D (For 1st eqn.)
x= . P = 3 × 3 ⇒ 9 = b ⇒ b = 9 (For 2nd eqn.)
2a
∴ x2 − 10x + 9 = 0 ⇒ x = 9, 1.
7. If the quadratic equation
2. (A) For equal roots, D = 0.
x2 + β(4x + β – 1) + 2 = 0, that is x2 + 4βx +
β2 – β + 2 = 0 has real and equal roots, then ∴ 64k2 – 4 × 9 × 16 = 0 ⇒ k = ± 3.
D = 0. i.e., b2 – 4ac = 0 3. No.
Here, a = 1, b = 4β, c = (β2 – β + 2) At x = 1, x2 + x + 1 = 12 + 1 + 1 = 3 ≠ 0
∴ (4β)2 – 4 × 1 × (β2 – β + 2) = 0 At x = – 1, x2 + x + 1 = (– 1)2 – 1 + 1 = 1 ≠ 0
⇒ 16β2 – 4β2 + 4β – 8 = 0 Hence, neither x = 1 nor x = – 1 is a solution
⇒ 12β2 + 4β – 8 = 0 of the equation x2 + x + 1 = 0.
⇒ 3β2 + β – 2 = 0 a b 2c
4. + =
⇒ 3β2 + 3β – 2β – 2 = 0 x −a x −b x−c
⇒ (β + 1)(3β – 2) = 0 ax − ab + bx − ab 2c
⇒ =
⇒ β + 1 = 0, 3β – 2 = 0 ( x − a)( x − b ) x−c
2 ⇒ (ax + bx – 2ab)(x – c)
⇒ β = – 1, .
3 = 2c(x2 – ax – bx + ab)

Q U A D R A T I C E Q U A T I O N S 155
⇒ (a + b – 2c) x2 – 2abx + bcx + cax = 0 ∴ α′ − β′ = α − β;
⇒ x[(a + b – 2c) x – (2ab – ca – bc)] = 0 ∴ (α′ − β′)2 = (α − β)2
2 ab − ac − bc ∴ (α′ + β′)2 − 4α′ β′ = (α + β)2 − 4αβ
⇒ x = 0 or x = .
a + b − 2c 4B 2 2
⇒ − 4 . C = 4b − 4 . c
5. x = ± 2 , ± 2 A2 A a2 a
2
Hint: Let y = (5 + 2 6 )x –3
b 2 − ac B2 − AC
1 ⇒ =

2
= (5 – 2 6 )x – 3 a2 A2
y
b2 − ac 2
a .
1 ⇒ =  
∴ y+ = 10 B2 − AC  A 
y
10 ± 96 9. Let ∆ABC is a right-angled
∴ y= triangle such that ∠C = 90° and
2×1 b > a.
(Using: D = b2 – 4ac) ∴ c2 = a2 + b2
⇒ y= 5 ± 2 6. ⇒ a2 + b2 = (3 5 )2 = 45
Now compare the exponent. ⇒ 4a2 + 4b2 = 180 ... (i)
1± 5 Let the new corresponding sides be a', b'
6. x = and c' such that
2
Hint: Use a2 + b2 = (a – b)2 + 2ab a' = 3a, b' = 2b and c' = 15 cm
2
Then, (3a)2 + (2b)2 = (15)2
 x   x 
x –  + 2x  x + 1  = 3 ⇒ 9a2 + 4b2 = 225 ... (ii)
 x +1 Subtracting equation (i) from equation (ii),
2 we have
 x2 + x − x  x2
⇒   +2 = 3 5a2 = 45 ⇒ a = 3
 x +1  x+1
Substituting a = 3 in equation (ii), we have
 x2 
2
 x2  9 × 9 + 4b2 = 225
⇒   +2 = 3 ⇒ 4b2 = 225 – 81 = 144 ⇒ b = 6
 x +1  x + 1
Hence, the original length of sides are 3 cm,
x2 5 cm and 3 5 cm.
Let y=
x+1 OR
⇒ y2 + 2y – 3 = 0 According to the question, the two times:
⇒ y = 1 or y = – 3 (i) t minutes past 2 p.m. and
x2 x2
⇒ = 1 or = –3  2 
x+1 x+1 (ii) 60 −  t − 3 minutes past 2 p.m. are
 4 
x2 – x – 1 = 0 or x2 + 3x + 3 = 0
equal. It means
Now solve.
 2 
7. Hint: Take a = 1 + m2, b = 2mc, c = c2 − a2 t = 60 −  t − 3
 4 
Use D = b2 − 4 ac = 0.
8. Hint: Let roots of t2
⇒ t+ − 3 = 60
Ax2 + 2Bx + C = 0 be α′ and β′ 4
∴ α′ = α + δ; ⇒ t2 + 4t – 252 = 0
⇒ 2
t + 18t – 14t – 252 = 0
β′ = β + δ

156 M A T H E M A T I C S – X
⇒ t(t + 18) – 14(t + 18) = 0 Squaring both sides of (i) and using (ii)
⇒ (t + 18)(t – 14) = 0 you will get the result.
⇒ t = – 18 (rejected), t = 14. 8. The given equation is
x −1 x − 3 1
WORKSHEET – 20 + = 3
x−2 x−4 3
1. (A) The wrong equation is x2 + 17x + q = 0 ( x − 1)( x − 4) + ( x − 2)( x − 3) 10
∴ q = (– 2) × (– 15) = 30 ⇒ =
( x − 2)( x − 4) 3
Now, the original equation will be
x2 + 13x + 30 = 0. Its roots are – 10, – 3. x2 − 5x + 4 + x2 − 5x + 6 10
⇒ 2
=
2 x − 6x + 8 3
2. (D) x = must satisfy kx2 – x – 2 = 0
3 x2 − 5x + 5 5
⇒ 2
=
4 2 x − 6x + 8 3
∴k× − – 2 = 0 ⇒ k = 6.
9 3 ⇒ 5x2 – 30x + 40 = 3x2 – 15x + 15
3. (C) ⇒ 2x2 – 15x + 25 = 0
Hint: For no real root D < 0. Let us use quadratic formula.
4. x2 + p(2x + 4) + 12 =0 − b ± b 2 − 4 ac 15 ± 225 − 4 × 2 × 25
⇒ x2 + 2px + 4p + 12 =0 x= =
2a 2×2
For real and equal roots, D =0
∴ 4p2 – 4 × (4p + 12) =0 15 ± 5 5
⇒x= ⇒ x = 5, .
⇒ 4p2 – 16p – 48 =0 4 2
⇒ 4(p – 6) (p + 2) =0 9. Yes, 25 m and 16 m
⇒ p – 6 = 0 or p + 2 =0 Hint: Let the two adjacent sides of the field
⇒ p = 6 or – 2. be a and b.
c Then 2(a + b) = 82 ⇒ a + b = 41
5. Product of roots =
a And ab = 400.
1 −q OR
A
⇒ × (− 2) = 3 cm and 9 cm
2 p+1
Hint: Let smaller leg = x 3 10 cm
⇒ − p −1 = − q y
From figure,
⇒ q−p=1 ... (i)
x2 + y2 = (3 10 )2 = 90 B x C
1 3
Also sum of roots = − 2 = ⇒ y2 = 90 − x2 ... (i)
2 p+1
Also (3x)2 + (2y)2 = (9 5 )2
1 1 ⇒ 9x2 + 4y2 = 405 ... (ii)
⇒ − = ⇒p=−3 ... (ii)
2 p+1 Use (i) and (ii) and then solve.
∴ From (i), q = −2
ASSESSMENT SHEET – 1
∴ p + q + 5 = − 3 − 2 + 5 = 0.
1. (C) D = ( − 5 ) – 4 × 2 × 1 = 5 – 8 = – 3 < 0.
2
6. Hint: Use D ≥ 0 for both the equation.
Given equation has no real roots.
b
7. Hint: Use sin α + cos α = – ... (i) 2. (C) 3x2 + 2 6x + 2 + x2 = 4x2 – 4x
a

and sin α . cos α =


c
... (ii)
i.e., (
6 + 2) x + 1 = 0
a which is not a quadratic equation.

Q U A D R A T I C E Q U A T I O N S 157
3. (x – 1)(x + 2) + 2 = 0 ⇒ x2 + x = 0
3bx 2b 2
⇒ x(x + 1) = 0 ⇒ x = 0 or –1 x2 – + 2 =0
a a
So, roots are 0 and –1.
1 3b
4. False, because a quadratic equation having Here, of coefficient of x is –
2 2a
negative discriminant has no real root.
2
Adding both the sides to  − 3b  , we get
5. 3x2 + 5 5x – 10 = 0 ... (i)  2a 
We divide 5 5x into two parts such that 3bx  3b  2 2b 2 2
3b
sum and product of them are 5 5x and x2 – +  −  + 2 =  − 
a  2a  a  2a 
– 30x2 respectively.
2
 3b  9b 2 2b 2 b2
Such parts are 6 5x and − 5x so equation (i) i.e.,  x −  = 2 − 2 =
forms as follows:  2a  4a a 4 a2
2 2
3x2 + 6 5x – 5x – 10 = 0 i.e.,  x − 3b  =  b 
3x ( x + 2 5 ) –  2a   2a 
or 5 (x + 2 5 ) = 0
3b b
or ( x + 2 5 )(3 x − 5) = 0 i.e., x − =±
2a 2a
i.e., x+ 2 5=0 or 3x – 5= 0
3b b 3b b
i.e., x − = or x − =−
5 2a 2a 2a 2a
i.e., x = – 2 5 or x=
3 2b b
i.e., x = or x =
Hence, the roots of equation a a

5 Hence, the roots of the given equation are


3x2 + 5 5 x − 10 = 0 are − 2 5 and .
3 2b b
6. If a quadratic equation has equal real roots, a and a .
then its discriminant vanishes. 8. Let the speed of the faster train be x km/hr
i.e., D= 0 and that of the slower train be y km/hr.
or b2 – 4ac = 0 Distance
Time =
From the given equation, Speed
a = k – 12; b = 2(k – 12); c = 2
Time taken by the slower train – Time taken
So, b2 – 4ac = 0 provides by the faster train = 3 hrs.
{2(k – 12)}2 – 4(k – 12) × 2 = 0
600 600
i.e., 4(k2 – 24k + 144) – 8k + 96 = 0 ⇒ – = 3 ... (i)
y x
i.e., k2 – 24k + 144 – 2k + 24 = 0
i.e., k2 – 26k + 168 = 0 The speed of the slower train is 10 km/hr
less than that of the faster train
i.e., k2 – 14k – 12k + 168 = 0
i.e., x – y = 10
i.e., k(k – 14) –12(k – 14) = 0 or x = y + 10 ... (ii)
i.e., (k – 14)(k – 12) = 0 Substitute x = y + 10 from equation (ii ) in
i.e., k – 14 = 0 or k – 12 = 0 equation (i) to get
i.e., k = 14 or 12. 600 600
− = 3
But k ≠ 12. ∴ k = 14. y y + 10
7. a2x2 – 3abx + 2b2 = 0 ⇒ 600(y + 10) – 600y = 3(y + 10)y
Dividing throughout by a2, we get ⇒ 200y + 2000 – 200y = y2 + 10y

158 M A T H E M A T I C S – X
⇒ y2 + 10y – 2000 = 0 5. (a – b) x2 + (b – c) x + (c – a) = 0
2
⇒ y + 50y – 40y – 2000 = 0 As this equation has equal roots, the
⇒ y(y + 50) – 40 (y + 50) = 0 discriminant of it vanishes.
i.e., D=0
⇒ (y + 50)(y – 40) = 0
⇒ (b – c)2 – 4 × (a – b) × (c – a) = 0
⇒ y = 40, – 50 ⇒ b2 – 2bc + c2 – 4ac + 4a2
Neglecting y = – 50 because speed is a non- + 4bc – 4ab = 0
negative quantity, we get ⇒ 2 2 2
4a + b + c – 4ab + 2bc – 4ac = 0
y = 40 ⇒ (2a – b – c)2 = 0
Substitute this value of y, i.e., y = 40 [∵ (x + y + z) = x + y2 + z2 + 2xy
2 2
in equation (ii) we get + 2yz + 2zx]
x = 40 + 10 = 50 ⇒ 2a – b – c = 0
Hence, speed of the faster train = 50 km/hr ⇒ 2a = b + c.
and speed of the slower train = 40 km/hr. Hence proved.

ASSESSMENT SHEET – 2 6. x2 – ( 3 + 1) x + 3 = 0 ... (i)


2
2 Coefficient of x = – ( 3 + 1)
1. (B) 7  2  + t   – 3 = 0
 3  3 3 +1
Half of coefficient of x = –
28 2 2
⇒ + t – 3= 0
9 3 2

3 28  Adding  − 3 + 1  to both the sides of


 
⇒ t=  3 −  2 
2 9
equation (i), we get
3 1 1 2
⇒ t= – × = − .  3 + 1 +
2 9 6 x2 – ( 3 + 1) x +  −  3
 2 
2. (A) Let us find the discriminant of:
2
x2 + 2 3x – 1 = 0  3 + 1
= − 
 2 
D = ( 2 3x ) – 4 × 1 × (–1) = 12 + 4 = 16 > 0
2

2
 3 + 1 3 + 1+ 2 3
⇒ x2 + 2 3x – 1 = 0 has real roots. ⇒ x −  = – 3
 2  4
3. Consider x2 + 5px + 16 has no real roots.
2
⇒ D<0  3 + 1 3 + 1− 2 3
⇒ x −  =
⇒ (5p)2 – 4 × 1 × 16 < 0  2  4

⇒ 25p2 < 64 ⇒ p < ± 64  3 + 1  3 − 1


2 2

25 ⇒ x −  =  
 2  2 
8 8
⇒ − <p< .
5 5 3 +1 3 −1
⇒ x– = ±
4. True. 2 2
Let equation is ax2 + bx + c = 0 3 +1 3 −1 3 +1 3 −1
Case I. a > 0 and c < 0 ⇒ ac < 0 ⇒ – ac > 0 i.e., x = + , –
2 2 2 2
∴ D = b2 – 4ac > 0 ∴ b2 ≥ 0
i.e., x = 3 or x = 1.
Case II. a < 0 and c > 0 ⇒ ac < 0 ⇒ – ac > 0
∴ D = b2 – 4ac > 0. Hence, x = 3 , 1.

Q U A D R A T I C E Q U A T I O N S 159
7. Given equation is According to the second given condition
a2b2x2 – (4b4 – 3a4)x – 12a2b2 = 0 ... (i) 2x – y = 10y – 1
General quadratic equation is 2x – 11y + 1 = 0 ... (ii)
Ax2 + Bx + C = 0 ... (ii) Putting x = 2 + y2 from (i) in equation (ii),
Comparing the coefficients of like powers of we get
x of equations (i) and (ii), we get 2(2 + y)2 – 11y + 1 = 0
A = a2b2; B = – (4b4 – 3a4); C = –12a2b2 ⇒ 4 + 2y2 – 11y + 1 = 0
Now, ⇒ 2y2 – 11y + 5 = 0
D = B2 – 4AC ⇒ 2y2 – 10y – y + 5 = 0
= {– (4b4 – 3a4)}2 – 4a2b2 (– 12a2b2)
⇒ 2y (y – 5) – 1 (y – 5) = 0
= 16b8 – 24b4a4 + 9a8 + 48a4b4
⇒ (y – 5)(2y – 1) = 0
= 16b8 + 9a8 + 24a4b4
1
= (4b4 + 3a4)2. ⇒ y = 5 or
2
Using the quadratic formula to solve
Let us use equation (i).
equation (i), we have
2 9
1
−B ± D When y = , x = 2 +  1  = = 2.25
x= 2  2 4
2A
But at the age of 2.25 years, Sumita cannot
4b 4 − 3 a 4 ± (4b 4 + 3 a 4 )2
= 1
2a2 b2 be mother. So, we must reject y = years.
2
4b 4 − 3b 4 ± (4b 4 + 3 a 4 ) When y = 5, x = 2 + 52 = 27
= Hence, present age of Sumita is 27 years
2 a2 b 2
and Riya is 5 years.
4b 4 − 3 a 4 + 4b 4 + 3 a 4
i.e., x = ,
2 a2 b 2 CHAPTER TEST
4b 4 − 3 a 4 − 4b 4 − 3 a 4 1. (C) Let us consider option (C).
2 2
2a b ⇒ 2x2 + 3 + 2 6x + x2 = 3x2 – 5x
8b 4 − 6a4 ⇒ (5 + 2 6 ) x + 3 = 0
i.e., x = ,
2 2
2a b 2 2
2a b which is not a quadratic equation.

4b 4 3a 2 3
i.e., x = ,– . 2. (B) 9x2 + x– 2 =0
4
a2 b2
1
8. Let the present age of Sumita be x years Let us add and subtract .
and that of her daughter Riya be y years. 64
According to the first given condition, 3 1 1
9x2 + x+ − − 2=0
x = 2 + y2 ... (i) 4 64 64
2
Riya would reach at the age of x years after 2  
(x – y) years. ⇒  3 x + 1  −  1 + 64 2  = 0
 8   
 8 
After (x – y) years, Sumita’s age
= x–y+x 1
Clearly, the required number is .
= (2x – y) years 64

160 M A T H E M A T I C S – X
3. (C) The given equation can be written as But – 9 is not a natural number.
x4 + x2 + 1 = 0 So, N = 12 is the required natural number.
Here, D = 12 – 4 × 1 × 1 = – 3 < 0 8. (b – c)x2 + (c – a) x + (a – b) = 0
As D < 0, there is no real root. A quadratic equation is a perfect square, if
4. 2x2 – kx + k = 0 has equal roots, if discrimi- its discriminant (D) is equal to zero.
nant = 0. Here, A = b – c, B = c – a and C = a – b
⇒ (– k)2 – 4 × 2 × k = 0 ⇒ k(k – 8) = 0 Now, D = 0
⇒ k = 0 or 8. ⇒ D = B2 – 4AC = (c – a)2 – 4(b – c)(a – b)
5. True. ⇒ c2 + a2 – 2ca – 4 (ab – b2 – ca + bc) = 0
Let us consider a quadratic equation ⇒ c2 + a2 + 4b2 + 2ca – 4bc – 4ab = 0
⇒ (c + a – 2b)2 = 0
3 x 2 − 7 3 x + 12 3 = 0 [∵ (x + y + z)2 = x2 + y2 + z2 + 2xy
Here, D = ( – 7 3 ) – 4 × 3 × 12 3
2
+ 2yz + 2zx]
⇒ D = 147 – 144 = 3 a+c
⇒ c + a – 2b = 0 ⇒ b = .
⇒ D>0 2
⇒ Roots are real and distinct. Hence proved.
9. Let the thickness of the region having the
7 3 ± 3 grass be x metres.
So, x = ∴ x = 4, 3 which are
2 3 From the adjoining figure, we have
rationals. AB = PQ = DC = SR = 50 m
OR AP = BQ = KQ = x m
No. KL = QR = JM = PS = (40 – 2x) m
(x – 1)2 + (2x + 1) = 0
⇒ x2 – 2x + 1 + 2x + 1 = 0
⇒ x2 + 2 = 0
Here, D = 02 – 4 × 1 × 2 = – 8 < 0.
Hence, the given equation has no real root.
1 2 Now, area of the grass field = 1184
6. x − 11x + 1 = 0
2 ⇒ AB × AP × 2 + QR × KQ × 2 =1184
1
Here, a = , b = − 11 , c = 1 ⇒ 50 × x × 2 + (40 – 2x) × x × 2 = 1184
2
⇒ 180x – 4x2 = 1184
D = b2 – 4ac
⇒ x2 – 45x + 296 = 0
1
( )
2 2
= − 11 –4× ×1=9 ⇒ x – 37x – 8x + 296 = 0
2 ⇒ (x – 37)(x – 8) = 0

x=
−b ± D
=
− − 11 ± 3 ( ) ⇒ x = 8 or x = 37
2×1 But x = 37 is not possible as 2x > 50
2a
∴ x= 8m
11 + 3 11 − 3 ∴ Length of the pond
α= , β= .
2 2 = JK = 50 – 16 = 34 m
7. Let the required natural number be N. And breadth of the pond = JM
N2 – 84 = (N + 8) × 3 ⇒ N2 – 3N – 108 = 0 = 40 – 16 = 24 m.
⇒ (N – 12)(N + 9) = 0 ⇒ N = 12 or – 9 ❑❑
Q U A D R A T I C E Q U A T I O N S 161
Chapter

2 ARITHMETIC PROGRESSIONS

WORKSHEET– 24 1
Third term = a + 2d = – 1 + 2 × =0
1. (D) Let us consider option (D). 2
2nd term – 1st term = – 6 – (–10) = 4 1 1
Fourth term = a + 3d = – 1 + 3 × =
3rd term – 2nd term = – 2 – (– 6) = 4 2 2
4th term – 3rd term = 2 – (– 2) = 4
1
∴ – 10, – 6, – 2, 2,........ is an A.P. Hence, the first four terms are: – 1, – , 0, 1 .
2 2
2. (B) 11th term of the A.P. – 62, – 59, – 56, ..., 7,
10 is – 62 + (11 – 1) × 3, i.e., – 32. 7. Let the first term be a and the common
difference be d.
3. (A) nth term = – 15 + (n – 1) × (– 3)
A.P. = a, a + d, a + 2d,........
= – 15 – 3n + 3 = – 3n – 12.
According to question,
4. ∵ Tn = 7 – 4n T3 = 16 and T7 = 12 + T5
∴ Tn – 1 = 7 – 4(n – 1) = 11 – 4n ⇒ a + 2d = 16 and a + 6d = 12 + a + 4d
Now, d = Tn – Tn – 1 = 7 – 4n – 11 + 4n ⇒ a + 2d = 16 and d =6
= – 4. ⇒ a = 4 and d =6
5. False So, the required A.P. will be 4, 10, 16, ......
Let first term be a and common difference 8. (i) Here, a = 9, d = 17 – 9 = 8
be d of an A.P.
Let 636 be the sum of n term of this A.P.
Then
where (n ∈ N)
3rd term, a + 2d = 4 ... (i)
th n
9 term, a + 8d = – 8 ... (ii) Let’s use Sn = [2a + (n – 1)d]
2
Subtracting equation (ii) from (i), we get
n
⇒ – 6d = 12 636 = [2 × 9 + (n – 1)8]
2
⇒ d = –2
Putting d = – 2 in (i), we get n
⇒ 636 =[18 + 8n – 8]
... a= 8 2
.
. . Given a + (n – 1)d = 0 n
⇒ 636 = [10 + 8n]
⇒ 8 + (n – 1) (– 2) = 0 2
⇒ – 2n = – 10 ∴ n = 5 n
⇒ 636 = × 2[5 + 4n]
th
5 term = a + (n – 1)d 2
= 8 + (5 – 1) (– 2) ⇒ 4n2 + 5n – 636 = 0
=0 Using quadratic formula
Thus its 5th term is 0. So the given statement
is false. – 5 ± 52 – 4.4. (– 636)
n=
2.4
6. First term =a=–1
1 1 – 5 ± 10201
Second term = a + d = – 1 + =– =
2 2 8

162 M A T H E M A T I C S – X
– 5 ± 101 15  6 28  17 × 15
= = ×  + =
8 2  5 5 5
96 – 106 = 51 5 .
= ,
8 8
2. (C)
∴ n = 12, – 13.25
Rejecting n = –13.25 due to negative Hint: Let an = –22
value. .
. . 5 + (n – 1)(– 3) = – 22
∴ n = 12 ⇒ – 3n = – 30 ⇒ n = 10 ∴ a10 = – 22.
Thus the number of terms will be 12. 3. (B) Common difference = – 2 – 1 = – 5 – (– 2)
33 = – 3.
(ii)
20 4. p = 4
1 Hint: Use: if a, b, c are in A.P. ⇒ 2b = a + c.
Hint : d=
60 5. False
1  1  In an A.P., having nth term an = a + (n – 1)d,
∴ an = + (n − 1)   we know that an is a linear polynomial in n.
15  60 
Here a n = n 2 + n + 1 is not a linear
1 1 polynomial in n. So it can't be nth term of
∴ an = + 10 ×
15 60 an A.P.
4 + 10 14 7 6. Let the first term be a and the common
= = = difference be d.
60 60 30
th
21 + 1 
∴ S11 =
11 1
+
2 15 30
7
{=
11 × 9
2 × 30 }
=
33
20
.
Middle term = 
 2 
th
 term = 11 term.

Sum of three middle terms:


9. ` 80n where n = 1, 2, 3, ..., 30; yes; ` 2400 a10 + a11 + a12 = 129
PRn ⇒ (a + 9d) + (a + 10d) + (a + 11d) = 129
Hint: S.I. = ⇒ S.I. = 80n
100 ⇒ 3a + 30d = 129 ... (i)
Interest at the end of 30 years = ` 2400. Sum of last three terms:
OR a19 + a20 + a21 = 237
Sn = 55350 ⇒ (a + 18d) + (a + 19d) + (a + 20d) = 237
Hint: Sequence is: 3a + 57d = 237
...(ii)
108, 117, 126, ......, 999
Subtracting equation (i) from equation (ii),
∴ an = 108 + (n – 1) . 9
we have
⇒ 999 – 108 = 9(n – 1) 27d = 108 ⇒ d = 4
⇒ 891 = 9(n – 1) ⇒ n = 100 Substituting d = 4 in equation (i), we have
100 129 – 120
∴ Sn = (108 + 999) = 50 × 1107 a = =3
2 3
= 55350. So the required A.P. will be 3, 7, 11, ...
OR
WORKSHEET – 25
a5 + a9 = 72 ⇒ 2a + 12d = 72 ... (i)
15  3  3  also a7 + a12 = 97 ⇒ 2a + 17d = 97 ... (ii)
1. (A) S15 = 2 × + (15 – 1) ×  5 – 
2  5  5  (i) – (ii) ⇒ − 5d = − 25

A R I T H M E T I C P R O G R E S S I O N S 163
⇒ d= 5 Substituting n = 10 in an = 10n – 8, we get
From (i), ⇒ a= 6 10th term = 10 × 10 – 8 = 92.
∴ A.P. is 6, 11, 16,....
WORKSHEET – 26
7. Sn = 2n – 3n2
Hint: a1 = 5–6=–1 1 5
1. (A) a = – 3, d = – – (– 3) =
a2 = 5 – 12 = – 7 2 2
a3 = 5 – 18 = – 13 5
∴ a11 = a + (11 – 1)d = – 3 + 10 × = 22.
.
.. d= a2 – a1 = – 6 2
n 2. (B) Let nth term be – 81.
... Sn = {− 2 + (n − 1) ( − 6 )} ∴ 21 + (n – 1)(– 3) = – 81
2
n ⇒ 24 – 3n = – 81
= {4 − 6n}
2 ⇒ n = 35.
= n(2 – 3n) = 2n – 3n2. 3. (C) a18 – a13 = a + 17d – a – 12d = 5d
8. a=7 = 5 × 5 = 25.
d = 11 – 7 = 4. 4. The given A.P. is 17, 14, ......, – 34, – 37, – 40.
Let an = 111 5th term from the end = – 40 + (5 – 1) × 3
⇒ a + (n – 1) × d = 111 = – 40 + 12 = – 28.
⇒ 7 + (n – 1) × 4 = 111 5. 178
⇒ 7 + 4n – 4 = 111
Hint: Let the first term be a and the common
⇒ 4n = 108 difference be d.
⇒ n = 27 a + 10d = 38 and a + 15d = 73.
27
... Sn = S27 = (7 + 111) OR
2
The given A.P. is 8 , 18 , 32 , ......
27
= × 118 or 2 2 , 3 2 , 4 2 , .........
2
= 27 × 59 Here d = 3 2 – 2 2 = 2
= 1593. Hence, the next two terms will be 4 2 + 2
9. 60°, 80°, 100°, 120° and 4 2 + 2 2 or 5 2 and 6 2 .
Hint: Let the angles be: 6. As 3k + 4, 7k + 1, 12k − 5 are in A.P.
a − 3d, a − d, a + d, a + 3d. ⇒ 2 × (7k + 1) = 3k + 4 + 12k − 5
OR ⇒ 14k + 2 = 15k − 1
∵ Sn = 5n2 – 3n ⇒ 2 + 1 = 15k − 14k
∴ Sn–1 = 5(n – 1)2 – 3(n – 1) ⇒ 3=k
= 5n2 + 5 – 10n – 3n + 3 Thus, value of k = 3.
= 5n2 – 13n + 8 7. n = 4 or 13
th
n term (an) = Sn – Sn – 1 n
= 5n2 – 3n – (5n2 – 13n + 8)
Hint: Use S n =
2
{ }
2 a + ( n − 1) × d .
= 10n – 8 8. Let the first term and the common difference
a1 = 10 × 1 – 8 = 2 be a and d respectively.
a2 = 10 × 2 – 8 = 12 Second term = 14
a3 = 10 × 3 – 8 = 22 ⇒ a + (2 – 1) × d = 14 [∵ an = a + (n – 1)d]
Therefore, the A.P. is 2, 12, 22,....... ⇒ a + d = 14 ...(i)

164 M A T H E M A T I C S – X
Third term = 18 511
⇒ a + (3 – 1)d = 18 ⇒ n= = 73
7
⇒ a + 2d = 18 ...(ii) th
n + 1
Solving equations (i) and (ii), we obtain ∴ Middle term is   = 37
th

d = 4 and a = 10.  2 
Sum of first 51 terms ∴ a37 = 1 + 36 × 7 = 253.
51 3. (A) 2 (p +10) = 2p + 3p + 2
S51 = [2a + (51 – 1)d]
2 ⇒ 2p + 20 = 5p + 2

{ Sn =
n
2
[2a + (n – 1)d ] } ⇒

18 = 3p
p = 6.

51 1 19
= [2 × 10 + 50 × 4] 4. a4 = 9 =
2 2 2

51 19
= × 220 = 5610. ⇒ a + 3d =
2 2

9. The sequence of all months' savings (in rupees) 19


⇒ 5 + 3d =
is 320, 360, 400, ....... 2
Let Sn = 20000 3
⇒ d =
n
⇒ 20000 =
2
{2a + ( n − 1) × d } 2
1 13
Using a = 320 ∴ First box = 6 =
2 2
d = 40,
we get Second box = 8.
n
20000 =
2
{
640 + ( n − 1) × 40 } 5. 15th term from end of – 10, – 20, – 30, ............,
– 980, – 990, – 1000
⇒ 40000 = n {40n + 600} th
= 15 term of – 1000, – 990, – 980, .............,
⇒ 2
40n + 600n − 40000 = 0 – 20, – 10
⇒ 4n2 + 60n − 4000 = 0 = –1000 + (15 – 1) × (– 990 + 1000)
⇒ n2 + 15n − 1000 = 0 = – 1000 + 140 = – 860.
⇒ 2
n + 40n − 25n − 1000 = 0 6. 6n – 1
⇒ n(n + 40) − 25(n + 40) = 0 Hint: Use
⇒ n = 25 or − 40 (Reject) an = Sn − Sn– 1
... n = 25 months. 7. Hint: Use S20 = S30 and show that S50 = 0.

WORKSHEET – 27 8. A.P.: 63, 65, 67, ...........


a = 63, d = 65 – 63 = 2
1. (D) 5a5 = 10a10 ∴ nth term of 63, 65, 67,.....
⇒ 5(a + 4d) = 10(a + 9d) = a + (n – 1) × d = 63 + (n – 1) × 2 ... (i)
⇒ 5a = – 70d ⇒ a = – 14d A.P.: 3, 10, 17,.........
Now a15 = – 14d + 14d = 0. a′ = 3, d′ = 10 – 3 = 7
2. (B) an = 505 ⇒ a + (n – 1) × d = 505 ∴ nth term of 3, 10, 17,.......
⇒ 1 + 7n – 7 = 505 = a′ + (n – 1)d′ = 3 + (n – 1) × 7 ... (ii)

A R I T H M E T I C P R O G R E S S I O N S 165
According to the question, 2. (A) We need to obtain the sum:
63 + (n – 1) × 2 = 3 + (n – 1) × 7 1 + 3 + 5 +............+ 49
[Using (i) and (ii)] an = a + (n – 1)d ⇒ 49 = 1 + (n – 1) × 2
⇒ 2n – 2 – 7n + 7 = 3 – 63 ⇒ n = 25
⇒ – 5n + 5 = – 60 n 25
∴ Sn = (a + an) = × (1 + 49) = 625.
⇒ – 5n = – 65 ⇒ n = 13. 2 2
9. The sequence of savings (in rupees) is 3. (D) an = a + (n – 1) × d
4, 5.75, 7.5, ........, 19.75 ⇒ 50 = 5 + (n – 1) × 3
Here, a = 4 50 – 5
⇒ n – 1= ⇒ n = 16.
d = 1.75 3
an = 19.75 4. Let missing terms be x and y such that 2, x,
⇒ a + (n − 1) × d = 19.75 26, y are in A.P.
⇒ 4 + (n − 1) × (1.75) = 19.75 Common difference = x – 2 = 26 – x = y – 26
⇒ 4 + 1.75n − 1.75 = 19.75 28
∴ x= = 14 and
⇒ 1.75n = 19.75 − 2.25 2
⇒ 1.75n = 17.50 y = 26 + 26 – 14 = 38.
⇒ n = 10 5. a9 = –10 ⇒ a + 8d = – 10
. th
. . In 10 week her saving will be ` 19.75. 5
OR ⇒a+8× = –10 ⇒ a = – 20
4
Let first term be a and common difference
be d. 5
Now, a27 = a + 26d = –20 + 26  
According to question, 4
a4 + a8 = 24 65 25 1
= – 20 + = = 12 .
⇒ a + 3d + a + 7d = 24 2 2 2
a + 5d = 12 ... (i) 6. 5, 7, 9
and a6 + a10 = 44 Hint: Let the three numbers be
⇒ a + 5d + a + 9d = 44 a − d, a, a + d.
a + 7d = 22 ... (ii) 7. The sequence of such numbers is:
Subtracting (i) from (ii), we get 101, 108, 115, .............., 997
2d = 10 ∴ an = 997 ⇒ 101 + (n − 1) × 7 = 997
∴ d= 5
129
Putting d = 5 in (i), we get ⇒ n = 129 ∴ S129 = {101 + 997}
2
a = –13
= 129 × 549 = 70821.
∴ First three terms of this A.P. will be – 13,
– 8, – 3. 8. S24 = 672
Hint: a1 = 5,
WORKSHEET – 28 a2 = 7,
... d= 2
3a + 2a – 7
1. (B) =a+4 ... S24 = 12(10 + 46) = 672.
2
⇒ 5a – 7 = 2a + 8 9. ` 40, ` 60, ` 80, ` 100
⇒ 3a = 15 Hint: Let the prizes (in `) be x, x + 20,
⇒ a = 5. x + 40, and x + 60.

166 M A T H E M A T I C S – X
OR ⇒ 103 + (n − 1) × ( − 2) = 49
According to the given question, we have ⇒ − 2n = 49 − 105
a= 8 ...(i) ⇒ n = 28
an = 33 ⇒ a + (n – 1)d = 33 ...(ii) 28
n S = {103 + 49)
Sn = 123 ⇒ (a + an) = 123 ...(iii) 2
2
= 14 × 152 = 2128.
Substituting the value of a from equation (i),
and that of an from equation (ii) in equation 6. –1, 4, 740
(iii), we get Hint: a3 = 7
n 123 × 2 a7 = 3 × a3 + 2.
(8 + 33) = 123 ⇒ n = ⇒ n=6
2 41 7. Let the first term be a and the common
Substituting a = 8 and n = 6 in equation (ii), difference be d.
we get n 6
∵ Sn = [2a + (n – 1)d] ∴ 42 = [2a + 5d]
33 – 8 2 2
8 + (6 – 1) × d = 33 ⇒ d = ⇒d=5 ⇒ 2a + 5d = 14 ...(i)
5
Thus, n = 6 and d = 5. a + 9d 1
∵ an = a + (n – 1)d ∴ =
a + 29d 3
WORKSHEET – 29 ⇒ 3a + 27d = a + 29d ⇒ 2a – 2d = 0 ⇒ a = d
1. (A) Hint: 12, 16, 20,...., 248 Substituting a = d in equation (i), we have
... 248 = 12 + (n – 1)4 d = 2 and so a = 2
236 Now, a18 = 2 + 17d = 2 + 17 × 2 = 36.
⇒ =n–1 Hence the first term is 2 and 18th term is 36.
4
⇒ n = 60. 8. Hint: mam = nan
⇒ m{a + (m – 1)d} = n{a + (n – 1)d}
n
2. (B) Sn = 90 ⇒ 90 =
2
{
4 + ( n − 1) × 8 } ⇒ m{a + (m – 1)d} – n{a + (n – 1) d} = 0
⇒ a(m – n) + {m(m – 1) – n(n – 1)} d = 0
⇒ 180 = n (8n − 4)
2 ⇒ a(m – n) + {(m2 – n2) – (m – n} d = 0
⇒ 2n − n − 45 = 0
⇒ a(m – n) + {(m – n) (m + n – 1) d = 0
⇒ (2n + 9) (n − 5) = 0
⇒ a + (m + n – 1) d = 0 {... m ≠ n}
⇒ n= 5
⇒ am + n = 0.
∴ an = a5 = a + 4 × d = 2 + 4 × 8 = 34.
9. The penalty (in `) is in A.P. as follows:
3. (C) an = an′
200, 250, 300, ........... 30 terms
⇒ 63 + (n − 1) × 2 = 3 + (n − 1) × 7 So, the contractor has to pay the penalty
⇒ 2n + 61 = 7n − 4 (in `) as the sum of the following series:
⇒ 5n = 65 200 + 250 + 300 + ............30 terms
⇒ n= 13. Here, a = 200, d = 250 – 200 = 50, n = 30
4. Common difference = 2p – 1 – p n
∴ Sum = [ 2a + (n – 1)d]
= 7 – (2p – 1) 2
∴ 2p – p + 2p = 7+1+1 30
= [ 2 × 200 + 29 × 50]
⇒ 3p = 9 ⇒ p = 3. 2
5. a = 103; d = 101 − 103 = −2 = 15(400 + 1450) = 27750
... an = 49 Hence, the required penalty is ` 27750.

A R I T H M E T I C P R O G R E S S I O N S 167
WORKSHEET – 30 6. 30
Hint: Let Sn < 0.
1. (B)
Hint: The sequence is: 1 5
7. − ,
3, 9, 15, ......., 99. 2 2
n Hint: a4 + a8 = 24
Use: Sn = {a + l} .
2 ⇒ 2a + 10d = 24 ...(i)
and a6 + a10 = 34
2. (A) an = Sn – Sn – 1
⇒ 2a + 14d = 34 ...(ii)
= (2n2 + 5n) – [2(n – 1)2 + 5(n – 1)]
Solve (i) and (ii).
= 2n2 + 5n – 2(n2 + 1 – 2n) – 5n + 5
8. Let the first term be a and the common
= 2n2 + 5n – 2n2 – 2 + 4n – 5n + 5 difference be d.
= 4n + 3. a3 = 7
⇒ a + 2d = 7 ...(i)
5 1 2  5 ∵ a7 = 3a3 + 2
3. Here, – – (– 1), i.e. and – – –  ,
6 6 3  6 ⇒ a + 6d = 3 × 7 + 2
a + 6d = 23 ...(ii)
1
i.e., are equal. So, the given sequence is Using equations (i) and (ii), we get
6
4d = 16 ⇒ d= 4 ...(iii)
1 Substituting d = 4 in (i), we have
in A.P. with common difference .
6 a + 2 × 4= 7 ⇒ a= –1 ...(iv)
Therefore, the next three terms will be Now, using the formula of sum of n terms:
n
2 1 2 1 2 1 Sn = [2a + (n – 1)d]
– + , – + 2 × and – + 3 × 2
3 6 3 6 3 6
20
∴ S20 = [2 × (– 1) + (20 – 1) × 4]
1 1 1 2
These are – , – and – .
2 3 6 = 10(– 2 + 76) [Using (iii) and (iv)]
4. ∵ 8x + 4, 6x – 2 and 2x + 7 are in A.P. = 740.
∴ 6x – 2 – (8x + 4) = 2x + 7 – (6x – 2) Sn 2 + 3n
9. =
⇒ – 2x – 6 = – 4x + 9 Sn′ 3 + 2n
⇒ 2x = 15
2 a + ( n − 1) × d 2 + 3n
⇒ =
15 2a′ + ( n − 1) × d′ 3 + 2n
⇒ x= .
2
5. 3, 9, 15  n −1
a+ × d
 2  2 + 3n
Hint: Let the three numbers be ⇒ = ...(i)
 n −1 ′ 3 + 2n
a − d, a, a + d. a′ +  × d
 2 
OR
a17 = a10 + 7 a7 a + 6d
... To obtain , i.e.,
⇒ a + 16d = a + 9d + 7 a7′ a′ + 6 d′
⇒ 7d = 7 n−1
replace by 6 i.e. n by 13
⇒ d = 1. 2

168 M A T H E M A T I C S – X
We get from (i), 2A + (n – 1)D 7n + 1
⇒ =
a + 6d 2 + 3 (13) a7 41 2a + (n – 1) d 4n + 27
= ⇒ =
a′ + 6 d′ 3 + 2 (13) a7′ 29
 n – 1
Thus, the ratio of 7th terms is 41 : 29. A+ D
⇒  2  = 7n + 1
WORKSHEET – 31  n – 1 4n + 27
a+ d
 2 
1. (C) a1 = x; a2 = y; l = 2x
To prepare the 5th term in numerator and
∴ d= y–x denominator of LHS of this last equation,
∴ 2x = x + (n – 1) .(y – x)
n–1
x we should put = 4, i.e. n = 9.
⇒ = n–1 2
y−x
Therefore,
x+y−x y
n= = A + 4D 7×9+1 A 64
y–x y–x = ⇒ 5 =
a + 4d 4 × 9 + 27 a5 63
y 1 3xy
∴ Sn = × ( x + 2x ) = . Hence, the required ratio is 64 : 63.
y−x 2 2( y − x)
7. Hint: Use a′ + (p – 1)d = a
2. (B) 10th term from end of 4, 9,....., 244, 249,
a′ + (q – 1)d = b
254
th a′ + (r – 1)d = c.
= 10 term from begining of 254, 249, 244,
...., 9, 4. OR
= 254 + 9 × (–5) = 254 – 45 = 209. Let the first term be a and the common
difference be d.
3. (D) 105, 112, 119, ........., 994 Now, a19 = 3 × a6 ⇒ a + 18d = 3 (a + 5d)
an = a + (n – 1)d 2a = 3d ...(i)
⇒ 994 = 105 + (n – 1) × 7 Also, a9 = 19 ⇒ a + 8d = 19 ...(ii)
⇒ 994 = 105 + 7n – 7 From equations (i) and (ii),
⇒ 994 = 98 + 7n 3
d + 8d = 19 ⇒ 19d = 38
994 – 98 2
∴ n= = 128.
7 ⇒ d = 2 and so a = 3.
[From equation (i)]
n 9
4. Sn = (a + l) ⇒ 144 = (a + 28) Hence the A.P. is a, a + d, a + 2d, ..........
2 2 i.e., 3, 5, 7, .........
⇒ a + 28 = 32 ⇒ a = 4.
8. As given numbers are in A.P.
5. Yes, ...2 × [2k2 + 3k + 6] = 4k + 8 + 3k2 + 4k + 4
Hint: a30 – a20 = a + 29d – a – 19d ⇒ 4k2 + 6k + 12 = 3k2 + 8k + 12
= 10d = – 40. ⇒ k2 − 2k = 0 ⇒ k (k − 2) = 0
6. Let the first term and common difference of ⇒ k = 0 or k = 2.
first A.P. be A and D respectively and that
of the second A.P. be a and d respectively. 9. ` 7250
Hint: The sequence is 125, 150, 175, .....
n
2
[2A + (n – 1) D ] 7n + 1 Here, a = 125, d = 25, n = 20
= n
n
2
[ 2 a + ( n – 1) d ] 4n + 27 Use Sn = [2a + (n – 1)d].
2

A R I T H M E T I C P R O G R E S S I O N S 169
WORKSHEET – 32 The sum of first 20 terms is
20
1. (A) Sn = 3n2 − n S20 = [2 × 2 + (20 – 1) × 5] = 10 × 99
2
Put n = 1 ∴ S1 = a1 = 2
Put n = 2 ∴ S2 = a1 + a2 = 10 = 990.
⇒ a2 = 8 7. Let the same common difference be d.
∴ d= a2 − a1 = 6. 30th term of one A.P. = 3 + (30 – 1) × d
= 3 + 29d ...(i)
2. (B) th
30 term of other A.P. = 8 + (30 – 1)d
Hint: a + 2d = 4
a + 8d = – 8 = 8 + 29d ...(ii)
– – + Now, the required difference
– 6d = 12
= (8 + 29d) – (3 + 29d)
d = –2
[Using equations (i) and (ii)]
a=8
a10 = a + 9d = 8 + 29d – 3 – 29d = 5.
= 8 – 18 = – 10. 8. Hint:
p
3. (D)
2
{ }
2 a′ + ( p − 1) d = a
Hint: an= Sn − Sn− 1.
q
4. Let the nth term be – 44. 2
{2 a ′ + (q − 1) d} = b
∴ an = – 44 r
⇒ a +(n – 1)d = – 44 {2 a ′ + (r − 1) d} = c
2
⇒ 40 + (n – 1)(– 4) = – 44
OR
⇒ (n – 1) = 21
⇒ n = 22. Let the first term and the common difference
of the given A.P. be a and d respectively.
5. – 8930 5th term = 0 ⇒ a + 4d = 0
Hint: a = − 5; d = − 8 − (− 5) = − 3. ⇒ a= – 4d ...(i)
an = − 230. Find n. 23rd term: a23 = a + 22d
n ⇒ a23 = – 4d + 22d
Then use Sn = {a + l} . [From equation (i)]
2
⇒ a23 = 18d ...(ii)
6. nth term is an = 5n – 3 11th term: a11 = a + 10d
Substituting n = n – 1, we have ⇒ a11 = – 4d + 10d
(n – 1)th term is an –1 = 5 (n – 1) – 3 = 5n – 8 [From equation (i)]
∴ Common difference is d = an – an – 1 a23
⇒ a11 = 6d ⇒ a11 = 6 ×
= 5n – 3 – 5n + 8 18
[From equation (ii)]
=5 ⇒ a23 = 3a11
Substitute n = 1 in an = 5n – 3 to get first
⇒ 23rd term = 3 × 11th term
term
Hence proved.
a1 = 5 × 1 – 3 ⇒ a1 = 2
9. Let the digits of the number be a – d, a and
Now, using a + d such the required number is
n 100(a – d) + 10a + a + d as the digits are in
Sn = [ 2a1 + (n – 1)d]
2 A.P.

170 M A T H E M A T I C S – X
So, the required number = 111a – 99d ...(i) ⇒ 7 + (n − 1) × 3 = 68
Sum of the digits = 15 ⇒ 3n = 64
⇒ a – d + a + a + d = 15 64
⇒ a=5 ...(ii) ⇒ n=
3
The number obtained by reversing the digits which is not a whole number so an = 68 not
= 100(a + d) + 10a + a – d possible.
= 111a + 99d ...(iii) 5. General term is an = (– 1)n 3n + 1
According the given condition, we have Substituting n = 1, 2, 3, 4 successively we get
111a – 99d = 594 + 111a + 99d
a1 = (–1) 32 = – 9, a2 = (–1)2 33 = 27
[Using equation (i) and (iii)]
a3 = (–1)3 34 = – 81, a4 = (–1)4 35 = 243.
⇒ – 2 × 99d = 594
Therefore, first four terms are – 9, 27, – 81,
⇒ d = –3 ...(iv) 243.
Using equations (i), (ii) and (iv), we arrive 6. The sequence is 23, 21, 19, ............, 5
that the original number is
∴ a = 23
111 × 5 – 99 × (–3), that is 852.
d = 21 − (23)
OR = −2
16 rows, 5 logs are placed in top row. ∴ an = 5
Hint: Put Sn = 200, a = 20, d = –1 ⇒ a + (n − 1) × d = 5
n ⇒ 23 + (n − 1) × (− 2) = 5
in formula Sn = [2a + (n – 1)d]
2 ⇒ 23 − 2n + 2 = 5
So 41n – n2 = 400 ⇒ − 2n = − 20
⇒ n = 16, 25
⇒ n = 10.
... n = 25 not possible
because if n = 25 then the number of logs in Hence, number of rows is 10.
top row 7. In the series (– 5) + (– 8) + (–11) + .....+ (– 230),
= –4 a = – 5, d = – 8 – (– 5) = – 3.
... n = 16 and a16 = 5. Let the number of terms be n, then
– 230 = – 5 + (n – 1) (– 3)
WORKSHEET – 33 [∵ an = a + (n – 1)d]
1. (A) a = 10, d = 7 – 10 = – 3 – 225
⇒ n – 1= ⇒ n = 76
a30 = a + 29d = 10 + 29(–3) = – 77. –3
Now, sum of first n terms is given by
2. (C) x + 10 – 2x = 3x + 2 – (x + 10)
n
⇒ x – 2x – 3x + x = 2 – 10 – 10 Sn = (a + an)
2
18
⇒ x= = 6. 76
3 = (–5 – 230) (∵ an = – 230)
2
3. (C) an = 2n + 1 ∴ a1 = 2 × 1 + 1 = 3 = – 38 × 235 = – 8930.
n n
Now, Sn = (a1 + an) = (3 + 2n +1) 8. n2
2 2
= n(n + 2) Hint: S7 = 49
S17 = 289
4. No.
Find a and d and then
Let an = 68 n
⇒ a + (n − 1) × d = 68 find Sn = [2a + (n – 1)d].
2

A R I T H M E T I C P R O G R E S S I O N S 171
9. To pick up the first potato, distance run 5. an = Sn − Sn − 1
= 2(5) m  3 ( n − 1)2 13 
3n2 13
To pick up the second potato, distance run =
2
+
2
n−
2
+
2
( n − 1)
= 2(5 + 3) = 2 (8) m  
To pick up the third potato, distance run 3n2 13 3 13 13 
= 2(8 + 3) = 2 (11) m =
2
+
2 2
(
n −  n2 + 1 − 2n +
2
n−
2
)

.........................................................................
3n2 13 3 3 13 13
......................................................................... = + n − n2 − + 3n − n+ .
2 2 2 2 2 2
∴ Sequence of the distance run is:
= 3n + 5
2 (5), 2 (8), 2 (11), ............, till 10 terms. ∴ a25 = 3 × 25 + 5 = 80.
∴ Total distance covered
6. Hint: Let a = first term
= 2 [5 + 8 + 11 + ........ + 10 terms] and d = common difference
 10  1 1
= 2
 2
{
2 ( 5 ) + (10 − 1) ( 3 ) 

} am =
n
⇒ a + (m – 1)d =
n
= 2  5 ( 37 ) 1 1
an = ⇒ a + (n – 1)d =
m m
= 10 × 37
1 1
= 370 m. Solving d= and a =
mn mn
WORKSHEET – 34 mn
∴ Smn =
2
{2a + (mn − 1) d}
1. (A) 2 × (6x – 2) = 8x + 4 + 2x + 7
1
15 = (mn + 1) .
⇒ 12x – 4 = 10x + 11 ⇒ x = . 2
2
7. Let a3 = x; a7 = y
2. (B) Sn = 1 + 3 + 5 + ......... (n terms) ∴x + y= 6
∴ a = 1, d = 2 xy = 8
n ⇒ x2 – 6x + 8 = 0
∴ Sn = [2a + (n – 1)d]
2 ⇒ (x – 4) (x – 2) = 0
n
⇒ Sn = [2 + (n – 1) × 2] = n2. ⇒ x = 4 or x = 2
2
⇒ x = 4, y = 2; x = 2, y = 4
3. (D) an = Sn − Sn − 1
⇒ a3 = 4 and a7 = 2
= n2 + 2n − (n − 1)2 − 2(n − 1)
1
= n2 + 2n − n2 − 1 + 2n − 2n + 2 ⇒ a = 5; d = –  ⇒ S16 = 20
2
= 2n + 1.
and a3 = 2 and a7 = 4
4. Let the common difference and the first term
be d and a respectively. 1
⇒ a = 1; d =  ⇒ S16 = 76.
Now, a18 – a14 = 32 2
⇒ a + 17d – a –13d = 32 8. (i) 12 (ii) 62
⇒ 4d = 32 Hint: (i) Sn = 636.
⇒ d = 8. (ii) Let an = a51 + 132.

172 M A T H E M A T I C S – X
9. l1 = Length of 1st semicircle = πr ⇒ 83 – 3n < 0
= π (0.5) ⇒ 3n > 83
l2 = Length of 2nd semicircle = π (1) 2
⇒ n > 27
l3 = Length of 3rd semicircle = π (1.5) 3
l4 = Length of 4th semicircle = π (2) ⇒ n = 28.
.............................................................. Sm m2 .
6. Hint: Use = 2
.............................................................. Sn n
∴Sequence is: 7. 6 or 12
π (0.5), π (1), π (1.5), π (2), ......... till 13th term Hint: Let Sn = 72
We will find n
⇒ [2a + (n – 1)d] = 72.
S13 = π [0.5 + 1 + 1.5 + 2 +....... + 13th term] 2
8. n = 9; angle = 32°.
= π  13 2 (0.5 ) + (13 − 1) × (0.5 ) 
{ } Hint: Sum of all angles = 360°.
 2 
9. Volume of concrete required to build the
13 13 22
= (7 π ) = × × 7 1 1
2 2 7 1st step = × × 50 m 3
4 2
= 143 cm.
Volume of concrete required for

WORKSHEET – 35 2 1
2nd step = ×   × 50 m3
4  2
1. (A)
Volume of concrete required for
Hint: a + 6d = 34
and a + 12d = 64 3 1
3rd step = ×   × 50 m 3
⇒ a = 4, d = 5. 4 2
..............................................................
2. (C)
..............................................................
Hint: d = 8 − 2 = 2 2− 2 = 2. Volume of concrete required for 15th step.
a =2 15 1
n = × × 50 m 3
Use Sn = [2a + (n – 1)d]. 4 2
2 ∴ Total volume of concrete required:
3. (D) ∵ 21 is an odd number ∴ a21 = 1
50
∴ 40 is an even number ∴ a40 = –1 S15 = [ 1 + 2 + 3 + ............ + 15]
8
4. Let nth term be 181
25 15  n 
a = 5, d = 8, an = 181 = × × (1 + 15 )  ∵ Sn = 2 {a + l}
Now, 5 + (n – 1) × 8 = 181 4 2

176 25 × 15 × 16
⇒ n – 1= = = 750 m3.
8 8
⇒ n = 23.
5. 28th term WORKSHEET – 36
Hint: Let an < 0 1. (A)
83 3n n
∴ – <0 Hint: Use: Sn = [2a + (n – 1)d].
4 4 2

A R I T H M E T I C P R O G R E S S I O N S 173
2. (B) an + b n a+b
Hint: Use: an = a + (n – 1)d. 9. We have =
a n −1 + b n −1 2
3. (B) ⇒ 2an + 2bn = an + abn–1 + ban–1 + bn
Hint: Common difference = – 2 – 1 or – 5 + 2,
⇒ an + bn – abn–1 – ban–1 = 0
i.e., – 3.
⇒ a (a – b) – bn–1 (a – b) = 0
n–1
4. 10n – 2
⇒ (a – b) (an–1 – bn–1) = 0
Hint: an = Sn– Sn – 1
n−1
a
= 5n2 + 3n – 5 (n – 1)2 – 3 (n – 1) ⇒ a = b or   =1
= 5n2 + 3n – 5 (n2 + 1 – 2n) – 3n + 3 b
= 5n2 + 3n – 5n2 – 5 + 10n – 3n + 3 n−1
Taking  a  =1
an = 10n – 2. b
5. a= 2 n−1 0
a =  a
⇒    
1 b
S5 = (S10 – S5 ) b
4 ⇒ n–1= 0
5 1 5 ... n= 1.

2
{4 + 4d} = 4  5 × ( 4 + 9d ) − 2 ( 4 + 4d ) OR
 
an = x
5 5
⇒ × 4 {1 + d} =  4 + 9d − 2 − 2d ⇒ a + (n − 1) × d = x
2 4
⇒ 1 + (n − 1) × 3 = x
⇒ 8 + 8d = 4 + 9d − 2 − 2d
⇒ 3n − 2 = x ...(i)
⇒ d = −6
n
∴ S30 = 15 {4 + 29 (− 6)} ∴ S=
2
(a + l)
= 15 × (− 170) = − 2550. n
590 = (1 + x )
n 2
6. Hint: S1 =
2
{
2a + ( n − 1) d } 1180 = n (1 + 3n − 2)
2n 1180 = 3n2 − n
S2 =
2
{
2 a + ( 2 n − 1) d } ⇒ 3n2 − n − 1180 = 0
3n ⇒ 3n2 − 60n + 59n − 1180 = 0
S3 =
2
{
2 a + ( 3 n − 1) d } ⇒ 3n(n − 20) + 59 (n − 20) = 0
Calculate 3(S2 – S1). − 59
⇒ n= (Reject) or n = 20
7. 900 3
Hint: S24 = 12(a1 + a24) ∴ From (i) ⇒ x = 3 × 20 − 2
Also note: x = 60 − 2 ⇒ x = 58.
a5 + a20 = a1 + a24
WORKSHEET – 37
a10 + a15 = a1 + a24
Hence given relation gives: 1. (C) Let x = nth term
3 (a1 + a24) = 225 ∴ x = 2 + (n – 1) 3 ⇒ x = 3n – 1
a1 + a24 = 75 n n
∴ Sn = {2 + x} ⇒ 155 = {2 + 3n − 1}
∴ S24 = 900. 2 2
2
⇒ 310 = 3n + n 2
⇒ 3n + n – 310 = 0
8. 7, 8, 9
Hint: Let the three numbers be: ⇒ n(3n+31) – 10 (3n + 31) = 0 ⇒ n = 10
a − d, a, a + d. ... x = 29.

174 M A T H E M A T I C S – X
2. (A) 7. Hint: Let the first term and the common
Hint: Use an = a + (n − 1) . d. difference be a and d respectively.
a9 = 0 ⇒ a + 8d = 0 ⇒ a = – 8d
3. 19668
a29 = a + 28d = – 8d + 28d = 20d
Hint: The sequence is:
a19 = a + 18d = – 8d + 18d = 10d.
103, 119, 135, ......., 791.
4. n = 6 8. 2, 6, 10, 14.
Hint: Let a = 3, b = 17 Hint: Let the four parts be:
∴ a, x1, x2, ..........., xn, b are in A.P. a − 3d, a − d, a + d, a + 3d.

b−a OR
14
∴ d= = The sequence is: 150, 146, 142, ..........
n+1 n+1
∴ Total number of workers who worked
14 3n + 17
∴ x1 = a + d = 3 + = ...(i) all the n days
n+1 n+1
= 150 + 146 + ....... + n terms.
14 n 17 n + 3 ∴ = n(152 − 2n)
xn = a + nd = 3 + = ...(ii)
n+1 n+1 Now had the workers not dropped then
x1 1 the work would have finished in (n − 8)
= ⇒ 3x1 = xn
xn 3 days with 150 workers working on each
day.
∴ Using (i) and (ii), we get
∴ Total number of workers who would
n = 6.
have worked all the n days is 150(n − 8).
n
5. Hint: S1 = (n + 1); a = 1, d = 1 ∴ n(152 − 2n) = 150(n − 8)
2
⇒ n2 − n – 600 = 0
S2 = n2 ; a = 1, d = 2
⇒ (n − 25) (n + 24) = 0
n
S3 =(3n – 1); a = 1, d = 3 ⇒ n = 25 or n = – 24 (Reject)
2
n n ∴ n = 25.
∴S1 + S3 = ( n + 1) + ( 3n − 1)
2 2
ASSESSMENT SHEET – 3
S1 + S3 = 2n2
∴S1 + S3 = 2S2. 1. (B) an = a + (n – 1)d
6. Let a1 = a and common difference = d. ⇒ 210 = 21 + (n – 1) × 21
Now, a1 + a7 + a10 + a21 + a24 + a30 = 540 210 – 21
⇒ n – 1=
a + a + 6d + a + 9d + a + 20d + a + 23d + a + 29d 21
= 540 ⇒ n = 10.
⇒ 6a + 87d = 540 2. (A) a18 – a14 = 32 ⇒ a + 17d – (a + 13d) = 32
⇒ 2a + 29d = 180 ...(i)
32
Further, the required sum ⇒ d= ⇒ d = 8.
4
30
S30 = (a + a30) = 15(a + a + 29d) 3. Sn = 2n2 + 5n
2 1
= 15(2a + 29d) ⇒ Sn– 1 = 2(n – 1)2 + 5(n – 1)
= 15 × 180 [Using equation (i)] = 2(n2 + 1 – 2n) + 5n – 5
= 2700. = 2n2 + n – 3

A R I T H M E T I C P R O G R E S S I O N S 175
nth term = Sn – Sn – 1 3x – 2y x – y
Common difference = d = –
= 2n2 + 5n – (2n2 + n – 3) x+y x+y
= 4n + 3.
2x – y
4. False, because nth term of an A.P. is always =
x+y
a linear polynomial in n.
Now, sum of n terms
4
5. First term of the A.P. = a = – n
3 = [2a + (n – 1)d]
2
Common difference of the A.P. = d
4 1 n  x–y 2x – y 
= – 1 –  –  = = 2× + (n – 1)
 3 3 2  x + y x+y 

Let the A.P. consists n terms.


1 13 n  2 x – 2 y + 2 nx – ny – 2 x + y 
∴ nth term = 4 = ...(i) =  
3 3 2 x+y 
But nth term is given by
an = a + (n – 1)d n  2nx – ny – y 
=  
2 x+y 
4 1
= – + (n – 1) n
3 3 = {n(2x – y) – y}.
n 5 2 (x + y )
= – ...(ii)
3 3 7. Let the first term and the common difference
From equations (i) and (ii), we get of the given A.P. be a and d respectively.
n 5 13 According to the given condition,
– = ⇒ n = 18
3 3 3 a11 2 a + 10d 2
= ⇒ =
Since n = 18 is even number so, the middle a18 3 a + 17 d 3
18
th
 18 
th [Using an = a + (n – 1)d]
most terms will be   and  + 1 
 2   2  ⇒ 3a + 30d = 2a + 34d
th th
terms, i.e., 9 and 10 terms. ⇒ a = 4d
4 1 a5 a + 4d
Now, a9 = – + (9 – 1) × Now, a =
3 3 21 a + 20d
4 8 4 4d + 4d
=– + = = (Substituting a = 4d)
3 3 3 4d + 20 d
4 1
And a10 = – + (10 – 1) 8d 1
3 3 = =
24d 3
4 9 5
=– + = i.e., a5 : a21 = 1 : 3.
3 3 3
Therefore, the required sum 5
S5  2 a + 4 d 
4 5 9 Now, = 2
= a9 + a10 = + = S21 21
3 3 3  2 a + 20 d 
2 
= 3.
 n 
 Using S n = 2 {2 a + (n – 1) d}
x–y
6. First term = a =
x+y

176 M A T H E M A T I C S – X
5 (8 d + 4 d ) 3 ab
= ⇒ Sn = .
21 (8 d + 20 d ) 2 (b – a )
[Substituting a = 4d] 4. Yes; the first term is ` 2000 of an A.P. and
60 d 5 the common difference is ` 200 which is
= = interest per year.
588 d 49
i.e., S5 : S21 = 5 : 49. 5. The integers between 100 and 200, which
are divisible by 9 are:
8. Distances covered by a girl during 1 st
108, 117, 126,........, 198.
minute, 2 nd minute, 3 rd minute,..... are
respectively 20 m, 18 m, 16 m,.......which Which is an A.P. with first term(a) = 108
form an A.P. with first term (a) = 20 m and and common difference (d) = 9.
common difference (d) = – 2 m. Let this A.P. has n terms nth term is given by
(i) Distance covered in 10th minute an = a + (n – 1)d ⇒ 198 = 108 + (n – 1) × 9
= 10th term of the A.P.
198 – 108 90
= a + (10 – 1)d = 20 + 9 × (– 2) = 20 – 18 ⇒n – 1= ⇒n=1+ ⇒ n = 11
9 9
= 2 m.
Now,
(ii) Distance covered in 10 minutes n
= sum of first 10 terms Sn = (a + l), l being last term
2
10
= [2a + (10 – 1)d] = 5[2 × 20 + 9 (– 2)] 11 11
2 S11 = (108 + 198) = × 306
2 2
= 5 × 22 = 110 m.
= 1683.
ASSESSMENT SHEET – 4 Required sum
= 101 + 102 +....+ 199 – S11
1. (D) 2 + 4 + 6 +....... n terms
99
= k(1 + 3 + 5 +..... n terms) = (101 + 199) – 1683 = 14850 – 1683
2
⇒ 2(1 + 2 + 3 +........ n terms)
= k(1 + 3 + 5 + ...... n terms) = 13167.

n(n + 1) n 6. We need to prove


⇒ 2 = k × × {2 + (n – 1) × 2} (x + 2y – z) (2y + z – x) (z + x – y) = 4xyz
2 2
...(i)
⇒ n2 + n = kn2
If x, y, z are in A.P., then
n +1
⇒ k= . z+ x
n y – x= z – y ⇒ y = ...(ii)
2. (D) a + 8d = 449 and a + 448d = 9 2
⇒ a= 457; d = – 1 Let us take LHS of (i)
Further, 0 = 457 + (n – 1) (– 1) (x + 2y – z) (2y + z – x) (z + x – y)
⇒ n= 458. = (x + z + x – z) (z + x + z – x)

3. Let the A.P. has n terms. So, an = 2a.  z+ x 


2a = a + (n – 1) × (b – a) z+x – 2  [Using (ii)]
 
⇒ 2a = a + n(b – a) – b + a
b z+ x
⇒ n = = 2x × 2z × = 2xz(z + x)
b– a 2
= 2xz × 2y [Using (ii)]
b
Sn = (a + 2a) = 4xyz = RHS. Hence proved.
2 (b – a )

A R I T H M E T I C P R O G R E S S I O N S 177
7. Let first term = a, So, a = 457 and d = – 1
Common difference = d Now, an = 300 ⇒ 457 + (n – 1) (–1) = 300
n ⇒ (n – 1) = 157 ⇒ n = 158.
Sum of first n terms = [2a + (n – 1)d]
2
5. True, the reason is:
Sum of first 20 terms = 400
d = 14 – 8 = 6, a = 8
20
⇒ [2a + (20 – 1)d] = 400 a53 = a + 52d = 8 + 52 × 6 = 320
2
2a + 19d = 40 ...(i) a41 = a + 40d = 8 + 40 × 6 = 248
Sum of first 40 terms = 1600 Now, a53 – a41 = 72.
40 6. The first two digit number divisible by 4 is
[2a + (40 – 1)d] = 1600
2 12 and the others are: 16, 20, 24,............., 96.
2a + 39d = 80 ...(ii) Now, we have to find the sum of the
Subtracting equation (i) from equation (ii), following series:
we get 12 + 16 + 20 +.............+ 96
20d = 40 ⇒ d = 2
Here, a = 12, d = 16 – 12 = 20 – 16 = 4
Substituting d = 2 in equation (i), we get
Let the number of terms in the series be n.
2a + 38 = 40 ⇒ a = 1
Now, nth term is given by
10 a + (n – 1)d = 96
sum of first 10 terms = [2a + (10 – 1)d] ⇒ 12 + (n – 1) × 4 = 96
2
= 5(2 + 9 × 2) ⇒ (n – 1) = 21
= 100. ⇒ n = 22

8. See Worksheet – 37, Sol. 8 (OR part). 22


Now, S22 = (12 + 96)
2
CHAPTER TEST  n 
∵ S n = 2 ( a + l) 
n n
1. (C) Sn = (a + an) ⇒ 399 = (1 + 20)
2 2 = 11 × 108 = 1188.
⇒ 21n = 2 × 399 ⇒ n = 38. 7. Let the profit to be ceased at nth day.
2. (B) a + d = 13 and a + 4d = 25 Sale on first day = ` 8100
⇒ a = 9, d = 4 Sale on second day = ` (8100 – 150)
Now, a7 = a + 6d = 9 + 24 = 33. = ` 7950
So, the sale (in `) will be day by day as
3p –1 follows:
3. (A) ∵ ap =
6 8100, 7950, 7800,..........n terms
3n – 1 1 Here, a = 8100, d = –150
∴ an = and a1 =
6 3 The profit will be ceased when it is equal
to or less than ` 1500.
n  1 3n – 1  n
Now, Sn = +  = 12 (3n + 1). Therefore, 8100 + (n – 1) × (– 150) ≤ 1500
2  3 6 
[∵ an = a + (n – 1)d]
4. Let the nth term be required ⇒ 8100 – 150n + 150 ≤ 1500
a9 = 449 ⇒ a + 8d = 449 ⇒ 150n ≥ 6750 ⇒ n ≥ 45
a449 = 9 ⇒ a + 448d = 9 Hence, the profit to be ceased at 45th day.

178 M A T H E M A T I C S – X
8. If x, y and z are in A.P., then Let the length of each rung decreases by
x cm from bottom to top.
x+z
y = ...(i) So, lengths (in cm) of all rungs from
2
bottom to top are respectively
Now, in the given equation, 45, 45 – x, 45 – 2x, .......25.
LHS = (x + 2y – z) (2y + z – x) (z + x – y) This is an A.P. of 11 terms
= (x + x + z – z) (x + z + z – x) (2y – y) ∴ 45 + (11 – 1) × (– x) = 25
[Using equation (i)] [Using a + (n – 1) d = an]
= 2x × 2z × y ⇒ – 10x = – 20 ⇒ x = 2 cm
= 4xyz So, the A.P. now becomes
= RHS. Hence proved. 45, 43, 41,........., 25
 2.5 m  Now, required length of wood
9. Total number of rungs =   +1 = Sum of this A.P.
 25 cm 
11 11
250 = (45 + 25) = × 70
= + 1 = 11 2 2
25
= 385 cm
Length of the largest rung = 45 cm
Hence, the required length of the wood is
Length of the shortest rung = 25 cm 3.85 m.
❑❑

A R I T H M E T I C P R O G R E S S I O N S 179
Chapter

3 CIRCLES

WORKSHEET – 41 6. See solved example 4.


7. AB = 13 cm, AC = 15 cm
1. (B) Join OX.
Hint:
In ∆XOY,
Use
∠OXY= 90°,
ar(∆OBC + ∆OAC + ∆OAB)
∴ XY = OY 2 – OX 2 = ar(∆ABC).
= 132 – 52 = 12 cm. 8. Hint: Join AB
2. (D) OT2 + TQ2 = OQ2 Let OA = r ⇒ OP = 2r
⇒ OT2 + (24)2 = (25)2 In ∆OAP,
⇒ OT2 = 625 – 576 = 49 OA r 1
sin θ = = =
⇒ OT = 7 cm. OP 2 r 2

3. (B) As AB || PR θ = 30°
∴ ∠APB = 2 × 30° = 60°
⇒ ∠BQR = ∠ABQ = 70°
In ∆ABP, AP = BP
(Alternate interiar angles)
∴ ∠A = ∠B
But, ∠A + ∠B = 180° – ∠APB = 120°
⇒ ∠A = ∠B = 60°
∴ ∆APB is an equilateral triangle.

WORKSHEET – 42
Also ∠ABQ = ∠BAQ = 70°
{∵ ∆AMQ ≅ ∆BMQ} 1. (B) As ∠OQP = 90º
∴ In ∆AQB, using Angle sum property ∴ x = 90º – 30º = 60º.
∠AQB = 180° – 70° – 70° = 40°.
4. False,
Perimeter of ∆ABC = AB + BC + AC 2. (Α) ∠QOR = 180º – 30º = 150º
= AB + BQ + CQ + CR + AR
= AB + BP + CQ + CQ + AP
= AB + (BP + AP) + 2CQ
= 2(AB + CQ)
= 2(8) = 16 cm.
5. LHS 1
∠PRQ = × 150º = 75º.
= AB + CD 2
= (AP + PB) + (CR + RD) 3. (C) BC = BP + PC
= AS + BQ + CQ + DS = BR + CQ
= (AS + DS) + (BQ + CQ) = 3 + [AC – AQ]
= AD + BC = 3 + [11 – 4]
= RHS. = 10 cm.

180 M A T H E M A T I C S – X
4. True. 8. Let the given parallelogram
Let M be the point of be ABCD whose sides
touches a circles at P, Q, R
contact and O be the
centre of the circle. and S as shown in the
adjoining figure.
∠ABM = ∠ACM Since, length of two
(∵ AB = AC)
tangents drawn from an
1 1 external point to a circle are equal.
∠ABM = ∠ACM
2 2 ∴ AP = AS ...(i)
∠OBM = ∠OCM ... (i) Similarly, we have
∠BMO = ∠CMO ... (ii) (Each 90°) PB = BQ ...(ii)
OM = OM ... (iii) (Common) DR = SD ...(iii)
Using equations (i), (ii) and (iii) in ∆BMO RC = QC ...(iv)
and ∆CMO, we have Adding these four equations, we have
AP + PB + DR + RC = AS + BQ + SD + QC
∆BMO ≅ ∆CMO (AAS corollory)
⇒ (AP + PB) + (DR + RC)
∴ BM = CM (CPCT)
= (AS + SD) + (BQ+ QC)
⇒ BC is bisected at the point of contact.
⇒ AB + DC = AD + BC
5. As AD + BC = AB + CD ∵ AB = DC and AD = BC
⇒ AD + 7 = 6 + 4 (ABCD is a parallelogram)
⇒ AD = 3 cm. ∴ AB = BC
Thus, AB = BC = CD = DA
Hence, ABCD is a rhombus.

6. 11 cm 9. Let line l be the


Hint: OQBP is a square tangent at a
∴ OQ = BP = 11 cm. point P to the
circle with centre
O. Let us take
any point Q on
7. Let the tangents be PQ and the tangent l as shown in the figure.
PR corresponding to the Join OQ to meet the circle at M.
chord QR of the circle with We know that if a point is met with the
centre O.
different points of a line, then the shortest
Join OQ, OR and OP.
line segment is the perpendicular on that
In ∆PQO and ∆PRO,
line. Consider the adjoining figure:
∠PQO = ∠PRO = 90°
(Angles formed between O M = OP (Radii of same circle)
tangent and corresponding OQ = OM + MQ
radius) ⇒ OQ = OP + MQ
PO = PO (Common) ⇒ OQ > OP
QO = RO (Radii of same circle)
i.e., OP < OQ
Therefore, we arrive at
Clearly, OP is the shorter than OQ. Similarly,
∆PQO ≅ ∆PRO (RHS axiom of congruence)
we can prove that OP is the shortest all OV,
So, PQ = PR (∴ CPCT)
V being a variable point on the line other
Thus, ∆PQR is an isosceles triangle.
than P. Therefore, OP is the perpendicular to
∴ ∠PQR = ∠PRQ. Hence proved.
line l.
C I R C L E S 181
Hence, tangent l ⊥ radius OP. ⇒ ∠PAC = 2∠CAO
2nd Part: Join OY
∠OYX = 90º
and ∠OAY = b + a = ∠OYA
[∵ OA = OY = radius]
⇒ b + a = 90º – a
⇒ b + 2a = 90º.

WORKSHEET – 43 Similarly,
1. (B) ∠Q = ∠R = 90° ∠CBQ = 2∠CBO
In quadrilateral PQOR, As ∠PAC + ∠CBQ = 180°
1 1 1
⇒ ∠PAC + ∠CBQ = × 180°
2 2 2
⇒ ∠CAO + ∠CBO = 90°
... ∠AOB = 90°.

∠P = 360° – (90° + 130° + 90°) = 50° 8. 9 cm


Hint: ∠AMP = 90°
2. (C)
Hint: AC = 10 cm
∴ ar(∆ABC)
= ar(∆AOB) + ar(∆BOC)
+ ar(∆AOC)
1
⇒ 24 = × (8 × r + 6 × r + 10 × r)
2 AM 1 AM
⇒ 48 = r × 24 ⇒ r = 2 cm. cos 60° = ⇒ =
AP 2 9
3. ∠PTQ + ∠POQ = 180º
9
⇒ ∠PTQ = 180º – 110º ⇒ AM = ∴ AB = 2AM ⇒ AB = 9 cm.
2
= 70º.
4. CP = CQ = 11 cm WORKSHEET – 44
BQ = CQ – CB
1. (A)
= 11 – 7 = 4 cm
See Worksheet – 41, Sol. 2.
∴ BR = QB = 4 cm.
2. (C) ∠PTQ + ∠POQ = 180º
5. See Worksheet – 42, Sol. 8.
6. Hint: XP = XQ
⇒ XA + AP = XB + BQ
⇒ XA + AR = XB + BR
 ∵ AP = AR

 and BQ = BR . ⇒ ∠PTQ = 180º – 115º = 65º.
7. Hint: 3. False.
As PA = AC ∵ ∠OQL = 90°
... ∆PAO ≅ ∆CAO (SSS)
∴ ∠OQS = 90° – ∠SQL = 40°
⇒ ∠PAO = ∠CAO (CPCT)

182 M A T H E M A T I C S – X
Similarly, So, by AAA criterion of similarity, we have
∠ORS = 90° – ∠SRM = 30° ∆TPM ~ ∆POM
TP PM TP 5
⇒ = ⇒ =
PO OM 7 2 6
35 6 35 6
⇒ TP = × = cm.
2 6 6 12
6. See Worksheet – 41, Sol. 5.
7. Let the given two
tangents be PA and PB
∵ In ∆SOQ, ∠OSQ = ∠OQS = 40° to the circle with
And in ∆SOR, ∠OSR = ∠ORS = 30° centre O.
∴ ∠QSR = OSQ + ∠OSR We need to prove
= 40° + 30° = 70°. ∠APB + ∠AOB = 180°.
4. Perimeter of ∆ABC We know that the angle formed by a
= AB + BC + AC tangent to the circle and the radius passing
= (AQ – BQ) + BC + (AR – CR) through the point of contact is 90°.
= AQ + AR + BC – (BP + PC) ∴ ∠PAO = ∠PBO = 90°
= 2AQ [∵ AQ = AR] Applying angle sum property in the
1 quadrilateral AOBP, we get
∴ AQ = AR = (Perimeter of ∆ABC).
2 ∠PAO + ∠AOB + ∠PBO + ∠APB = 360°
5. Let OT intersects PQ at M. ⇒ 90° + ∠AOB + 90° + ∠APB = 360°
⇒ ∠AOB + ∠APB = 180°.
Hence proved.
8. We have given
l || m to a circle. DE
is intercept made
by tangent at C,
between l and m.
We have to prove ∠DEF = 90°
OM = 2
7 –5 2 Construction: Join A to F, F to B and F to C.
Proof: In a triangles ADF and DFC, we have
= 24 = 2 6 cm ... (i)
DA = DC
Let ∠PT M = θ ... (ii) (Tangents drawn from an external point
Then ∠T PM = 90° – θ ... (iii) are equal in length)
And ∠O P M = θ ... (iv) DF = DF (Common)
(∵ OP ⊥ PT) AF = CF (Radii of the same circle)
And ∠MOP = 90° – θ ... (v) ∴ ∆ADF ≅ ∆CDF (SSS)
In ∆TPM and ∆POM, ⇒ ∠ADF = ∠CDF (CPCT)
⇒ ∠ADC = 2∠CDF ... (i)
∠TP M = ∠POM = 90° – θ
Similarly, ∠CEB = 2∠CEF ... (ii)
[From (iii) and (v)] Now, ∠ADC + ∠CEB = 180°
∠PTO = ∠MPO = θ Sum of the interior angles on the same side
[From (ii) and (iv)] of transversal is 180°.
∠TMP = ∠PMO (Each 90°) ⇒ 2∠CDF + 2∠CEF = 180°

C I R C L E S 183
⇒ ∠CDF + ∠CEF = 90° ... (iii) PQ = PB (Each radius)
In ∆DEF, So, by SAS creterion of similarity, we have
∠DEF + ∠EDF + DFE = 180° ∆RPQ ~ ∆RPB
⇒ 90° + ∠DFE = 180° [From (iii)] ∴ ∠RBP = ∠RQP
⇒ ∠DFE = 90°. But RQ ⊥ PQ,
Hence proved. ∴ ∠RQP = 90°
∴ ∠RBP = 90°
WORKSHEET – 45 ⇒ BR is tangent at B. Hence proved.

1. (D) Let radius = r 5. Join OR and OS.


∴ PO = PB = BQ = r
AB = 17 2 – 152 = 8 cm

Let AP = x ∴ AS = x
In quadrilateral OSDR,
∠O + ∠S + ∠D + ∠R = 360°
Now, ⇒ ∠O + 90° + 90° + 90° = 360°
AC = AR + RC (∵ OS ⊥ AD and OR ⊥ CD)
⇒ 17 = 8 – r + 15 – r ⇒ ∠O = 90°
⇒ 2r = 23 – 17 = 6 ⇒ OSDR is a square.
⇒ r = 3 cm. ⇒ DR = DS = r
Now, ABCD is a subscribed quadrilateral
2. (B) AB = OB 2 – OA 2 ∴ AB + CD = BC + DA
= 169 – 25 ⇒ x + 27 + 25 = 38 + r + x
⇒ r = 14 cm.
= 144 = 12 cm.
6. See Worksheet – 44, Sol. 4.
3. True, because in right angled isosceles
triangle AOB, OR
We know that two
OP = a2 + a2 = a 2 tangents drawn from
an external point to a
4. In ∆APQ,
circle are equal in
∠PAQ = ∠AQP = θ (say)
length.
(∵ AP = PQ = radius) ∴ PA = PB
∠RPB = ∠QAP = θ ⇒ ∠PBA = ∠PAB = θ (say) ...(i)
(Corresponding angles) ⇒ ∠APB = 180° – 2θ ...(ii)
∠RPQ = ∠AQP = θ (Using ∆APB)
(Alternate angles)
Further, PA is tangent and AO is corres-
Now, in ∆RPQ and ∆RPB, ponding radius
RP = RP (Common) ⇒ PA ⊥ AO
∠RPQ = ∠RPB (Each θ) ⇒ ∠PAO = 90°

184 M A T H E M A T I C S – X
⇒ ∠OAB = 90° – ∠PAB 8. See solved example 4 for first part.
⇒ ∠OAB = 90° – θ ...(iii) Second part:
[Using equation (i)] x + y = 12 ...(i)
Dividing equation (ii) by equation (iii), we 10 – x = 8 – y
get ⇒ – x + y = – 2 ...(ii)
Solving (i) and (ii):
∠APB 180° – 2θ 2 ( 90° – θ ) 2y = 10
= = =2
∠OAB 90° – θ 90° – θ ⇒ y =5
∴ x =7
⇒ ∠APB = 2∠OAB. Hence proved.
AD = 7 cm, BE = 5 cm, CF = 3 cm.
7. Let the given quadrilate-
ral be ABCD subscribing WORKSHEET–46
a circle with centre O. Let
1. (A)
the sides AB, BC, CD and
As CD = DB = 4 cm
DA touch the circle at P,
AD = CD = 4 cm
Q, R and S respectively
∴ AB = 4+4
(see figure). = 8 cm.
Join OA, OB, OC, OD, OP, OQ, OR and OS.
We need to prove 2. (B) Hint: AB = 2AM
∠AOB + ∠COD = ∠BOC + ∠DOA = 180°.
Proof: In ∆AOP and ∆AOS,
OP = OS (Radii of same circle)
AP =AS (Tangents from external points)
AO = AO (Common) 3. Let CQ = x and
∴∆AOP ≅ ∆AOS(SSS axiom of congruence) BQ = y such that x + y = c ... (i)
∴∠1 = ∠8 ...(i) (CPCT) As CR and CQ are
Similarly, we can prove that tangents from the
∠2 = ∠3, ∠4 = ∠5 and ∠6 = ∠7 ... (ii) external point C,
As, ∠1, ∠2, ∠3, ∠4, ∠5, ∠6, ∠7 and ∠8 are ∴ CQ = CR
subtended at a point ⇒ x = a – r ... (ii)
∴ ∠1 + ∠2 + ∠3 + ∠4 + ∠5 + ∠6 + ∠7 + ∠8 Similarly,
= 360° y= b – r ... (iii)
⇒ ∠1 + ∠1 + ∠2 + ∠2 + ∠5 + ∠5 + ∠6 + ∠6 Add (i) and (ii).
= 360° x + y = a + b – 2r
Also, ∠8 + ∠8 + ∠3 + ∠3 + ∠4 + ∠4 + ∠7 + ∠7 ⇒ c = a + b – 2r [Using (i)]
= 360° a+b–c
⇒ r= .
[Using results from equations (i) and (ii)] 2
⇒ 2(∠1 + ∠2) + 2(∠5 + ∠6) = 360° 4. 16 cm
Also, 2(∠3 + ∠4) + 2(∠7 + ∠8) = 360° Given: AP = 5 cm
⇒ BP = 12 – 5 = 7 cm
⇒ 2∠AOB + 2∠COD = 360°
also AP = 5 cm = AQ
Also, 2∠BOC + 2∠DOA = 360° ∴ QC = 14 – 5 = 9 cm
⇒ ∠AOB + ∠COD = ∠BOC + ∠DOA ∴ BC = BR + RC
= 180° = BP + CQ
Hence proved. = 7 + 9 = 16 cm.

C I R C L E S 185
5. See Worksheet – 45, Sol. 7. ASSESSMENT SHEET– 5
6. See Worksheet – 43, Sol. 7.
1. (B) BC = 2AB
7. For proof of theorem see solved example 4.
2nd Part. = 2 169 – 25
See Worksheet – 41, Sol. 5. = 24 cm.
8. Join OB, OG, OA, OH and OC.
2. (C) If a circle touches the sides of a
Radius = OD = OG = OH = 4 cm
quadrilateral, then the angles subtended by
HC = DC = 6 cm each pair of opposite sides are supplementary.
BG = BD = 8 cm ∴ ∠AOB + ∠COD = 180°
Let AG = AH = x ⇒ 130° + ∠COD = 180°
ar(∆OBC) ⇒ ∠COD = 50°.
1 3. In right triangle AOP,
= × base × height
2 Perpendicular
1 tan (∠APO) =
= × 14 × 4 Base
2 AO
= 28 cm2 ⇒ tan 30° =
AP
1
ar(∆OAB) = × ( x + 8 ) × 4 1 5
2 ⇒ = ⇒ AP = 5 3
3 AP
= (2x + 16) cm2
∴ AP = BP = 5 3 cm.
1
ar(∆OBC) = × ( x + 6 ) × 4
2 4. True, because
= (2x + 12) cm2 ∠MAB = ∠ACB
∴ ar(∆ABC) = 28 + 2x + 16 + 2x + 12 and ∠NAC = ∠ABC,
But ∠ABC = ∠ACB
⇒ ar(∆ABC) = (4x + 56) cm2 ... (i)
∴ ∠MAB = ∠ABC
In ∆ABC,
and ∠NAC = ∠ACB
AB + BC + CA But these are alternate pair of interior angles.
s=
2 ∴ MN || BC.
x + 8 + 14 + x + 6 5. Note that tangents
= drawn from an
2
= x + 14 external point to a
circle are equal in
∴ ar(∆ABC) = s ( s – AB ) ( s – BC ) ( s – CA ) length.
Tangents are drawn from A, B and C, so we
⇒ ar(∆ABC) = ( x + 14) × 6 × x × 8 ... (ii) respectively get
Comparing equations (i) and (ii), we get AQ = AR ... (i)
BQ = BP ... (ii)
4x + 56 = ( x + 14 ) × 6 × x × 8 And CR = CP ... (iii)
⇒ 42 (x + 14)2 =
(x + 14) × 6 × x × 8 Taking equation (i), we
(On squaring both sides) AQ = AR = AC + CR = AC + CP
⇒ 16 (x + 14) (x + 14 – 3x) = 0 [From (iii)]
⇒ x = 7 as x ≠ –14 (∵ x > 0) = AC + BC – BP
So, AB = x + 8 = 7 + 8 = 15 cm = AC + BC – BQ [From (ii)]
and AC = x + 6 = 7 + 6 = 13 cm. = AC + BC – (AQ – AB)

186 M A T H E M A T I C S – X
= AC + BC + AB – AQ =
25 – 16 = 3 cm
⇒ 2 AQ = Perimeter of ∆ABC
(Pythagoras theorem)
1
⇒ AQ = (Perimeter of ∆ABC) Further in ∆ORP and ∆PRT,
2
Hence proved. ∠ORP = ∠PRT = 90°
(Proved above)
6. We are given two
∠OPR = ∠PTR
tangents AP and BP
corresponding to (∵ ∠OPR = 90° – ∠RPT = 90° – (90° – ∠PTR))
the chord AB of a Therefore,
circle with centre O. ∆ORP ∼ ∆PRT (AA criterion)
We need to prove OR OP
⇒ =
∠PAB = ∠PBA PR TP
Join AO, BO and PO. (Corresponding sides)
In ∆AOP and ∆BOP, 3 5
∠PAO = ∠PBO = 90° ⇒ =
4 TP
(Angle between tangent and corresponding
radius) 20
⇒ TP = cm.
OP = OP (Common) 3
AO = BO (Radii of same circle) 8. Join QO and TO
Therefore, to intersect PQ
∆AOP ≅ ∆BOP (RHS criterion) at R.
∴ PA = PB (CPCT) In ∆TOP and
Thus, ∆ PAB is an isosceles triangle. ∆TOQ,
⇒ ∠PAB = ∠PBA. Hence proved. TP = TQ
(Tangents from same external point)
7. Let O be the
centre of the PO = QO (Radii of same circle)
circle. Join OT to TO = TO (Common side)
meet PQ at R. So, ∆TOP ≅ ∆TOQ
Join OP and OQ. (SSS criterion of congruence)
In ∆PTR and ∆QTR, ∴ ∠POR = ∠QOR ... (i) (CPCT)
PT = QT Now, in ∆POR and ∆QOR,
(Tangents from an external point to the PO = QO (Radii of same circle)
same circle) ∠POR = ∠QOR [From (i)]
∠PTR = ∠QTR
OR = OR (Common side)
(Line segment joining the centre and the
point of intersection of the tangents bisects So, ∆POR ≅ ∆QOR
the angle between the tangents) (SAS criterion of congruence)
TR = TR (Common) ∴ ∠PRO = ∠QRO ... (ii)
Therefore, (Corresponding angles)
∆PTR ≅ ∆QTR (SAS criterion) ∠PRO + ∠QRO = 180º ... (iii)
⇒ PR = QR (CPCT) (Linear pair axiom)
Also, ∠ORP = ∠ORQ = 90° From (ii) and (iii),
(Angle between chord and the line segment ∠PRO = 90° ... (iv)
joining the centre and mid-point of the In ∆TPO,
chord) ∠TPO = 90° ... (v)
∴ ∠ORP = ∠PRT = 90°
(Angle between tangent and
In ∆OPR, OR = OP2 – PR 2 corresponding radius)

C I R C L E S 187
Further, in ∆PRO and ∆TPO, In quadrilateral OQCR,
∠PRO = ∠TPO = 90° ∠OQC = ∠ORC = 90°
(Angles between tangent and
[From (iv) and (v)]
corresponding radius)
∠POR = ∠TOP (Common angle) and ∠QCR = 90° (Given)
So, ∆PRO ~ ∆TPO ⇒ ∠QOR = 90°
(AA criterion of similarity) ⇒ OQCR is a square
∴ ∠OPR = ∠OTP ⇒ CQ = CR = r
1
⇒ BQ = a – r, AR = b – r,
⇒ ∠OPQ = ∠PTQ ⇒ AP = b – r, PB = a – r
2
But AB = c
(∴ ∠OTP = ∠OTQ) ∴ b – r + a – r= c
i.e., ∠PTQ = 2∠OPQ Hence proved. ⇒ 2r = a + b – c
a+b–c
ASSESSMENT SHEET – 6 ⇒ r= . Hence proved.
2
1. (D) Join OA 6. Draw a line QT
passing through Q
AT and perpendicular to
cos 30° =
OT QP to meet SR at T.
In ∆PQR,
3
⇒ AT = 6 × = 3 3 cm. PQ = PR
2
(Tangents from an external point)
2. (D) ∠BAT = ∠ACB = 55°. ∴ ∠PRQ = ∠PQR ... (i)
3. AD = DC = 4 (Angles opposite to equal sides)
⇒ AD = 4 cm ∠PQR + ∠PRQ + ∠QPR = 180° ... (ii)
(Angle sum property for a triangle)
From equations (i) and (ii),
∠PQR + ∠PQR + 30° = 180°
⇒ ∠PQR = ∠PRQ = 75° ... (iii)
Now, ∠TQR + ∠PQR = 90°
(Angle between tangent)
⇒ TQR = 15° ... (iv) [Using (iii)]
∴ SR || QP and QT ⊥ QP
Similarly CD = DB = 4 cm
∴ QT ⊥ SR
⇒ DB = 4 cm
⇒ ST = TR ... (v)
∴ AB = AD + DB = 4 + 4 = 8 cm. (∵ TQ passess through the centre of the
4. False, because the centres of the circles lie on circle)
the perpendicular of PQ, which passes In ∆STQ and RTQ,
through A. ST = TR [From (v)]
5. Let the sides AB, ∠STQ = ∠RTQ (∵ QT ⊥ SR)
BC and CA of the TQ = TQ (Common)
∆ABC touch the ∴ ∆STQ ≅ ∆RTQ (SAS criterion)
circle with centre O ⇒ ∠SQT = ∠RQT = 15° [Using (iv)]
at the point P, Q
∠SQT + ∠TQR = 15° + 15°
and R respectively.
⇒ ∠RQS = 30°.

188 M A T H E M A T I C S – X
OR BD = BE and CE = CF
∠APB = 120° Now, BD = AB – AD = 8 – x = BE
∴ ∠APO = ∠OPB = 60° and CE = BC – BE = 10 – (8 – x)
In right-angled ∆OAP = 2 + x = CF.
AP AF = AC – CF = 12 – (2 + x)
cos 60° = = 10 – x = AD
OP
1 But AD = x
AP
⇒ = ⇒ OP = 2AP ∴ 10 – x = x ⇒ x = 5 cm, i.e., AD = 5 cm
2 OP
BE = 8 – x = 8 – 5 = 3 cm
7. Let O be the centre and CF = 2 + x = 2 + 5 = 7 cm
of the circle with Thus, AD = 5 cm, BE = 3 cm and CF = 7 cm.
radius r.
Join SO and RO. CHAPTER TEST
In quadrilateral
1. (C) ∵ OA ⊥ AT
DROS,
∴ ∠OAT = 90°
∠SDR = 90° ... (i) (Given) In ∆OAT,
and ∠OSD = ∠ORD = 90° ... (ii) AT
(Angle between tangent and cos T =
OT
corresponding radius) AT
Therefore, ⇒ cos 30° =
4
∠SOR = 90° ... (iii)
⇒ Quadrilateral DROS is a rectangle ⇒ AT = 2 3 cm.
[From (i), (ii) and (iii)] 2. (A) ∠PAO = 90°
But SO = SR (= r) ∠OAB = ∠PAO – ∠PAB
∴ DROS is a square.
⇒ r = SO = DR = SD = OR ... (iv)
We know that tangents drawn from an 1
external point to a circle are equal. = ∠PAO – (∠PAB + ∠PBA)
2
∴ BQ = BP = 27 cm (∵ ∠PAB = ∠PBA)
(Tangents from point B)
1
QC = BC – BQ = 38 – 27 = 11 cm = ∠PAO – (180° – ∠APB) (ASP)
2
RC = QC = 11 cm
(Tangents from point C) 1
= 90° – (180° – 50°) = 25°.
DR = DC – RC = 25 – 11 = 14 cm 2
∴ r = 14 cm [From (iv)] 3. In right ∆POQ,
Hence, radius of the incircle is 14 cm.
PQ = 132 – 52 = 12 cm
8. Let AD = x. We
know that the PR = PQ = 12 cm
tangents drawn
from an external ar( PQOR)
point to a circle = ar(∆PQO) + ar(∆PRO)
are equal.
∴ AD = AF = x 1  1 
=  × PQ × QO  +  × PR × RO 
2  2 

C I R C L E S 189
7. We know that the
1  1 
=  × 12 × 5  +  × 12 × 5  tangents drawn from an
2  2 
external point to a circle
= 2 × 30 = 60 cm2 are equal in length.
4. True, as ∠BPA = 90°, (∵ AB is diameter) ∴ AQ = AR ... (i)
∠PAB = ∠OPA = 60° (∵ OP = OA) BQ = BP ... (ii)
and CP = CR ... (iii)
Now,
AQ = AB + BQ
= AB + BP [From (ii)]
= AB + (BC – PC)
= AB + BC – CR [From (iii)]
Also OP ⊥ PT. = AB + BC – (AR – AC)
∴ ∠APT = 30°
= AB + BC + CA – AR
and ∠PTA = 60° – 30° = 30°.
= AB + BC + CA – AQ [From (i)]
5. Let the given chord be AB ⇒ AQ + AQ = AB + BC + CA
and two tangents to the
1
circle with centre O be AP ⇒ AQ = (AB + BC + CA)
and BP. 2
We need to prove Hence proved.
∠PAB = ∠PBA 8. ∠BCA = 90° ... (i) (Angle in semicircle)
Join OA and OB. ∠PCA = ∠PCB + ∠BCA
Proof:
In ∆AOB, AO = BO
∴∠ABO = ∠BAO ... (i)
As, the tangent is perpendicular to the radius
passing through the point of contact,
∠PAO = ∠PBO = 90° ... (ii)
⇒ 110° = ∠PCB + 90°
Again, ∠PAO= ∠PBO [Using (ii)]
⇒ ∠PCB = 110° – 90° = 20° ...(ii)
⇒ ∠PAB + ∠BAO = ∠PBA + ∠ABO
⇒ ∠PAB + ∠ABO = ∠PBA + ∠ABO Since PC is a tangent and CB is a chord
[Using (i)] ∴ ∠BAC = ∠PCB = 20° ... (iii) [Using (ii)]
⇒ ∠PAB = ∠PBA. Now, applying angle sum property in ∆ABC,
we have
6. Hint: AP = AU,
∠CBA + ∠BCA + ∠CAB = 180°
BP = BQ,
⇒ ∠CBA + 90° + 20° = 180°
CR = CQ,
DR = DS, [Using (i) and (ii)]
ET = ES ⇒ ∠CBA = 180° – 90° – 20° = 70°.
FT = FU. ❑❑

190 M A T H E M A T I C S – X
Chapter

4 CONSTRUCTIONS

WORKSHEET – 50
1. (D) Since, the angle between two radii of a circle and the angle between corresponding two
tangents are supplementary.
∴ Required angle = 180° – 35° = 145°.
8
2. (B) Out of the numbers 8 and 5 in , the greater one is 8, so the required number of
5
points is 8.
3.

AC 3
AB = 8.5 cm and =
CB 7
AC = 2.55 cm, CB = 5.95 cm.
4. Steps of construction:
First, we draw a triangle ABC such that AB = 4 cm, AC = 5 cm and ∠BAC = 90°. Further, we
will draw a triangle A′BC′ similar to triangle ABC using the following steps.
Step I: Draw a ray BX making an acute angle with BC on the side opposite to vertex A.
Step II: Locate five points X1, X2, X3, X4 and X5 on BX so that BX1 = X1X2 = X2X3 = X3X4 = X4X5.
Step III: Join X3C and draw X5C′ || X3C to intersect BC extended at C′.
Step IV: Draw C′A′ || CA to intersect BA extended at A′.

C O N S T R U C T I O N S 191
A
90°

90°

m 5c
m
4c

B C C

X1

X2

X3

X4

X5 X

Then, ∆A′BC′ ~ ∆ABC.


5. Steps of construction:
Step I: First, draw a circle with radius as 5 cm and centre at O. Then take a point P so that
OP = 11 cm.
Step II: Bisect OP to find mid-point M of OP. Then take M as centre and MP = MO as radius,
draw a circle to intersect the previous circle at Q and R.
Step III: Join PQ and PR which are the required tangents.
After measuring PQ and PR, we find PQ = PR = 9.8 cm (approximately).
Justification:
Join OQ and OR.
In ∆OPQ, OP = 11 cm, OQ = 5 cm and PQ = 9.8 cm
∴ OP2 – OQ2 = 112 – 52 = (11 + 5)(11 – 5) = 96
And PQ2 = (9.8)2 = 96.04
Clearly, OP2 – OQ2 ≈ PQ2
⇒ OP2 = OQ2 + PQ2
Also, OP2 = OR2 + PR2
Therefore, ∆POQ and ∆POR are right triangles with ∠PQO = ∠PRO = 90°.

192 M A T H E M A T I C S – X
So, tangents are perpendicular to radii passing through their respective points of contact.
i.e., PQ ⊥ OQ and PR ⊥ OR.
6. ∠C = 180° – (∠B + ∠A) = 180° – 150° = 30°.
Steps of construction:
In order to construct a triangle similar to ∆ABC, follow the following steps:
Step I: First, construct a ∆ABC in which BC = 7 cm, ∠B = 45° and ∠C = 30°.
Step II: Make an acute angle CBX such that X is on the side opposite to vertex A.

C O N S T R U C T I O N S 193
Step III: Locate four points namely X1, X2, X3 and X4 on BX such that BX1 = X1 X2 = X2 X3 =
X3 X4.

Step IV: Join X3C and draw a line X4C′ || X3C to intersect BC produced at C′.
Step V: Draw a line C′A′ parallel to side CA of ∆ABC to intersect BA produced at A′. Then,
∆A′BC′ is the required triangle.

WORKSHEET – 51
1. (B) Since 4 + 7 = 11, therefore, B will be joined to A11.
2. (D) The required angle and the angle between the two tangents are supplementary.
∴ Required angle = 180° – 60° = 120°.
3. Here, 5 + 8 = 13
AB = 7.6 cm
AC : BC = 5 : 8
AC = 2.92 cm and BC = 4.68 cm

194 M A T H E M A T I C S – X
4. Steps of construction:
First, we draw ∆ABC with the given
measurements. Then we draw another
triangle A′BC′ similar to ∆ABC and of
3
scalar factor using the following steps:
4

Step I: Draw a ray BX such that ∠CBX is


an acute angle.
Step II: Mark X1, X2, X3, X4 on BX such
that BX1 = X1X2 = X2X3 = X3X4.
Step III: Join X4C and draw X3C′ || X4C.
Also, draw A′C′ || AC.
Thus, ∆A′BC′ ∼ ∆ABC.

5. Justification: In ∆PQR and ∆PQ′R′, ∠P = 45° is common and RQ || R′Q′.


∴ ∆PQR ~ ∆PQ′R′
We have draw PP1 = P1P2 = P2P3 = P3P4
P3 P4 1
∴ =
PP3 3

P3 P4 1
⇒ +1 = +1
PP3 3

P3 P4 + PP3 4
⇒ =
PP3 3

PP4 4
⇒ =
PP3 3
And P3Q || P4Q′

PQ ′ 4
∴ = .
PQ 3
Also, ∆PQR ~ ∆PQ′R′
PQ ′ PR ′ R ′ Q ′
Hence, = = .
PQ PR RQ

6. Steps of construction: In order to construct a pair of required tangents, follow the following
steps:
Step I: Draw a circle with radius OA = 3 cm and centre O.

C O N S T R U C T I O N S 195
Step II: Take any point P outside the circle
drawn in step I and join OP.
Step III: Obtain mid-point M of OP obtained
in step II and draw another circle with radius
OM = PM and centre M to intersect the
circle drawn in step I at A and B.
Step IV: Join PA and PB.
These PA and PB form the required pair of
tangents.

WORKSHEET – 52
1. (C) The next step should be the line parallel
to B5C should be passed through B4 as the
4
sides of required triangle are of the
5
corresponding sides of ∆ABC.
2. (C) Two distinct tangents to a circle can be
constructed from P only when P is situated
at a distance more than radius (here 2r)
from the centre.
3. False. In the ratio 3 + 2 : 3 – 2 , i.e.,
11 + 6 2 : 7, 11 + 6 2 is not a positive
integer, while 7 is.

4. Steps of construction: In order to construct a


∆ABC and its similar triangle with given
measurements, follow the following steps:
Step I : Draw a ∆ABC in
which BC = 7 cm, ∠B = 45° and
∠C = 180° – (45° + 105°) = 30°.
Step II: M a k e a n a c u t e
∠CBX such that X is on the
opposite side of the vertex
A and locate points B 1 , B 2 ,
B 3 and B 4 on BX such that
BB 1 = B 1 B 2 = B 2 B 3 = B 3 B 4 .
Step III: Join B3C and draw
B 4C′ || B 3 C to intersect BC
produced at C′. Also draw
C′A′ || CA to intersect BA produced at A′.
Hence, ∆A′BC′ ~ ∆ABC.
5. Steps of construction: In order to draw a pair of tangents to the given circle, follow the
following steps:
Step I: Draw a radius AO in the given circle with centre O and draw another radius making
an angle AOB of measure 180° – 60° = 120°.

196 M A T H E M A T I C S – X
Step II: Make ∠OAP = 90° and ∠BOP = 90° to intersect each other at P.
Such obtained AP and BP are the required tangents such that ∠APB = 60°.

6. We are given a ∆PQR with each


side of measure 6 cm.
Steps of construction: In order to
construct ∆ABC follow the
following steps:
Step I: Make an acute angle RQX
and locate seven points Q 1, Q 2,
Q3, Q4, Q 5, Q 6 and Q 7 on the ray
OX such that QQ 1 = Q 1Q2 = Q 2Q3
= Q 3Q4 = Q 4Q5 = Q 5Q6 = Q 6Q7.
Step II: Join Q7 R and draw Q6 C
parallel to Q7 R to intersect QR at
C.
Step III: Draw CA parallel to RP
to intersect BP (B and Q coincide)
at A.
Then, ∆ABC is the required
triangle.

C O N S T R U C T I O N S 197
WORKSHEET – 53 2. (B) Line segment A5B7 divides the line
segment AB in the ratio 5 : 7.
1. (A) Angle between the tangents is less than 180°. 3. False, because the point P lies inside the circle.

4.

Measuring the tangent AP, we get AP = 4.0 cm


5. We are given a circle of radius
4 cm and centre O.
Steps of construction: In order to
draw the required pair of tangents,
follow the following steps.
Step I: Draw a pair of radius OA
and OB inclined at an angle of
180° – 120° = 60° to intersect the
given circle at A and B respectively.
Step II: Draw perpendiculars AP and
BP which intersect each other at P.
Then AP and BP are the required
tangents.
Justification: In quadrilateral,
OBPA, applying Angle sum property,
we have
∠O + ∠A + ∠B + ∠P = 360°
⇒ 60° + 90° + 90° + ∠P = 360°
⇒ ∠P = 360° – 240° ⇒ ∠P = 120°.
Angle between the tangents is 120°.

198 M A T H E M A T I C S – X
6. Steps of construction:
First, we draw ∆ABC with the given
measurements. Then we draw another
triangle A′BC′ similar to ∆ABC and of
3
scalar factor using the following steps:
4
Step I: Draw a ray BX such that ∠CBX is
an acute angle.
Step II: Mark X1, X2, X3, X4 on BX such
that BX 1 = X1X2 = X2X3 = X3X4.
Step III: Join X4C and draw X3C′ || X4C.
Also, draw A′C′ || AC.
Thus, ∆A′BC′ ∼ ∆ABC.

In the figure, ∆A′BC′ ~ ∆ABC.

WORKSHEET – 54

1. (D) The minimum number of points should


be 9 as 9 > 5 out of the numerator and
9
denominator of . The next step is to be
5
joined B5 to C.
2. (B) Angle of inclination, here θ, can lie
between 0° and 180°. So, the most
appropriate option is (B), i.e., 0 < θ < 180°.
3. In the adjoining figure,
∆AB′C′ ~ ∆ABC such that

AB′ B′ C′ AC′ 3
= = = .
AB BC AC 2

C O N S T R U C T I O N S 199
4. Steps of construction:
First, we draw ∆ABC with the given measurements. Then we draw another triangle A′BC′
3
similar to ∆ABC and of scalar factor using the following steps:
4
Step I: Draw a ray BX such that ∠CBX is an acute angle.
Step II: Mark X1, X2, X3, X4 on BX such that BX1 = X1X2 = X2X3 = X3X4.
Step III: Join X4C and draw X3C′ || X4C.
Also, draw A′C′ || AC.
Thus, ∆A′BC′ ∼ ∆ABC.

1 5
5. 2 =
2 2
Steps of construction: In order to construct an isosceles triangle and another triangle having
5
of its corresponding sides, follow the steps given below:
2
Step I: Construct an isosceles triangle having any length of equal sides by drawing base
QR = 8 cm and altitude PM = 4 cm passing through the mid-point M of side QR.
Step II: Draw a ray QX such that ∠RQX is and acute angle; and divide the ray in five equal
parts, namely QQ1, Q1Q2, Q2Q3, Q3Q4 and Q4Q5.
Step III: Join Q2R and draw Q5R′ || QR intersecting QR produced at R′.

200 M A T H E M A T I C S – X
Step IV: Draw R′P′ || RP intersecting QP produced at P′.

P′Q QR′ P′R′ 5


Hence ∆P′QR′ is formed so that = = = .
PQ QR PR 2

C O N S T R U C T I O N S 201
6. Steps of construction: In order to draw the required pairs of tangents, follow the following
steps:
Step I: Draw a line segment AB of length 9 cm. Taking A as centre and radius 4 cm; and
B as centre and radius 3 cm, draw circles.

Step II: Find the mid-point M of AB. Then, taking M as centre and radius as AM = MB,
draw a circle to intersect circles drawn in step I at P, Q and R, S respectively.
Step III: Join AR, AS, BP and BQ.
Thus, obtained AR, AS and BP, BQ are the required pairs of tangents.

202 M A T H E M A T I C S – X
ASSESSMENT SHEET – 7
1. (C) The required angle say θ and the angle between the tangents are supplementary.
∴ θ + 120° = 180° ⇒ θ = 60°.
2. (C) Since AB is divided in the ratio s : t.
Therefore, the minimum number of points on AX would be s + t.
3. AB is the required tangent drawn from
point A to the circle with centre O
such that OA = 5 cm and OB = 3 cm.

4. True, because the angle between the tangents must be less than 180°.
5.

7
We are given a circle with centre O and radius = = 3.5 cm. We take points P and Q on
2
its extended diameter AB. Draw tangents PP1, PP2 from P and QQ1, QQ2 from Q, which
are the required tangents.

C O N S T R U C T I O N S 203
6. We are given a ∆PQR with
PQ = QR = RP = 5 cm. To
draw ∆AQB ~ ∆PQR and
with given scale factor,
draw ∠RQX < 90° and mark
Q1, Q 2, Q3 and Q4 such that
QQ 1 = Q 1 Q 2 = Q2 Q 3 = Q 3 Q 4 .
Join Q 4R and draw Q3B || Q4R
and AB || PR.
Hence ∆AQB ~ ∆PQR with

AQ QB AB 3
= = = .
PQ QR PR 4

7. First we draw ∆ABC using the given measurements. Further, we follow the steps given
below:
(a) Draw acute angle BCX and mark C1, C2 and C3 on it such that CC1 = C1C2 = C2C3. Join
BC3 and draw B′C2 || BC3 to meet BC at B′.

204 M A T H E M A T I C S – X
(b) Draw B′A′ || BA to meet AC at A′. ⇒ CC1+ C1C2 = 2CC1 and CC1 + C1C2
Thus ∆A′B′C ~ ∆ABC. + C2C3 = 3CC1
Justification: ⇒ CC2 = 2CC1 and CC3 = 3CC1
In ∆ABC and ∆A′B′C, AB || A′B′ and BC is
CC2 2
transversal. ⇒ =
CC3 3
∴ ∠ABC = ∠A′B′C′ = 90°
Similarly, ∠BAC = ∠B′A′C CB′ 2
⇒ = (Using Basic propor-
Therefore, ∆A′B′C ~ ∆ABC. CB 3
tionality theorem in ∆BCC3)
A′B′ B′ C A′ C
⇒ = = ...(i) From (i) and (ii),
ΑΒ BC AC
Let us take the ray CX. A′B′ B′ C A′ C 2
= = = .
CC1 = C1C2 = C2C3 ΑΒ BC AC 3

8. Let the common centre


of the two circles be O.
The point P is taken on
the outer circle. To draw
a pair of tangents from
P to the inner circle, we
follow the instructions
given below:
(a) Join PO and find its
mid-point M. Taking M
as centre, draw a circle
passing through P and
O to intersect the inner
circle at A and B.
(b) Join PA and PB.
PA and PB are the
required tangents.
Length of PA: On
measuring, the length of
PA is 4.0 cm.
Verification: Join AO.
AO = 3 cm.
In ∆AOP, ∠OAP = 90°
∴ PO2 = AO2 + PA2 (Pythagoras theorem)
⇒ 52 = 32 + PA2
⇒ PA2 = 25 – 9
⇒ PA2 = 16 ⇒ PA = 4
Clearly, PA is 4.0 cm.

C O N S T R U C T I O N S 205
ASSESSMENT SHEET – 8
1. (C) Q7 to R.
2. (D) 5 + 7 = 12.
3.

(a) Join OP and find its mid-point M.

AP : PB = 4 : 5. (b) Taking M as centre and radius = MP =


MO, draw a circle to intersect the given
4. False, because in the ratio 3 –1 : 3 +1 , circle at A and B.
i.e., 2 – 3 : 1, 2 – 3 is not a positive (c) Join PA and PB.
integer, while 1 is. PA and PB are the required tangents.
5. To draw a pair of tangents from P to the On measuring, PA = 6.35 cm and
circle with centre O, we follow the steps as PB = 6.35 cm. Clearly, PA and PB are of
given: same length.

6. We know that the angle between a pair of tangents to a circle and the angle between
their corresponding radii are supplementary. Therefore, the angle between these radii
= 180° – 90° = 90°.
PA and PB are the required tangents drawn from the external point P to the circle with
centre O and radius 4 cm.
In quadrilateral OAPB formed by tangents PA, PB and radii OA, OB, each internal
angle is of 90° and each side is of length 4 cm. Therefore, OAPB is a square. Perimeter
of square OAPB = 4 × 4 = 16 cm.

206 M A T H E M A T I C S – X
7. First we construct a ∆ABC with the
given measurements. Then we
construct a ∆A′BC′ similar to ∆ABC
5
and scale factor . For it, we follow
7
the steps given below.
(a) Draw a ray BX such that O < ∠CBX
< 90° and mark points B1, B2, B3, B4,
B5, B6 and B7 on it such that BB1 =
B1B2 = B2B3 = B3B4 = B4B5 = B5B6 =
B6B7.
(b) Join B7C and draw B5C' || B7C to
intersect BC at C′ and hence draw
A′C′ || AC to intersect AB at A′.
Thus ∆A′BC′ ~ ∆ABC.
Justification: In ∆CBB7,
BB 5 5
CB7 || C′B5 and = .
BB7 7
BC′ 5
So, by Thale's theorem, = ...(i)
BC 7
A′B 5
Similarly, in ∆ABC, = ...(ii)
AB 7
C O N S T R U C T I O N S 207
Now, in ∆ABC,
BC′ A′ B 5
= = [Using (i) and (ii); and ∠B = 90° (Given)]
BC AB 7

A′B BC′ A′C′ 5


∆A′BC′ ~ ∆ABC and = = = . Hence justified.
AB BC AC 7

8. First we draw an isosceles triangle ABC with base BC = 7 cm and altitude AD = 4 cm.
Altitude passes through the mid-point D of BC. Hence we construct a ∆A′BC′ similar to
1 3
∆ABC and of scalar factor 1 , i.e., using following the steps given below:
2 2

(a) Draw an acute angle CBX opposite to the vertex A with respect to BC.
(b) Mark points X1, X2, X3 on ray BX such that BX1 = X1X2 = X2X3.
(c) Join X2C and draw X3C′ || X2C to meet BC produced at C′.
(d) Draw C′A′ || CA to meet BA produced at A′.
3
Thus formed ∆A′BC′ is similar to ∆ABC and of scalar factor .
2

208 M A T H E M A T I C S – X
CHAPTER TEST

1. (B) A line segment can't be divided in the ratio 6 +1 : 6 – 1 , i.e., 7 + 2 6 : 5 as 7 + 2 6 is


not a positive integer while 5 is.
2. (C) Line segment P 3 Q 2 divides PQ in 3 : 2 at M. Therefore, P 3M : Q 2M = 3 : 2 and so
Q2M : P3M = 2 : 3.
1
3. True, because the irrational ratio 3 : can be converted into the rational ratio that is 3 : 1.
3
4. 180° – 60° = 120°
Radius = 3 cm
Required tangents are
PA and PB.

5. Steps of construction: In order to draw pairs of tangents, follow the steps given below.
Step I: Draw a line segment AB = 6 cm and then taking A and B as centres, draw the circles
of radii 3 cm and 2 cm respectively.

C O N S T R U C T I O N S 209
Step II: Find M as mid-point of AB and taking it as centre and radius as AM = MB, draw a
circle intersecting the circle having the centre as A at P, Q and the circle having the centre as
B at R, S.
Step III: Join AR, AS, BP and BQ.
Thus, AR, AS and BP, BQ are the required pairs of tangents.
6. Steps of construction: In order to construct triangles ABC and AQR, follow the steps given
below:
Step I: Draw any line XY and take any point D on it.
Step II: Draw any ray DZ such that ∠ZDY = 90°. Locate point C on DZ such that CD = 3 cm.
Step III: Make an ∠DCB = 30° such that CB intersects XY at B.
Step IV: Locate a point A on
XB such that AB = 5 cm and by
joining AC, we find ∆ABC.
Step V: Make an acute angle
YAT and locate T1, T2 and T3
on the ray AT such that
AT1 = T1T2 = T2T3.
Step VI: Join T 2B and draw
T3Q || T2B to intersect line AY
at Q. Also, draw QR to intersect
AC extended at R.
Thus, ∆AQR is obtained such
that
∆ABC ~ ∆AQR and
AQ QR AR 3
= = = .
AB BC AC 2

‰‰

210 M A T H E M A T I C S – X
Chapter

5 SOME APPLICATIONS OF TRIGONOMETRY

WORKSHEET – 57 6. Height = 94.64 m, Distance = 109.3 m


Hint:
3 QM
1. (A) cos 30° = tan 45° = ⇒ YM = QM
l YM
⇒ l = 2 3 m. But XP = YM
∴ XP = QM
65
2. (B) sin 30° = 40 + QM
l tan 60° = .
QM
⇒ l = 130 m.
7. Let BD be the tower of
height h m and CD be
3. Let the height of the the pole. In right-angled
triangle ABD,
tower be h.
BD
Perpendicular tan 45° =
tan 30° = AB
Base h
⇒ 1 = ⇒ AB = h
1 h AB
⇒ = ⇒ h = 10 3 m.
3 30 In right-angled triangle ABC,
4. Let OA be the horizontal BC BD + CD
tan 60° = ⇒ 3=
ground and K be the AB AB
position of the kite at a h+5
⇒ = 3
height h m above the h
ground, then AK = h m. It 5 5
is given that OK = 100 m, ⇒ h= ⇒ h=
3 −1 1.732 − 1
∠AOK = 60°.
In ∆AOK, right angled at A, we have ⇒ h = 6.83 m.
h 8. Let the chimney be AB with base B and
sin 60° = ⇒ h = 100 sin 60° anyone is walking from the point C to D.
100
In ∆ABD, ∠B = 90° and ∠D = 45°
3
⇒ h = 100 × = 50 3 = 50 × 1.732 ∴ ∠DAB = 45° ⇒ BD = BA ...(i)
2
∴ h = 86.60 m. In right angled ∆ABC,
AB
5. 4.28 m, 2.14 m tan 30° =
BC
3.7 AB
Hint: sin 60° = 1
l ⇒ =
3 CD + BD
3.7 1 AB
tan 60° = . ⇒ = [Using equation (i)]
x 3 50 + AB
⇒ 3 AB = 50 + AB

S O M E A P P L I C A T I O N S O F T R I G O N O ... 211
⇒ AB =
50
×
3 +1
=
50 ( 3 +1 ) 5. Let the balloon be at the point O, the thread
be OA and the required height be OB.
3 –1 3 +1 2
Case I. The cable is inclined at 60°.
⇒ AB = 25 ( )
3 + 1 = 25 (1.732 + 1)
⇒ sin 60° =
OB
= 68.30 m. OA
3 OB
⇒ =
WORKSHEET – 58 2 215

1. (A) ∠ACB = ∠XAC = 45° 215 3


⇒ OB =
2
20 20
sin (∠ACB) = and tan (∠ACB) = 215 × 1.732
x y
= = 186.19 m.
2
⇒ x = 20 2 m and y = 20 m.
Case II. The cable is inclined at 60° – 15°
h = 45°
2. (D) tan 60° = .
20 OB 1 OB
sin 45° = ⇒ =
h OA 2 215
⇒ 3=
20
215 2 215 2
⇒ OB = × =
⇒ h = 20 3 m. 2 2 2
3. Let the length of 215 × 1.414
shadow of pole = = 152 m (approx.)
2
AB be BC = x, then
So, reduced height = 186.19 m – 152 m
AB = x.
= 34.19 m.
Also let θ be the
angle of elevation of 6. Let AB is a hill and C and D be two city
Sun's altitude. centres subject to the angles of elevation
In right-angled of the top A of hill AB at C and D are 30°
triangle ABC, and 60° respectively, then ∠ACB = 30°,
x ∠ADB = 60°, AC = 9 km.
tan θ = x ⇒ θ = 45°
In right-angled ∆ABC,
Hence the angle of elevation of the Sun's
altitude is 45°. AB
sin 30° =
4. Let the angle of 9
elevation be θ. Let 1
the observer be AB ⇒ AB = 9 × sin 30° = 9 × = 4.5
2
with his eye at A
In right-angled ∆ABD, we have
and the tower be
EC. AB
sin 60° =
AD
∴ CD = AB = 1.5 m
⇒ AD = AB cosec 60°
ED = 30 – 1.5 = 28.5 m
2 9× 3
And AD = BC = 28.5 m ⇒ AD = 4.5 × =
3 3× 3
In right-angled ∆ADE,
DE 28.5 = 3 3 = 3 × 1.732
tan θ = = =1 ⇒ θ = 45°.
AD 28.5 = 5.196 ≈ 5.20 km.

212 M A T H E M A T I C S – X
7. Let the ladder and the 3. True.
wall upto which the AB h
ladder reaches be AC and tan C = =
BC x
BC respectively.
11h
(a) In ∆ABC, If AB =
10
BC
sin 30° = 11x
AC and BC =
10
1 BC
⇒ = ⇒ BC = 2 m. AB h
2 4 Then, tan C = = .
BC x
AB
(b) Also, cos 30° = 4. Let the height of the
AC
tower CD be y metres
3 AB and the horizontal
= ⇒ AB = 2 3 m.
2 4 distance of point A
8. 2 m from the building BC
is AB = x metres.
Hint: h = height of pedestal
In right-angled triangle
h ABC,
tan 45° = ⇒ x=h
x 20
tan 45° = ⇒ x = 20 m
x
h + 1.46
tan 60° = . Also, in right-angled triangle ABD,
h
20 + y
tan 60° =
x
WORKSHEET – 59
⇒ 20 3 = 20 + y
1. (B) In ∆ABC,
BC
⇒ y = 20 ( 3 −1 m)
tan A =
AB Thus, the height of the tower is 20 ( )
3 − 1 m.

⇒ tan A =
BC 5. 7 ( 3 +1 m)
3 BC
AE
(... AB = 3 BC) Hint: tan 45° =
EC
1 ⇒ EC = 7m
tan A = = tan 30°
3 DE
tan 60° = .
∴ Angle of elevation is 30°. EC
2. (C) In ∆ABC, 6. Let the width of the river be AC such that
AB AC = AB + BC
tan 60° =
BC
30
⇒ 3 =
BC
30
BC = , ∴ BC = 10 3 m. Let P be the point on the bridge such that
3 BP = 3 m.

S O M E A P P L I C A T I O N S O F T R I G O N O ... 213
In right-angled triangle ABP, 8. Let the tower be PQ and the objects be A
3 and B.
tan 30° = ⇒ AB = 3 3 m ∵ ∠XQA = 45°
AB
3 and ∠XQB = 60°
Also in ∆CBP, tan 45° = ⇒ BC = 3 m ∴ ∠QAP = 45°
BC
and ∠QBP = 60°
Now, AC = AB + BC= 3 3 + 3 = 3( 3 + 1) m
(Alternate angles)
Hence, width of the river is 3( 3 + 1) m.
In right ∆APQ,
7. Let the tower be BC the flagstaff ∠PAQ + ∠PQA = 90°
be AB and the point on the plane
⇒∠PQA = 90° – 45° = 45° (∵∠PAQ = 45°)
be P.
∴ AP = PQ = 150 (∵ PQ = 150 m)
Let BC = h
⇒ AB + BP = 150 ...(i)
In right-angled ∆BCP,
In right ∆BPQ,
h
tan 30° = PQ 150
PC tan 60° = ⇒ 3 = BP
BP
⇒ PC = h cot 30° ...(i)
150
In right-angled ∆ACP, ⇒ BP = ...(ii)
3
5+h
tan 60° =
PC 150
Putting BP = in equation (i), we get
⇒ PC = (5 + h) cot 60° ...(ii) 3
Comparing equations (i) and (ii), we have 150  1 
AB + = 150 ⇒ AB = 150  1 – 
h cot 30° = (5 + h) cot 60° 3  3
1 3 –1 3
⇒ h 3 = (5 + h) ⇒ AB = 150 × ×
3 3 3
⇒ 3h = 5 + h
⇒ h = 2.5
= 50 × 3 – 3 ( )
Hence, the height of the tower is 2.5 m. = 50 (3 – 1.73) = 50 × 1.27
OR ⇒ AB = 63.50 m
Let the two planes be at A and B respectively. Thus, distance between the two objects is
Also P be the point on the ground 63.50 m.
In right-angled triangle APC,
3125 WORKSHEET – 60
tan 30° =
PC OP a
1. (C) tan 45° = =
⇒ PC = 3125 3 m PQ QP
Also in right-angled triangle BPC, a
⇒ 1 =
BC QP
tan 60° =
PC ⇒ QP = a m
⇒ BC = 3125 3 × 3 = 3 × 3125 1
∴ AB = BC − AC = 3 × 3125 – 3125 ∴ ar(∆OPQ) = × QP × OP
2
= 2 × 3125 = 6250 m.
1 1
Hence, distance between the two planes is = × a × a = a2.
6250 m. 2 2

214 M A T H E M A T I C S – X
TP 40 3 +1
2. (B) tan 60° = ⇒ PM = ×
PO 3 –1 3 +1
TP
⇒ 3 =
40 ⇒ PM = 20 ( 3 +1 ) ... (iii)

⇒ TP = 40 3 m. From equations (ii) and (iii),

3. True PQ = 3 × 20 ( )
3 + 1 = 20 3 +( 3 )
AB Hence height of the tree and the width of the
tan 30° =
BC
1 AB
river are 20 3 +( )
3 m and 20 ( 3 +1 m )
⇒ =
3 81 respectively.

81 81 3 6. 6.34 m
⇒ AB = =
3 3 y1 + y2
Hint: tan 45° =
81×1.732 15
⇒ AB = = 46.76 m.
3 y1 + y2 = 15 …(i)
4. Let the height of the pole AB = x m y1
tan 30° =
Length of the rope AC = 20 m 15
In ∆ABC, 15
∠ACB = 30º ⇒ y1 = …(ii)
3
AB
∴ sin 30° = ∴ y2 = 15 − y1 = 6.34 m.
AC
1 x 7. Distance = 17.32 m, Height = 40 m
⇒ =
2 20 10
Hint: tan 30° =
⇒ x = 10 m x
∴ Height of the pole = 10 m. ⇒ x = 10 3 m
5. Let the tree be PQ, the width of the river be y
tan 60° =
MP and the person moves from M to S. 10 3
In right ∆PQS, ⇒ y = 30 m.
PQ 8. Let the plane moves from P to Q in 30
tan 45° =
PS seconds and the points H and K on the
PQ ground be just below the points P and Q
⇒ 1=
PS respectively.
⇒ PS = PQ
⇒ PM + MS = PQ
⇒ PM + 40 = PQ ...(i)
In right ∆PQM,
PQ PQ
tan 60° = ⇒ 3 =
PM PM
⇒ PQ = 3 PM ...(ii) In right ∆AQK,
From equations (i) and (ii), QK 1 3600 3
tan 30° = ⇒ =
PM + 40 = 3 PM AK 3 AK

S O M E A P P L I C A T I O N S O F T R I G O N O ... 215
⇒ AK = 10800 ...(i) 3. False.
In right ∆APH, Let height of the tower is h metres so the
angle of elevation is 30°.
PH
tan 60° = h
AH tan 30° =
BC
3600 3 1 h
⇒ 3 = ⇒ =
AH 3 BC
[∵ PH = QK = 3600 3 ] ⇒ BC = h 3 ... (i)
⇒ AH = 3600 ...(ii) When height = 2h,
Now, PQ = HK = AK – AH AB
tan θ =
⇒ PQ = 10800 – 3600 BC
[Using (i) and (ii)] 2h
⇒ tan θ = [From (i)]
⇒ PQ = 7200 h 3
2
Distance travelled ⇒ tan θ = ≠ tan 60°.
∵ Speed = 3
Time
4. Let AB be the ladder leaning against a wall
PQ 7200 OB such that ∠OAB = 60° and OA = 9.6 m.
∴ Speed = = In ∆OAB right angled at O, we have
30 30
3600 OA
cos 60° =
= 240 m/s = 240 × km/h AB
1000
= 864 km/h. OA
⇒ AB =
cos 60°
WORKSHEET – 61 9.6
⇒ AB = = 19.2 m.
0.5
AB
1. (A) tan C = 5. Let the point, cloud and reflection of the
BC
cloud be at P, Q and Q′ respectively.
AB
⇒ tan C =
AB
(∵ BC = AB)
⇒ tan C = 1 = tan 45°
⇒ C = 45°.

AB
2. (D) tan C =
BC
15
⇒ tan 60° =
BC Let PM = x, QM = y
15 We have to find QB, i.e., y + h
⇒ 3 =
BC In right-angled triangle QPM,
15 y y
⇒ BC = ⇒ BC = 5 3 m. tan α = ⇒ x= … (i)
3 x tan α

216 M A T H E M A T I C S – X
Also in right-angled triangle Q′PM, 8. Let the tower, the flagstaff
y + 2h and the point on the plane
tan β = be AB, BC and P respec-
x
tively.
y tan β
⇒ = y + 2h Let AB = y and AP = x
tan α
[From equation (i)] In ∆ABP,

 tan β  y
⇒ y − 1  = 2h tan α =
x
 tan α 
1 tan α
 2 tan α  ⇒ = ...(i)
⇒ y + h = h 1 +  x y
 tan β − tan α 
In ∆ACP,
h ( tan β + tan α )
= . h+y
tan β − tan α tan β =
x
Hence proved.
1 tan β
⇒ = ...(ii)
6. Hint: x h+y
From equations (i) and (ii), we have
tan α tan β
=
y h+y
⇒ y tan β = h tan α + y tan α
⇒ y (tan β – tan α) = h tan α

ED = AC = length of ladder = l (say) h tan α


⇒ y = . Hence proved.
b+x tan β – tan α
BC = y
Now, cos α = , sin α = ,
l l
WORKSHEET– 62
y+a x
cos β = , sin β = ,
l l AB
1. (C) sin 30° =
y y+a AC

cos α − cos β
Consider, = l l
1 AB
sin β − sin α x b + x
− ⇒ =
l l 2 15
−a a 15
= = . ⇒ AB = m.
−b b 2
7. 250 m
h 5 AB h
Hint: tan α = =
x 7 2. (B) tan C = =
BC 3h
h
tan β = 1
150 + x ⇒ tan θ = = tan 30°
3
1
= .
2 ⇒ θ = 30°.

S O M E A P P L I C A T I O N S O F T R I G O N O ... 217
3. True. 1
h−
AB ⇒ 2
As tan θ = 3 = [From (i) and (ii)]
BC 3
h−
2
AB
⇒ tan θ = 1 3 +1
AB ⇒ h= = = 1.366 km.
3 −1 2
(∵ AB = BC)
⇒ tan θ = 1 = tan 45° 6. Let the aero-
∴ θ = 45°. plane's first
situation be at
4. Let AB be the tower and C be the point on A and second
the ground. at B. Let the
In ∆ABC, point of obser-
vation be at O.
AB
tan 30° = From right-angled ∆AOD,
BC
AD
1 AB tan 45° = ⇒ OD = 3000 m
⇒ = OD
3 30
Again from right-angled ∆BOC,
30
⇒ AB = = 10 3 m. 3000
3 tan 30° =
3000 + DC
Hence, the height of the tower is 10 3 m.
5. Let the height (PQ) of mountain be h km.
⇒ DC = 3000 ( )
3 −1 m
In right-angled ∆ABC, Now speed of the plane

sin 30° =
BC
=
Distance
=
(
3000 3 − 1 )
1 Time 15

⇒ BC =
1
km [∵ D C = AB ]
2 = 146.42 m/sec
1 7. Let the height of
⇒ DP = km …(i)
2 the tower PQ and
AB the width of the
Also, cos 30° = canal AP be h and
1
x respectively.
3 Let the another
⇒ AB = km …(ii)
2 point be B such that
Similarly, from right-angled ∆APQ, AB = 20 m
AP = PQ = h …(iii) In right ∆APQ,
And from right-angled ∆CDQ , h
tan 60° =
x
DQ
3 = h= 3x ...(i)
CD
In right ∆BPQ,
h − DP
⇒ 3 = [From (iii)] h
h − AB tan 30° =
20 + x

218 M A T H E M A T I C S – X
⇒ h 3 = 20 + x ... (ii) BC
2. (A) sin 30° =
From equations (i) and (ii), we have AB
3x× 3 = 20 + x ⇒ 2x = 20 1 BC 5
⇒ = ⇒ BC = m.
⇒ x = 10 2 5 2

Substitute x = 10 in equation (i) to get 3. Wire is AB.


h = 10 3 CE = BD = 14 m.
AE = AC + CE
Hence, the height of the tower is 10 3 m
and the width of the canal is 10 m. ⇒ 20 = AC + 14
⇒ AC = 6 m
8. Let O be centre of
In ∆ABC, ∠C = 90°,
the balloon of radius
r and P the eye of 6
sin 30° =
the observer. Let PA, AB
PB be tangents from 1 6
P to the balloon. ⇒ = ⇒ AB = 12 m.
2 AB
Then,
∠APB = α 4. False, because the tangent of the angle of
α elevation doubles not
... ∠APO = ∠BPO = the angle of elevation.
2
... OL ⊥ PX, ∠OPL = β 5. BC is the multi-storeyed
building with the foot
α OA
... In ∆OAP, sin = B and the top C as the
2 OP
point of observation.
α
⇒ OP = r cosec AD is the building with
2 bottom A and the top D. Draw DE || AB
OL (see figure).
In ∆OPL, sin β =
OP Given angles are ∠XCD = 30° and
α ∠XCA = 45°. ∠CDE and ∠XCD are alternate
⇒ OL = r cosec sin β.
2 interior angles.
∴ ∠CDE = ∠XCD = 30°.
ASSESSMENT SHEET – 9
Similarly, ∠CAB = ∠XCA = 45°
1. (C) Given: BE = AD = 8 m

1 In right triangle ABC,


AB : BC = 1 :
3 CE + BE CE +8
tan 45° = ⇒1=
AB AB
i.e., AB : BC = 3 :1
⇒ AB = CE + 8 ...(i)
AB 3 Also, in right triangle DCE,
i.e., =
BC 1
CE 1 CE
tan 30° = ⇒ =
AB DE 3 AB
∴ tan θ = = 3 = tan 60°
BC (∵ DE = AB)
⇒ θ = 60°. ⇒ AB = 3 CE ...(ii)

S O M E A P P L I C A T I O N S O F T R I G O N O ... 219
From equations (i) and (ii), we get 7. Let the lower window, upper window and
the balloon be at A, B and C respectively.
( )
3 – 1 CE = 8
AB = AD = EF = FG = 2 m ...(i)
BG = AF ...(ii)
8 3 +1
⇒ CE = × In right triangle BCG,
3 –1 3 +1
CG
tan 30° =
8 × (1.73 + 1) BG
= = 10.92 m.
3 –1 1 CG
⇒ =
Substituting CE = 10.92 in (i), we get 3 BG
AB = 10.92 + 8 = 18.92 m 1
Further, BC = BE + CE = 8 + 10.92 ⇒ CG = BG
3
= 18.92 m ...(iii)
Hence, the height of the multi-storyed
building and the distance between the two In right triangle ACF,
buildings is 18.92 metres each. CF
⇒ tan 60° =
6. Let AB be the first tower AF
with bottom A and CD CG+ 2
be the second tower ⇒ 3 = [Using (i) and (ii)]
BG
with bottom C.
BE = 80 m CG+ 2
⇒ BG = ...(iv)
3
CD = 160 m
From equations (iii) and (iv),
AB = CE
∵ XD || BE and BD is the transversal 1 CG+ 2
CG = ×
3 3
∴ ∠DBE = ∠XDB = 30°
In right triangle BDE, ⇒ 3CG = CG + 2
DE CD – CE ⇒ CG = 1 ...(v)
tan 30° = = Now, CE = CG + FG + EF
BE BE
= 1 + 2 + 2 [Using (i) and (v)]
1 160 – AB
⇒ = =5
3 80
Hence, the height of the balloon is 5 metres.
80
⇒ 160 – AB = 8. Let the tower be AB, the flagstaff BC and
3 the point on the plane P.
80 80 3 Let AB = y and AP = x
⇒ AB = 160 – = 160 –
3 3 In ∆PAB,
AB y
480 – 80 3 tan α = = ...(i)
= AP x
3 In ∆PAC,
480 – 80 × 1.732 AC
= = 113.81 tan β =
3 AP
Hence, the height of the first tower is 113.81 h+y
⇒ tan β = ...(ii)
metres. x

220 M A T H E M A T I C S – X
From equations (i) and (ii), 4. False, the angle of elevation will remain
y h+ y unchanged if the height of the tower is
x = = increased by 10% too.
tan α tan β
5. Let the trucks be at A
y h y
⇒ = + and B; the balloon at P
tan α tan β tan β and the vertical line PQ.
∠XPA = 45°
 1 1  h
⇒ y – = ∠XPB = 60°
 tan α tan β  tan β
∠PAQ and
tan β – tan α h ∠PBQ are alternate
⇒ y = interior angles with
tan α tan β tan β
∠XPA and ∠XPB respectively.
h tan α ∴ ∠PAQ = 45° and ∠PBQ = 60°
⇒ y=
tan β – tan α In right ∆PAQ,

Hence, the height of the tower AB is PQ PQ


tan 45° = ⇒ 1=
h tan α AQ 100 + BQ
.
tan β – tan α ⇒ BQ = PQ – 100 ...(i)
In right ∆PBQ,
ASSESSMENT SHEET – 10 PQ PQ
tan 60° = ⇒ 3=
BQ BQ
10 2 1 10 2
1. (A) cos 45° = ⇒ = ⇒ PQ = 3 BQ = 3 (PQ – 100)
AC 2 AC
[Using (i)]
⇒ AC = 20 m.
25 ⇒ PQ ( )
3 – 1 = 100 3
2. (B) ∠YAM = ∠XYA = 45°; tan 45° =
AM 100 3
⇒ AM = 25 m ⇒ PQ =
3 –1
25
Time = = 5 seconds. 100 3 3 +1
5 ⇒ PQ = ×
3 – 1 3 +1
3. Let the kite be at A and the thread AB.
AC = 75 m; (
= 50 3 + 3 )
∠ABC = 60° Therefore, height of the balloon is
AC
sin 60° =
AB
(
50 3 + 3 metres. )
6. Let P be the point on the bridge; A and B
3 75
⇒ = are the two points on the opposite banks of
2 AB the river such that AB is the width of the
150 3 river. ∠X1PA = 45°, ∠X2PB = 30°.
⇒ AB = × Draw PC ⊥ AB to meet AB at C.
3 3
∠PAB = ∠X1PA
= 50 3 = 50 × 1.732
= 45°
⇒ AB = 86.6
∠PBA = ∠X2PB
⇒ The length of the string to the nearest
= 30°.
metre is 87 metres.

S O M E A P P L I C A T I O N S O F T R I G O N O ... 221
In right ∆ PAC, B′ M x+y
And cos β = = ...(v)
PC 30 A′B′ l
tan 45° = ⇒ 1= ⇒ AC = 30
AC AC Add equations (iv) and (v) to get
In right ∆PBC, x + y + 3z
3sin β + cos β = ...(vi)
PC 1 30 l
tan 30° = ⇒ =
CB 3 CB Divide equation (iii) by equation (vi) to get
3 sin α + cos α
⇒ CB = 30 3 = 1. Hence proved.
3 sin β + cos β
Now, AB = AC + CB = 30 + 30 3
= 30 + 30 × 1.732 8. Let the observer be at P on the ground.
= 30 + 51.96 = 81.96
Hence the width of the river is 81.96 metres.
7. Let the wall be AM.
Let the ladder changes its position from
AB to A′B′. Let the length of the ladder be
l such that
AB = A′B′ = l When the plane is at A vertically above C,
∠APC = 60°.
Let BM = y and
When the plane is at B vertically above D,
A′M = z
∠BPD = 30°.
In right triangle
AC = BD = 1.2 km = 1200 metres.
ABM,
AB = CD
AM In right triangle APC,
sin α =
AB
AC
x ⇒ tan 60° =
+z PC
⇒ sin α = 3
l AC 1200
⇒ PC = =
tan 60° 3
x + 3z
⇒ 3 sin α = ...(i) In right triangle BPD,
l
BD BD
BM tan 30° = =
And cos α = PD PC+CD
AB
1 1200
y ⇒ =
⇒ cos α = ...(ii) 3 1200
l + AB
3
Add equations (i) and (ii) to get
x + y + 3z 1200
3 sin α + cos α = ...(iii) ⇒ + AB = 1200 3
l 3
1200
In right triangle A′B′M, ⇒ AB = 1200 3 –
3
A′M z
sin β = = 2
A′B′ l ⇒ AB = 1200 ×
3
3z
⇒ 3 sin β = ...(iv) AB = 800 × 3 metres
l

222 M A T H E M A T I C S – X
Distance travelled AB 4. True, because the vertical tower, length of
Speed = = the shadow and the ray of the sun make a
Time 15 sec
right angled isosceles triangle.
800 3 m
= 5. Let the ships be at A and B; and the tower
15sec be PQ.
1
800 3 × km
= 1000
15
h
60 × 60

800 × 3 × 60 × 60
= km/h ∠PAQ = ∠XPA = 30°
1000 × 15 ∠PBQ = ∠XPB = 45°
= 192 3 km/h = 192 × 1.73 km/h In right ∆ BPQ,
= 332.16 km/h ∵ ∠PBQ = 45°, ∴ ∠BPQ = 45°
Hence, the aeroplane is flying at a speed of ⇒ BQ = PQ = 75 ...(i)
332.16 km/hr. In right ∆ PAQ,
PQ
CHAPTER TEST tan 30° =
AB + BQ
1. (A) From the adjoin- ⇒ AB + 75 = 75 3 [Using (i)]
ing figure, angle of
depression of P is
⇒ AB = 75 ( )
3 – 1 m.

∠XOP = α and angle 6. 8 3 m


of depression of Q is Hint: A′C = AC
∠XOQ = 90° – β. 8
cos 30° =
AC
h
2. (B) tan 30° = BC
20 3 tan 30° = .
8
⇒ h = 20 m. 7. Let the window be
at P and height of
the opposite house
3. Let the tower be BC and the length of
be h.
shadow be AB.
In right ∆ APQ,
BC
tan 60° = 60
AB tan 45° =
AQ
20 ⇒ AQ = 60 ⇒ BP = 60
⇒ 3 =
AB In right ∆ BCP,
20 h – 60
⇒ AB = m tan 60° = ⇒ 60 3 = h – 60
3 60
⇒ h = 60 + 60 3 = 60 1 + ( 3 )
⇒ AB = 20 3 m.
3 Thus, the required height is 60 1 + ( 3 m.)
S O M E A P P L I C A T I O N S O F T R I G O N O ... 223
8. Let the cloud be at C, the point of 1 h – 60
observation be at P and the reflection of ⇒ =
3 PQ
the cloud in the lake be at D. Let Q be any
point just below the cloud, 60 m above the ⇒ PQ = 3 ( h – 60 ) ...(i)
water level.
In right ∆DPQ,
h + 60
tan 60° =
PQ

h + 60
⇒ PQ = ...(ii)
3
Using equations (i) and (ii), we have
h + 60
⇒ 3 ( h – 60 ) =
3
⇒ 3h – 180 = h + 60
⇒ h = 120
In right ∆CPQ,
Hence, height of the cloud is 120 metres.
CQ
tan 30° =
PQ ❑❑

224 M A T H E M A T I C S – X
Chapter

6 PROBABILITY

WORKSHEET – 66 7. Let A = The event that 5 will not come up


either time.
1. (C) Sample space: {HH, HT, TH, TT} Now sample space is given by
Favourable events: {HT, TH} S ={(1, 1), (1, 2), (1, 3), (1, 4), (1, 5),
2 1 (1, 6), (2, 1), (2, 2), (2, 3), (2, 4), (2, 5),
∴ Required probability = = .
4 2 (2, 6), (3, 1), (3, 2), (3, 3), (3, 4),
2. (D) Probability of non-happening of an (3, 5), (3, 6), (4, 1), (4, 2), (4, 3), (4, 4),
event = 1 – Probability of happening of (4, 5), (4, 6), (5, 1), (5, 2), (5, 3), (5, 4),
that event (5, 5), (5, 6), (6, 1), (6, 2), (6, 3), (6, 4),
(6, 5), (6, 6)}
3 4
= 1− = . Total number of outcomes in sample space
7 7 n(S) = 36
3. (C) Sample space is {1, 2, 3, 4, 5, 6} and ∴ A = {(1, 5), (2, 5), (3, 5), (4, 5),
favourable events are {2, 3, 5} (5, 1), (5, 2), (5, 3), (5, 4),
3 1 (5, 5), (5, 6), (6, 5)}
∴ Required probability = = .
6 2 ∴ ( )
n A = 11
4. No, because the number of favourable out- ∴ n(A) = n(S) – n ( A ) = 36 – 11 = 25
comes of getting ‘6’ and ‘not 6’ are respecti-
n(A) 25
vely 1 and 5; and so their probabilities are (i) P(A) = =
n(S) 36
1 5
and . n(A) 11
6 6 (ii) P ( A ) = = .
n(S) 36
5. Sample space: {1, 2, 3, ......... ,99}
8. Total number of marbles = 5 + 8 + 4 = 17
∴ n(S) = 99.
i.e., n(S) = 17
The numbers divisible by 3 and 5 both are
numbers divisible by 15. (i) Let E1 be the event ‘a red marble is
taken out’.
So, favourable outcomes are: {15, 30, 45, 60,
75, 90} ∴ n(E1) = 5
Let E be the event getting a number n(E1 ) 5
Now, P(E1) = = .
divisible by 3 and 5. n(S) 17
∴ n(E) = 6 (ii) Let E2 be the event ‘a white marble is
n(E) 6 2 taken out’.
∴ P(E) = = = . ∴ n(E2) = 8
n(S) 99 33
n(E2 ) 8
1 5 Now, P(E2) = = .
6. (i) (ii) n(S) 17
23 46
(iii) Let E3 be the event ‘a non-green marble
Hints: is taken out’.
(i) Prime numbers are 5 and 7. ∴ n(E3) = 17 – 4 = 13
(ii) Perfect square numbers are n(E 3 ) 13
9, 16, 25, 36, 49. Now, P(E3) = = .
n(S) 17

P R O B A B I L I T Y 225
9. The sample space is OR
S = {1, 2, 3, 4, 5, 6, 7, 8} n(S) = 100
∴ n(S) = 8 Let E be the event of getting a prime.
(i) Let E1 be the event that the arrow will The primes from 1 to 100 are:
point at 8, then
2, 3, 5, 7, 11, 13, 17, 19, 23, 29, 31, 37, 41,
n(E1) = 1 43, 47, 53, 59, 61, 67, 71, 73, 79, 83, 89, 97.
n (E1 ) 1 ∴ n(E) = 25
∴ P(E1) = = .
n(S) 8
n(E) 25 1
(ii) Let E2 be the event that the arrow will Now, P(E) = = = .
n(S) 100 4
point at 1, 3, 5 or 7; then
n(E2) = 4 5. Let S be the sample space and E be the
event that the sum of numbers appearing
n(E2 ) 4 1
∴ P(E2) = = = . on the dice is a prime.
n(S) 8 2
∴ E = {(1, 1), (1, 2), (1, 4), (1, 6), (2, 1),
(iii) Let E3 be the event that the arrow will (2, 3), (2, 5), (3, 2), (3, 4), (4, 1), (4, 3),
point at 3, 4, 5, 6, 7, or 8; then (5, 2), (5, 6), (6, 1), (6, 5)}
n(E3 ) = 6 ∴ n(E) = 15, n(S) = 36
n(E3 ) 6 3 n(E) 15
∴ P(E3 ) = = = . Now, P(E) = = .
n(S) 8 4 n(S) 36
(iv) Let E4 be the event that the arrow will 6. The sample space is
point at 1, 2, 3, 4, 5, 6, 7 or 8; then
S = {HH, HT, TH, TT}
n(E4 ) = 8
∴ n(S) = 4
n(E 4 ) 8
∴ P(E4) = = = 1. (i) The outcomes for at least one head:
n(S) 8
{HH, HT, TH}
3
WORKSHEET – 67 ∴ Probability (at least one head) = .
4
1. (Α) (ii) The outcomes for at most one head:
Hint: Outcomes in favourable event of {HT, TH, TT}
getting the sum as a perfect square are
3
(1, 3), (2, 2), (3, 1), (3, 6), (4, 5), (5, 4), (6, 3). ∴ Probability (at most one head) = .
4
2. (D)
(iii) The outcomes for one head: {HT, TH}
Hint: |x| ≤ 4 ⇒ – 4 ≤ x ≤ 4
2 1
⇒ x = – 4, – 3, – 2, – 1, 0, 1, 2, 3, 4. ∴ Probability (one head) = = .
4 2
3. (B) P(getting a number less than 3)
OR
2 1
= = . Total number of students = 23
6 3
∴ n(S) = 23
4. No, because the theoretical probability of Let E be the event that the selected student
1 is not from A, B and C.
getting a head on tossing a coin is and
2 ∴ n(E) = 23 – 4 – 8 – 5 = 6
1
the experimental probability tends to n(E) 6
2 Now, P(E) = = .
when the number of tosses increases. n(S) 23

226 M A T H E M A T I C S – X
7. All possible outcomes are 15 5
(1 × 1), (1 × 4), (1 × 9), (2 × 1), (2 × 4), (2 × 9), ∴ n(E1) = 15 ∴ P(E1) = = .
36 12
(3 × 1), (3 × 4), (3 × 9), (ii) Favourable outcomes are
i.e., 1, 4, 9, 2, 8, 18, 3, 12, 27. E2 = {(4, 6), (5, 5), (5, 6), (6, 4), (6, 5),
∴ n(S) = 9 (6, 6)}
∴ n(favourable outcomes) = 5 6 1
∴ n(E2) = 6 ∴ P(E2) = = .
5 36 6
Hence, probability (xy is less than 9) = . (iii) Favourable outcomes are
9
8. Let S be the sample space, then E3 = (2, 2), (4, 4), (6, 6)
n(S) = 52 3 1
∴ n(E3) = 3 ∴ P(E3) = = .
(i) Let E1 be the event that the card drawn 36 12
is neither a hearts nor a king. (iv) Favourable outcomes are
Number of hearts = 13 E4 = {(2, 3), (2, 6), (4, 3), (4, 6), (6, 3),
Number of kings = 4 (6, 6), (3, 2), (3, 6), (3, 4), (6, 4), (6, 2)}
But one king is of hearts. 11
∴ n(E1) = 52 – 13 – 4 + 1 = 36 ∴ n(E4) = 11 ∴ P(E4) = .
36
n(E1 ) 36 9
Now, P(E1) = = = . WORKSHEET – 68
n(S) 52 13
(ii) Let E2 be the event that the card drawn 1. (C)
is an ace of spades. Hint: Sample space is {1, 2, 3, 4, 5, 6} and
Since number of ace of spades is 1 set of favourable numbers is {2, 4, 6}.
∴ n(E2) = 1 2. (A)
1 Hint: The sum of probabilities of having a
∴P(E2) =
52 particular event and not having the same
event is one.
(iii) Let E3 be the event that the card drawn
is either a black card or a king. 3. (D) Total number of coins
Number of black cards = 100 + 50 + 20 + 10 = 180
= Sum of numbers So, total number of elementary events
of cards of clubs = 180
and spades Favourable number of elementary events
= 13 + 13 = 26 = 180 – 10 = 170
Number of kings = 4 170 17
But 2 kings are black ∴ Required probability = = .
180 18
∴ n(E3) = 26 + 4 – 2 = 28
4. False, because the probability of each
28 7
∴ P(E3) = = . 1
52 13 outcome will be only when the two
2
9. Number of outcomes in sample space outcomes are equally likely otherwise not.
n(S) = 62 = 36
5. No, because areas of regions 3, 5 and 7 are
(i) Favourable outcomes are not equal.
E1 = {(1, 1), (1, 2), (1, 4), (1, 6),
(2, 1), (2, 3), (2, 5), (3, 2), (3, 4), 6. All possible outcomes are given by
(4, 1), (4, 3), (5, 2), (5, 6), (6, 1), {(1, 1), (1, 2), (1, 3), (1, 4), (1, 5), (1, 6),
(6, 5)} (2, 1), (2, 2), (2, 3), (2, 4), (2, 5), (2, 6),

P R O B A B I L I T Y 227
(3, 1), (3, 2), (3, 3), (3, 4), (3, 5), (3, 6), ∴ n(E2) = 5
(4, 1), (4, 2), (4, 3), (4, 4), (4, 5), (4, 6),
n(E 2 ) 5
(5, 1), (5, 2), (5, 3), (5, 4), (5, 5), (5, 6), ∴ P(E2) = = .
n(S) 46
(6, 1), (6, 2), (6, 3), (6, 4), (6, 5), (6, 6)}
9. Number of all possible outcomes,
5 n(S) = 52
(i)
36 (i) Number of spades = 13
Hint: Favourable outcomes: Number of aces = 4
(2, 6), (6, 2), (3, 5), (5, 3), (4, 4). But 1 ace is of spades.
(ii) 0 ∴ Number of favourable outcomes
Hint: No, favourable outcome is possible. = 13 + 4 – 1 = 16
(iii) 1 ∴ P(card drawn is a spades or an ace)
Hint: Favourable outcomes are the 16 4
= = .
same as the outcomes in sample space. 52 13
7. Let the number of black balls be x. (ii) Number of red kings = 2
So, total number of balls = x + 5 ∴ P(card drawn is a red king)
Probability of drawing a black ball is 2 1
x = = .
P1 = 52 26
x+5 (iii) Number of kings = 4
Also, probability of drawing a red ball is Number of queens = 4
5 ∴ Number of favourable outcomes
P2 =
x+5 = 52 – 4 – 4 = 44
According to the question, we have ∴ P (card drawn is neither a king nor a
P1 = 2. P2 queen)
44 11
x 2×5 = = .
⇒ = ⇒ x = 10. 52 13
x+5 x+5
(iv) Number of kings = 4
Hence, the number of black balls is 10. Number of queens = 4
8. Number of all cards = 50 – 5 + 1 = 46 Number of favourable outcomes
i.e., n(S) = 46 =4+4= 8
(i) Let E1 be the event that the number ∴ P(card drawn is either a king or a
on the card taken out is a prime less
8 2
than 10. queen) = = .
52 13
Prime numbers from 5 to 9 are 5
and 7.
ASSESSMENT SHEET – 11
∴ n(E1) = 2
1. (D) We know that probability (P) of any
n(E1 ) 2 1
∴ P(E1) = = = . 15
n(S) 46 23 event can be 0 ≤ P ≤ 1. Therefore, >1
14
(ii) Let E2 be the event that the number on can’t be probability of an event.
the card taken out is a perfect square. 2. (B) Let the number of bolts be x. Then
The perfect square numbers from x
5 to 50 are 9, 16, 25, 36 and 49. = 0.035 ⇒ x = 400 × 0.035 ⇒ x = 14.
400

228 M A T H E M A T I C S – X
3. Number of cards = 50 54
⇒ x= = 18
Prime numbers from 51 to 100 are: 53, 59, 3
61, 67, 71, 73, 79, 83, 89, 97
y
Therefore, number of all possible outcomes P(selecting a green marble) =
x+y+z
= 50.
And number of favourable outcomes = 10. 4 y
⇒ =
9 54
10 1
∴ Required probability = = .
50 5 4 × 54
⇒ y= = 24
4. True, 9
Ratio of probabilities Now, substituting x = 18, y = 24 in
= Ratio of areas of regions a, b and c x + y + z = 54, we get
= Ratio of areas of corresponding sectors 18 + 24 + z = 54
= Ratio of corresponding angles ⇒ z = 54 – 42
= 60° : 120° : 180° = 1 : 2 : 3. ⇒ z = 12
5. A leap year contains 366 days, wherever Hence, the jar contains 12 white marbles.
52 weeks and 2 days. There are 52 7. Number of blue triangles = 3
Thursdays in 52 weeks. Therefore, a leap Number of red triangles = 8 – 3 = 5
year consists 52 Thursdays and 2 days.
Number of blue squares = 6
These two days may be one choice out of
the seven given below: Number of red squares = 10 – 6 = 4
(i) Thursday and Friday Number of all possible outcomes
(ii) Friday and Saturday = 8 + 10 = 18
(i) P(a lost piece is a triangle)
(iii) Saturday and Sunday
(iv) Sunday and Monday 8 4
= = .
(v) Monday and Tuesday 18 9
(vi) Tuesday and Wednesday (ii) P(a lost piece is a square)
(vii) Wednesday and Thursday 10 5
= = .
So, two days are either Thursday and 18 9
Friday or Wednesday and Thursday. (iii) P(a lost piece is a blue square)
Clearly, the number of all possible 6 1
outcomes is 7 and the number of favourable = = .
18 3
outcomes is 2. (iv) P(a lost piece is a red triangle)
2
Hence, required probability = . 5
7 = .
18
6. Let number of blue marbles = x
Number of green marbles = y 8. Let E be the event of placing at least one
letter in the wrong envelope. Anyone
Number of white marbles = z
envelope can be filled with a letter in 4
Therefore, x + y + z = 54 ways. Second one can be filled by 3 ways.
x Third one can be filled by 2 ways. And the
P(selecting a blue marble) =
x+y+z fourth one can be filled with the remaining
one letter in 1 way. Therefore, the four
1 x
⇒ = letters are placed in the four envelopes in
3 54 4 × 3 × 2 × 1 = 24 ways. Out of these 24

P R O B A B I L I T Y 229
ways, only 1 way is such that all the letters 1
are placed in the right envelopes and 23 Therefore, the probability of getting 6 is
6
ways are such that at least one letter is
5
placed in the wrong envelope. and that of not 6 is .
6
23
∴ P(E) = . 5. All possible outcomes are given by
24
S = {1, 2, 3, ....., 1000}
ASSESSMENT SHEET – 12 ∴ n(S) = 1000
(i) Let E1 be the event that the first player
1. (B) P(getting X or Y) wins a prize. Then,
= P(getting X) + P(getting Y) E1 = Perfect square numbers
2 1 3 1 greater than 500 and less
= + = = . than 1001.
6 6 6 2
= 529, 576, 625, 676, 729,
2. (C) Given: P(E) = 3 P(E’) ... (i) 784, 841, 900, 961.
We have P(E) + P(E’) = 1 ... (ii ) ∴ n(E1) = 9
(i ) and (ii ) gives P(E) = 3 {1 – P(E)} n ( E1 ) 9
3 Now, P(E1) = =
i.e., 4 P(E) = 3, i.e., P(E) = . n(S) 1000
4
(ii) Let E2 be the event that the second
3. The number of outcomes when a pair of player wins a prize, if the first has won.
dice is rolled = 62 = 36.
∴ n(E2) = n(E1) – 1 = 9 – 1 = 8
The outcomes such that the sum is divisible
And number of all possible outcomes
by 3 are:
= n(S) – 1 = 1000 – 1 = 999
(1, 2), (1, 5), (2, 1), (2, 4), (3, 3), (3, 6), (4, 2),
(4, 5), (5, 1), (5, 4), (6, 3), (6, 6). n(E 2 ) 8
Now, P(E2) = = .
These are 12 outcomes. 999 999
The outcomes such that the sum is divisible 6. Let E1 be the event ‘the mobile phone is
by 2 are: acceptable to Varnika’ and E2 be the event
‘the mobile phone is acceptable to the
(1, 1), (1, 3), (1, 5), (2, 2), (2, 4), (2, 6), (3, 1),
trader’.
(3, 3), (3, 5), (4, 2), (4, 4), (4, 6), (5, 1), (5, 3),
(5, 5), (6, 2), (6, 4), (6, 6). ∴ n(E1) = Number of good mobile
phones
These are 18 outcomes.
= 42
The outcomes such that the sum is divisible
by 6 are: And n(E2) = Number of good mobile
phones + Number of mobile
(1, 5), (2, 4), (3, 3), (4, 2), (5, 1), (6, 6).
phones having only minor
These are 6 outcomes. defects
Now, the number of outcomes which are = 42 + 3 = 45
divisible by 3 or 2 is 12 + 18 – 6 = 24.
Number of all mobile phones is given by
24 2 n(S) = 48
Hence, the required probability = = .
36 3
n(E1 ) 42 7
4. No. (i) P(E1) = = =
n(S) 48 8
An unbiased die has six equally likely
outcomes. These are: 1, 2, 3, 4, 5 and 6. n(E 2 ) 45 15
(ii) P(E2) = = = .
Each of them has equal probability. n(S) 48 16

230 M A T H E M A T I C S – X
7. When two dice are thrown, the sample n(E 2 ) 4 1
space is given by Now, P(E2) = = = .
n(S) 8 2
S = {(1, 1), (1, 2), (1, 3), (1, 4), (1, 5), (1, 6),
(2, 1), (2, 2), (2, 3), (2, 4), (2, 5), (2, 6),
CHAPTER TEST
(3, 1), (3, 2), (3, 3), (3, 4), (3, 5), (3, 6),
(4, 1), (4, 2), (4, 3), (4, 4), (4, 5), (4, 6), 1. (B) Number of faces having B or C
(5, 1), (5, 2), (5, 3), (5, 4), (5, 5), (5, 6), =2+1=3
(6, 1), (6, 2), (6, 3), (6, 4), (6, 5), (6, 6)} Number of all faces = 6
∴ n(S) = 36 3 1
P(getting B or C) = = .
(i) Let A1 be the event ‘product of the 6 2
numbers on the top of dice is 6’. Then
2. (C) P(drawing a green ball)
A1 = {(1, 6), (2, 3), (3, 2), (6, 1)}
= 3 × P(drawing a red ball)
n(A1) = 4
n 5
n(A1 ) 4 1 ⇒ = 3× ⇒ n = 15.
∴ P(A1) = = = . 5+n 5+n
n(S) 36 9
3. (C) The even prime numbers from 1 to 6 is
(ii) Let A2 be the event ‘product of the
2 only
numbers on the top of dice is 12’.
1
Then, ∴ P(getting an even prime number) = .
A2 = {(2, 6), (3, 4), (4, 3), (6, 2)} 6
∴ n(E2) = 4 4. Case I. 2 dice are thrown.
Number of all outcomes in the sample
n(E2 ) 4 1
∴ P(E2) = = = . space, n(S) = 62 = 36
n(S) 36 9
Favourable numbers, n(E1) = 1
(iii) Let A3 be the event ‘product of the
1
numbers on the top of dice is 7’. Then, ∴ P(E1) =
36
A3 = Nil
∴ n(E3) = 0 Case II. 1 die is thrown.
Number of all outcomes, n(S) = 6
n(E3 ) (0)
∴ P(E3) = = = 0. Favourable numbers, n(E2) = 1
n(S) 36
1
8. On tossing a coin 3 times, all possible ∴ P(E2) =
outcomes are given by the sample space as 6
S = {HHH, HHT, HTH, HTT, THH, THT, So, the student throwing one die has the
TTH, TTT} better chance because he has more
probability.
∴ n(S) = 8
(i) Let E1 be the event of getting all heads, OR
then The sample space is
E1 = {HHH} S = {HHH, HHT, HTH, HTT, THH, THT,
∴ n(E1) = 1 TTH, TTT}
n(E1 ) 1 ∴ n(S) = 8
Now, P(E1) = = . The outcomes having at least two heads are
n(S) 8
E = {HHH, HHT, HTH, THH}
(ii) Let E2 be the event of getting at least
2 heads, then ∴ n(E) = 4
E2 = {HHH, HHT, HTH, THH} n(E) 4 1
∴ P(E) = = = .
∴n(E2) = 4 n(S) 8 2

P R O B A B I L I T Y 231
5. False, because there are equal probabilities Now, required probability
1
of getting the head or tail, that is . Area of circular region
2 =
Area of rectangular region
OR
Total number of outcomes, n(S) = 36
11
Favourable outcomes, 11 11
= 14 = = .
E = {(2, 6), (3, 5), (4, 4), (5, 3), (6, 2)} 6 14 × 6 84
∴ n(E) = 5
5 1 1
∴ P(E) = . 8. (i) (ii)
36 2 4
6. There are 52 cards in the pack. Therefore, Hint: A, B, C and D can be arranged in 24
the number of outcomes in sample space is ways. A before B can be arranged in the
given by following ways:
n(S) = 52 CADB, DACB, CABD, DABC, ABCD,
ABDC, CDAB, DCAB, ACDB, ADCB,
Number of hearts cards = 13
ACBD, ADBC.
Number of queens = 4
Out of these A just before B occurs in
Number of queens of hearts = 1 6 ways.
So, number of favourable outcomes is given
9. Let the number of white balls be x and the
by
number of red balls be y.
n(E) = 52 – (13 + 4 – 1) = 36
x y
Now, the required probability will be given Therefore, =
2 3
by
⇒ 3x – 2y = 0 ... (i)
n(E) 36 9
P(E) = = = . and 2(x + y) = 3y + 8
n(S) 52 13
⇒ 2x – y = 8 ... (ii)
7. Area of the rectangular region Solving equations (i) and (ii ), we get
= Length × Breadth x = 16, y = 24
= 3 × 2 = 6 m2 Number of non-red balls = number of white
Area of circular region balls = 16
= π × Radius2 P(choosing ball is not red)
22 1 11 2 16 16 2
= × = m = = = .
7 4 14 16 + 24 40 5
❑❑

232 M A T H E M A T I C S – X
Chapter

7 COORDINATE GEOMETRY

WORKSHEET – 72 1
1. (C)
= {– 5(5 – 5) – 4(5 – 7) + 4(7 – 5)}
2
Hint: Condition of collinearity must be
1
satisfied, = (0 + 8 + 8) = 8 sq. units.
2
i.e., k(− 2 − 4) + 6(4 − 3) + (− 3) (3 + 2) = 0
−3 7. Let A (1, 7), B (4, 2), C (− 1, − 1), D (− 4, 4)
⇒ − 6 k + 6 − 15 = 0 ⇒ k = .
2 be the vertices.
2. (B)
2 2
Hint: Let the ratio is k : 1 ∴ AB = ( 4 − 1) + (2 − 7 ) = 34
∴ Using section formula 2 2
−k + 5
BC = (− 1 − 4) + ( − 1 − 2 ) = 34
0= ⇒ k = 5.
k +1 2 2
3. 2 units
CD = ( − 4 + 1) + ( 4 + 1) = 34

2 2
Hint: Distance =
2
 2 + 8  + (2 − 2)2 AD = ( − 4 − 1) + (4 − 7 ) = 34
 
 5 5 Also,
10 2 2
=
5
= 2 units. AC = ( − 1 − 1) + ( − 1 − 7 ) = 68
4. 2 2 2
Hint: Use AO = OB and apply distance
BD = (4 + 4) + (2 − 4 ) = 68
formula. So, AB = BC = CD = AD and diagonals,
5. P(– 7, 0) i.e., AC = BD.
Hint: Any point on x-axis is P(x, 0) ⇒ ABCD is a square.
OR 8. Draw DE ⊥ AB and join BD.
Since, P is equidistant from A and B, Since, diagonals of a parallelogram bisect
each other.
∴ PA = PB ⇒ PA2 = PB2
⇒ (x – 3)2 + (y – 6)2 = (x + 3)2 + (y – 4)2
⇒ x2 – 6x + 9 + y2 + 36 – 12y
= x2 + 6x + 9 + y2 – 8y + 16
⇒ – 6x – 6x – 12y + 8y + 9 + 36 – 9 – 16 = 0
⇒ – 12x – 4y + 20 = 0
⇒ 3x + y – 5 = 0. Hence proved.
∴ Mid-point of AC= Mid-point of BD
6. LetA(– 5, 7) ≡ A(x1, y1); B(– 4, 5) ≡ B(x2, y2)
1+ a 2–4
and C(4, 5) ≡ C(x3, y3) ⇒ =
2 2
Now, area of ∆ABC
{∵ Comparing only x-coordinate}
1
= {x1 (y2 – y3 ) + x2 (y3 – y1 ) + x3 (y1 – y2 )} ⇒ 1+a =–2
2
∴ a = – 3.

C O O R D I N A T E G E O M E T R Y 233
Now, 4. Since diagonals of parallelogram bisect each
1 other.
ar(∆ABD) = {1(3 + 3) + 2(– 3 + 2) – 4(– 2 – 3)} ∴ Mid-point of AC = mid-point of BD
2
 6 + 9 1+ 4   8+p 5 
1 i.e.,  2 , 2  =  2 , 2
= (6 – 2 + 20) = 12 ...(i)    
2
Also, 15 8+p
⇒ = ⇒ p = 7.
1 2 2
ar(∆ABD) = × base × altitude OR
2
1 Given vertices are:
= × AB × DE A(– 3, 0), B(5, – 2) and C(– 8, 5). We know that
2
1 centroid G is  x1 + x2 + x3 , y1 + y2 + y3 
= × 26 × DE ...(ii)  3 3 
2
 −3 + 5 − 8 0 − 2 + 5 
 ∵ AB = 12 + 5 2 = 26  ∴ Centroid =  ,  = (−2, 1).
   3 3 
Comparing equations (i) and (ii), we have 5. True, because distance between (6, 4) and
1 24 26 (7, – 2); and distance between (6, 4) and
× 26 × DE = 12 ⇒ DE = × (5, – 2) are equal.
2 26 26
6. (1, 3)
12 26 Hint: Use section formula.
∴ DE =
13 7. Area of triangle
Hence, a = – 3 and required height
1
=  x1 ( y2 − y3 ) + x2 ( y3 − y1 ) + x3 ( y1 − y2 )
12 26 2
= units.
13
1
=  2 (0 + 4 ) − 1 ( − 4 – 3 ) + 2 ( 3 – 0 )
2
WORKSHEET – 73
1 21
1. (B) Let coordinates of A be (x, y). =  8 + 7 + 6  = sq. units.
2 2
As O will be mid-point of AB,
8. As the given points are collinear, the area
x+1 of the triangle formed by these points must
∴ 2= ⇒ x=3
2 be zero.
y+4 Let (2, 1) ≡ (x1, y1); (p, –1) ≡ (x2, y2); and
and − 3 = ⇒ y = − 10.
2 (– 1, 3) ≡ (x3, y3)
2. (C) Any point on y-axis be (0, y) Now, area of the triangle = 0
1
∴ (6 )2 + ( 5 − y )
2
=
2
(0 + 4 ) + ( 3 − y )
2 ⇒ [x (y – y3) + x2(y3 – y1) + x3(y1 – y2)] = 0
2 1 2
⇒ y= 9 1
⇒ [2(–1 – 3) + p(3 – 1) – 1(1 + 1)] = 0
∴ Point is (0, 9). 2
1
⇒ (– 8 + 2p – 2) = 0
3. (A) 2
Hint: Let the ratio is k : 1. ⇒ 2p = 10 ⇒ p = 5.
Now use section formula. 9. Hint: Proceed as done in solved example 5.

234 M A T H E M A T I C S – X
WORKSHEET – 74 Squaring both sides, we get

( )
2
(2a – 1)2 + (a + 2)2 = 5 2
1. (D)
⇒ 4a2 – 4a + 1 + a2 + 4a + 4 = 50
1
Area = {4(– 6 + 5) + 1(– 5 – 5) – 4(5 + 6)} ⇒ 5a2 = 45 ⇒ a = ± 3
2
Thus, a = ± 3.
1
= (– 4 – 10 – 44) = – 29 7. Let the required ratio be k : 1.
2
= 29 sq. units.
2. (B) Condition of collinearity must be satisfied
∴ − 5 (p + 2) + 1(− 2 − 1) + 4(1 − p) = 0 m1 x2 + m2 x1
⇒ − 5 p − 10 − 3 + 4 − 4p = 0 Using x=
m1 + m2
⇒ p = −1.
m1 y2 + m2 y1
3. 2 : 3 and y= , we have
m1 + m2
Hint: Use section formula.
4. True 1 – 7k + 3 9k + 5
= and 6 =
Hint: Any point P on x-axis will be of type 2 k+1 k+1
P(x, 0) ⇒ k + 1 = –14k + 6 and 6k + 6 = 9k + 5
∴ Let A (7, 6), B (− 3, 4) 1 1
⇒ k= and k =
∴ Use PA = PB. 3 3
5. k = – 8 Hence, the required ratio is 1 : 3.
Hint: Using section formula 8. Let D = Mid-point of BC
 7 + 5 3+7 
=  ,
 2 2 
= (6, 5)
2
∴ Median AD = (6 − 3 ) + 02
= 3 units.
Coordinates of A
9. Hint: Show that all sides are equal.
 1 × 6 + 2 × 3 −6 × 1 + 2 × 3 
=  ,  = (4, 0)
 3 3  WORKSHEET – 75
∴ As it lies on 2x + y + k = 0
⇒ 2 × 4 + 0 + k= 0  x + x + x 3 y1 + y2 + y 3 
1. (A) Centroid is  1 2 , 
∴ k = – 8.  3 3 
6. Let O be the centre and P be the point on  −3 + 5 − 8 0 − 2 + 5 
=  ,  = (− 2, 1).
the circumference such that O ≡ (2a, a – 7)  3 3 
and P ≡ (1, – 9).
2. (C) ar(∆ABC)
10 2
Radius = OP = 1
2 = {2(1 + 2) – 2(– 2 – 3) + 3(3 – 1)}
2
= 5 2 units
1
(6 + 10 + 6) = 11 sq. units.
i.e., ( 2a – 1)2 + ( a – 7 + 9)2 =5 2 =
2

C O O R D I N A T E G E O M E T R Y 235
8. 25 sq. units
3. (D) AB = (– 2 – 2)2 + (3 – 4)2 = 16 + 1
Hint: Join SQ.
= 17 . Find ar(∆PQS)
4. Let ratio is k : 1. and ar(∆RQS)
∴ Using section formula
3k − 2 7k + 2
2 = ; y =
k +1 k +1 ∴ Required area = ar(∆PQS) + ar(∆RQS).
7k + 2 9. Each side of a square and rhombus are
⇒ 4 =k ; y =
k +1 equal, but the diagonals of a square are
equal and that of a rhombus may or may
7 (4 ) + 2 not equal.
⇒ k =4 ; y =
4+1
=6 Side PQ = (3 – 2)2 + (4 + 1)2
∴ Ratio is 4 : 1; y = 6.
= 1 + 25 = 26
5. True,
∵ Pythagoras Theorem is satisfied. Side QR = (– 2 – 3)2 + (3 – 4)2
AB2 = (– 2)2 + (1 – 3)2
=4+4=8 = 25 + 1 = 26
AC = (– 1)2 + (4 – 3)2 = 2
2
Side RS = (– 3 + 2)2 + (– 2 – 3)2
BC2 = (1)2 + (3)2 = 10
∴ AB2 + AC2 = BC2 = 1 + 25 = 26
⇒ ∠A = 90°.
6. k = 0, 6 Side SP = (2 + 3)2 + (– 1 + 2)2
Hint: PQ = 58 ⇒ PQ2 = 58 = 25 + 1 = 26
⇒ (k − 3)2 + (2 + 5)2 = 58
⇒ (k − 3)2 = 9
Diagonal PR = (– 2 – 2)2 + (3 + 1)2
⇒ k − 3 = ± 3 ⇒ k = 0 or 6. = 16 + 16 = 4 2
7. Let the coordinates of R be (x, y)
Diagonal QS = (– 3 – 3)2 + (– 2 – 4)2

= 36 + 36 = 6 2

4× 3 + 3× 2 Clearly, the sides are equal but the


4× 2 + 3×1
Then, x = and y = diagonals are not equal. Hence, PQRS is a
4+3 4+3
rhombus but not a square.
11 18
⇒ x= and y = WORKSHEET – 76
7 7
Therefore, the coordinates of R are 1. (C) As given points are collinear.
So, a(b − 1) + 0 (1 − 0) + 1 (0 − b) = 0
 11 18 
 , . ⇒ ab − a − b = 0
7 7 
⇒ a + b = ab

236 M A T H E M A T I C S – X
1 1
⇒ + = 1.
a b

2. (B)

∵ A, B, C are collinear so condition of


collinearity must be satisfied ar(∆BCD)
⇒ x1(y2 – y3) + x2(y3 – y1) + x3(y1– y2) = 0
1
⇒ – 1(3 – 3) + (– 5) (3 – p) + 0(p – 3) =0 = {– 4(– 6 – 5) – 1(5 + 5) + 4(– 5 + 6)}
2
⇒ 0 – 15 + 5p =0
⇒ 5p = 15 1
= (44 – 10 + 4) = 19 sq. units.
p = 3. 2
3. (A) Now, ar(quadrilateral ABCD)
2 2
= ar(∆ABD) + ar(∆BCD)
Distance = (cos θ − sin θ ) + ( sin θ + cos θ )
= 53 sq. units + 19 sq. units
= 72 sq. units.
= (
2 sin 2 θ + cos 2 θ = ) 2.
7. Let the points P, Q and R divide AB into four
4. (A) Let A ≡ (x, – 1) and B ≡ (3, 2). equal parts AP = PQ = QR = RB as shown
AB = 5 below in the adjoining figure.

⇒ (3 – x)2 + (2 + 1)2 = 5
⇒ 9 – 6x + x2 + 9 = 25 (On squaring) Clearly, Q is the mid-point of AB
⇒ x2 – 6x – 7 = 0
 –2+2 2+8 
⇒ (x – 7) (x + 1) = 0 ⇒ x = 7 or – 1. ∴ Q≡  ,
 2 2 
5. False.
i.e., Q ≡ (0, 5)
6 + 9 1+ 4 P is the mid-point of AQ
As mid-point of AC =  , 
 2 2 
 –2+0 2+5 
∴ P≡  ,
 15 5 
=  ,   2 2 
 2 2
 7
i.e., P ≡  – 1, 
8+ p 2+3  2
and mid-point of BD =  , 
 2 2  and R is the mid-point of QB.
∴ Mid-point of AC = Mid-point of BD 0+2 5+8 
∴ R≡  , 
15 8+p  2 2 
⇒ = ⇒ p = 7.
2 2 13 
i.e., R ≡  1, .
6. ar(∆ABD)  2 
1 7
= {– 5(– 5 – 5) – 4(5 – 7) + 4(7 + 5)} 
Hence, the required points are P  – 1,  ,
2  2
1  13 
= (50 + 8 + 48) = 53 sq. units. Q(0, 5) and R  1,
2 .
 2 

C O O R D I N A T E G E O M E T R Y 237
8. Let P(2, – 3) and Q(10, y) be the given
points. Then, PQ = 10  2k + 1  7 k + 3
∴ 3 + –9 = 0
2  k +1  k +1
⇒ (10 − 2 )2 + {y – ( –3 )} = 10
⇒ 6k + 3 + 7k + 3 – 9k – 9 = 0
2
⇒ 64 + ( y + 3 ) = 10 3
⇒ 4k = 3 ⇒ k=
4
⇒ 64 + y 2 + 9 + 6y = 10 Hence, the required ratio is 3 : 4.
Squaring both sides, we get
y2 + 6y + 73 = 100 WORKSHEET–77
⇒ y2 + 6y – 27 = 0
⇒ 2
y + 9y – 3y – 27 = 0 1. (A)
⇒ y(y + 9) – 3(y + 9) = 0 Hint: Use condition of collinearity.
⇒ (y + 9) (y – 3) = 0 2. (D)
⇒ y = – 9 or 3. Hint: Any point on x-axis be (x, 0).
OR
Let ratio be k : 1
 9
 −1,  3. (C)
 2
Hint: Origin is (0, 0).
Hint:
4. False
Hint: A, B, C will not form a triangle.
R divides AB internally.
∵ ar(∆ABC) = 0
i.e. ratio 3 : 1.
Now, use section formula. 5. Diagonals AC and BD cut each other at the
mid-point P.
9. Let P (x, y), A (7, 1), B (3, 5)
As P is equidistant from A and B 3 + x1 2 + y1
∴ =2 ; =−5
... PA = PB ⇒ PA2 = PB2 2 2
⇒ (7 – x)2 + (1 – y)2 = (3 – x)2 + (5 – y)2 ⇒ x1 = 1 ; y1 = − 12.
⇒ 49 + x2 – 14x + 1 + y2 – 2y
= 9 + x2 – 6x + 25 + y2 – 10y
⇒ – 8x + 8y = – 16 ⇒ x – y = 2.
OR
Let the required ratio be k : 1 and the point
of division be (l, m) using section formula,
we have

x2 − 1 y2 + 0
Similarly, =2 ; =−5
2 2
2k + 1 7k + 3 ⇒ x2 = 5 ; y2 = − 10.
l= and m =
k +1 k +1
Hence, two other vertices of the parallelo-
 2k + 1 7 k + 3  gram are (1, – 12) and (5, – 10).
So, the point of division is  , .
 k +1 k +1  6. Area of quadrilateral ABCD
This point lies on the line 3x + y – 9 = 0. = ar(∆ABD) + ar(∆BCD)

238 M A T H E M A T I C S – X
1
= { 1(– 3 – 21) + 7(21– 1) + 7(1 + 3)} Coordinates of Q are  – 4 – 10 , 6 – 8 
2  2 2 

1 i.e., (– 7, – 1).
+ { 7(2 – 21) + 12(21 + 3) + 7(– 3 – 2)}
2  8 – 10 – 6 – 8 
Coordinates of R are  , ,
 2 2 
i.e., (– 1, – 7).
Now, ar(∆ABC)
1
= {8(6 + 8) – 4(– 8 + 6) – 10(– 6 – 6)}
2
1
= (112 + 8 + 120) = 120 sq. units.
2
1 1
= (– 24 + 140 + 28) + (– 133 + 288 – 35) and ar(∆PQR)
2 2
1
= 72 + 60 = 132 sq. units. = {2(–1 + 7) – 7(– 7 – 0) – 1(0 + 1)}
2
 1± 3 7 ± 5 3  1
7.  ,  = (12 + 49 – 1) = 30 sq. units.
 2 2  2
Hint: ar( PQR) 30 1
So, = =
As AB = BC = AC ar( ABC) 120 4
∴ AB = BC 1
⇒ AB2 = BC2 ⇒ ar(∆PQR) = ar(∆ABC).
4
⇒ (x − 3)2 + (y − 4)2 = 26 ...(i) Hence proved.
Similarly,
AC = BC ⇒ (x + 2)2 + (y − 3)2 = 26 ...(ii) WORKSHEET – 78
Solve (i) and (ii).
1. (A)
8. PA = PB Hint: Use formula of centroid of a triangle.
⇒ PA2 = PB2 2. (C)
⇒ (x − 3) + (y − 6)2 = (x + 3)2 + (y − 4)2
2
Using condition of collinearity,
⇒ x2 + 9 − 6x + y2 + 36 − 12y
k[3k – 1] + 3k[1 – 2k] + 3[ – k] = 0
= x2 + 9 + 6x + y2 + 16 − 8y
⇒ 3k2 – k + 3k – 6k2 – 3k = 0
⇒ 3x + y = 5.
⇒ – 3k2 – k = 0
 8– 4 –6+6  ⇒ – k(3k + 1) = 0
9. Coordinates of P are  , ,
 2 2  1
i.e., (2, 0). ⇒ k = − or 0.
3
3. (D) Using mid-point formula:
2a − 2 4 + 3b
1= ; 2a +1 =
2 2
⇒ a= 2 ; b = 2.

C O O R D I N A T E G E O M E T R Y 239
4. Let the ratio is k : 1 Here, LHS
∴ Using section formula, the point is
1
 8 k + 3 9k − 1  =
2
{
a ( c – b ) + b ( a – c) + c (b – a)}
p , 
 k +1 k +1 
1
As it lies on x − y − 2= 0 = ( ac – ab + ab – bc + bc – ac ) = 0
2
⇒ 8k + 3 − 9k + 1 − 2k − 2 = 0
⇒ − 3k + 2 = 0 = 0 = RHS. Hence proved.
2 8. 24 sq. units
⇒ k=
3 1
∴ Ratio is 2 : 3. Hint: Area of rhombus = d × d2
2 1
5. Let the coordinates of vertices of the triangle where d1, d2 = length of diagonals.
ABC are A(x1, y1), B(x2, y2) and C(x3, y3).
9. ar(∆ABC) = 5
1

2
{k (6 – 1) – 2 (1 – 2k ) + 3 ( 2k – 6 )} = 5
1
⇒ (5k – 2 + 4k + 6k – 18 ) = 5
2
Since, the point (1, 2) is the mid-point of AB. 1 15
x1 + x2 y +y ⇒ (15k – 20 ) = 5 ⇒ k – 10 = 5
∴ = 1, 1 2 = 2 2 2
2 2
15 15
i.e., x1 + x2 = 2, y1 + y2 = 4 ...(i) ⇒ k – 10 = ± 5 ⇒ k = 10 ± 5
2 2
Similarly, x2 + x3 = 0, y2 + y3 = – 2 ...(ii)
15 2
and x3 + x1 = 4, y3 + y1 = – 2 ...(iii) ⇒ k = 15 or 5 ⇒ k = 2 or .
2 3
∴ x1 + x2 + x3 = 3, y1 + y2 + y3 = 0
...(iv)
WORKSHEET – 79
Solving results (i), (ii), (iii) and (iv), we get
x1 = 3, y1 = 2, x2 = – 1, y2 = 2, x3 = 1, y3 = – 4. 1. (B) A(0, 0), B(3, 3 ), C(3, λ)
Hence the required vertices are A(3, 2), As AB = BC = AC
B(– 1, 2) and C(1, – 4).
∴ AB = AC ⇒ AB2 = AC2
6. (7, 2) or (1, 0)
⇒ 12 = 9 + λ2
Hint: ar(∆ PAB) = 10
⇒ λ2 = 12 − 9
⇒ ar(∆ PAB) = + 10 or ar(∆ PAB) = − 10
⇒ λ2 = 3 ∴λ=± 3
Let P (x, y) ∴ Use area of triangle
1 ∴ Required value of λ = − 3 .
=  x1 ( y2 − y3 ) + x2 ( y3 − y1 ) + x3 ( y1 − y2 ).
2 2. (D) Mid-point of (6, 8) and (2, 4) is P(4, 6).
7. The given points would be collinear, if the ∴ If A(1, 2), then
area of triangle formed by them as vertices
2 2
is zero. AP = ( 4 − 1) + (6 − 2 )
1
{
i.e., 2 a (c + a – a – b ) + b ( a + b – b – c ) = 9 + 16
+ c (b + c – c – a)} = 0 = 5 units.

240 M A T H E M A T I C S – X
3. 2 or – 4 8

Hence, the required points are P  1, – 
Hint: Use Pythagoras Theorem.  3
4.  10 
and Q  –1, –  .
 3 
PA 2
∴ = ⇒ PA : AQ = 2 : 1 7. A(1, 10), B(– 7, – 6), C(9, 2)
PQ 3
Hint: x1 + x2 = 2 × (– 3) = – 6
∴ Using section formula.
x2 + x 3 = 2
2 1
x= ; y= . Adding, x3 + x1 = 10
3 3
2 1 2(x1 + x2 + x3) = 6
∴ Coordinates of A are  ,  . x1 + x2 + x 3 = 3
3 3
∴ x1 = 1, x2 = – 7, x3 = 9
 
5.  2 − 11 , 5  ;  2 + 11 , 5  Using same method,
 2 2  2 2 y1 + y 2 + y 3 = 6
Hint: and y1 = 10, y2 = – 6, y3 = 2.
Let AB = AC = 3. 8. Area of ∆DBC
Use distance formula.
1
=  x {5 – (– 2)} + (− 3)(– 2 – 3 x) + 4(3 x – 5)
2

1
= (28 x – 14) = (14 x – 7)
6. Let the points of trisection be P(x1, y1) and 2
Q(x2, y2) such that P is the mid-point of
Area of ∆ABC
A(3, – 2), Q(x2, y2) and Q is the mid-point
of P(x1, y1), B(– 3, – 4). 1
=  6 {5 – (– 2)} + (− 3)(– 2 – 3) + 4(3 – 5)
2
i.e., AP = PQ = QB
1 49
= (42 + 15 – 8) =
2 2

⇒ AP : PB = 1 : 2 From question

Using section formula, ar( ∆DBC) 1


=
ar( ∆ABC) 2
– 3 + 2×3 – 4 + 2 × (–2)
∴ x1 = , y1 =
1+ 2 1+ 2 14 x – 7 1
⇒ =
49 2
8
⇒ x1 = 1, y1 = – 2
3
Again AQ : QB = 2 : 1 14x – 7 1 – 14 x + 7 1
⇒ = or =
49 2 49 2
2×(– 3) + 3 2×(– 4) – 2 2
∴ x2 = , y2 = 2
2+1 2+1
⇒ 8x – 4 = 7 or – 8x + 4 = 7
10 11 –3
⇒ x2 = – 1, y2 = – . ⇒ x= or .
3 8 8

C O O R D I N A T E G E O M E T R Y 241
WORKSHEET – 80 ⇒ 4a + 10b – 23 = 0
23 – 4 a
1. (B) As the distance of a point from x-axis is ⇒ b= ...(i)
10
equal to its y-coordinate, i.e., 3.
Using AD2 + CD2 = AC2, we have
2. (A) A(a + b, a − b), B(2a + b, 2a − b),
C(a − b, a + b), D(x, y) (a – 3)2 + (b – 4)2 + (a – 1)2 + (b + 1)2
Mid-point of AC = Mid-point of BD = (3 – 1)2 + (4 + 1)2
⇒ x = −b ⇒ a2 + b2 – 6a – 8b + 9 + 16 + a2 – 2a + b2
⇒ y = b. + 2b + 1 + 1 = 4 + 25
⇒ a + b2 – 4a – 3b = 1
2 ...(ii)
3.  – 1 , 0  ;  – 5 , 
2 Using equations (i) and (ii), we get
  
 3   3  2
 23 – 4 a   23 – 4 a 
Hint: a2 +   – 4a – 3   =1
 10   10 
⇒ 116a2 – 464a – 261 = 0
Use AP : PB = 1 : 2
⇒ 4a2 – 16a – 9 = 0
and AQ : QB = 2 : 1
(Dividing by 29)
4. Let the third vertex be C(x, y) of the given
∆ABC. Using, AC = AB and BC = AB, 16 ± 256 + 4 × 4 × 9 9 1
⇒a= = or –
we have x2 + y2 = 12 and 2× 4 2 2
2
(x – 3)2 + ( y – 3 ) = 12 9 1
Substitute a = and a = – successively
2 2
1 5
to get b = and b = .
2 2
9 1
Hence, the required vertices are  , 
 2 2
 1 5
Solving these, we obtain and  – ,  .
 2 2
x = 0, y = 2 3 or x = 3, y = – 3 .
OR
Hence, the required vertex is ( 0, 2 3 ) or
(4, 5), (2, 3), (6, 9)
( 3, – 3 ).
Hint:
5. Let ABCD be the given square such that
A(3, 4) and C(1, – 1). Let D(a, b) be the
unknown vertex.

x1 + x2 = 6 y1 + y2 = 8
∴ x2 + x3 = 8 y2 + y3 = 12
x1 + x3 = 10 y1 + y3 = 14

Using AD2 = CD2, we have Adding,


(a – 3)2 + (b – 4)2 = (a – 1)2 + (b + 1)2 2(x1 + x2 + x3) = 24 2(y1 + y2 + y3) = 34

242 M A T H E M A T I C S – X
⇒ x1 + x2 + x3 = 12 ⇒ y1 + y2 + y3 = 17 1 1 1 1
⇒ k2 + k+ – – =0
∴ x1 = 4 ∴ y1 = 5 2 16 16 2
x2 = 2 y2 = 3 2
2
 1 3
x3 = 6; y3 = 9. ⇒ k+  –   = 0
 4 4
6. Let O(x, y) be the circumcentre passing
 1 3  1 3
through A(3, 0), B(– 1, – 6) and C(4, – 1). ⇒  k + –  k + +  = 0
 4 4  4 4
Then OA = OB = OC
Taking OA = OB 1
⇒ k = – 1 or .
⇒ OA2 = OB2 (Squaring) 2

7. SP = ( at 2 – a)2 + (2at – 0)2

= a2 (t 2 – 1)2 + 4 a2 t 2

= a t 4 – 2t 2 + 1 + 4t 2 = a t 4 + 2t 2 + 1
⇒ (x – 3)2 + (y – 0)2 = (x + 1)2 + (y + 6)2 = a(t2 + 1)
⇒ x2 – 6x + 9 + y2 = x2 + 2x + 1 + y2 + 12y 2 2
+ 36  a   – 2a 
SQ =  2 – a  +  2 – 0 
⇒ 2x + 3y + 7 = 0 ...(i) t   t 
2
and OA = OC ⇒ OA = OC (Squaring)2
a2 2 a2 2 4 a2
⇒ (x – 3)2 + y2 = (x – 4)2 + (y + 1)2 = – + a +
t4 t2 t2
⇒ x – 6x + 9 + y2 = x2 – 8x + 16 + y2 + 2y
2

+1 1 2  1 
=a 4
+ 2 + 1 = a  2 + 1
⇒ x – y – 4= 0 ...(ii) t t t 
Solving equations (i) and (ii), we get
1 1 1 1
x = 1, y = – 3. Now, + = +
Thus, the coordinates of the centre are (1, – 3) SP SQ 2
a t +1 (
1 
a  2 +1  )
t 
2
Now, radius = OA = ( 3 – 1) + (0 + 3 )
2
1 1 t2 
=  2 + 
= 4+ 9 = 13 units.
a  t +1 1+ t 2 

OR  1  1 + t2
1
=
 = 
The area of the triangle formed by the given  a  1 + t 2
a
points must be zero.
which is independent of t.
1
i.e.,
2
{k ( 2k – 6 + 2k ) – (k – 1) (6 – 2k – 2 + 2k ) 8. As P(x, y) is mid-point of AB,

– ( 4 + k )( 2 – 2k – 2k )} =0
⇒ k(4k – 6) – (k – 1) × 4 – (4 + k) (2 – 4k) = 0 3+k 4+6
x= and y =
⇒ 4k2 – 6k – 4k + 4 – 8 + 16k – 2k + 4k2 =0 2 2
1 1 3 k
⇒ 8k2 + 4k – 4 = 0 ⇒ k2 + k– =0 i.e., x= + and y = 5
2 2 2 2

C O O R D I N A T E G E O M E T R Y 243
The value of x and y will satisfy x + y – 10 = 0 OR
3 k k 3 True,
∴ + + 5 – 10 = 0 ⇒ =5–
2 2 2 2 AB = (– 4 + 6)2 + (6 – 10)2 = 20 = 2 5
⇒ k = 7. BC = (3 + 4)2 + (– 8 – 6)2 = 245 = 7 5

WORKSHEET – 81 AC = (3 + 6)2 + (– 8 – 10)2 = 405 = 9 5

1. (A) Let the coordinates of the third vertex ∴ AB + BC = AC.


C be (x, y). Also, AB = 2 ⇒ AB = 2 AC .
AC 9 9
3–2+x 5 2 + 1+ y 1
∴ = and = 6. As A, P, B will be collinear,
3 3 3 3
i.e. x = 4 and y = – 2 ∴ a( y − b) + x(b − 0) + 0 . (0 − y) = 0
∴ C ≡ (4, – 2). ⇒ ay − ab + xb = 0 ⇒ ay + xb = ab
y x x y
⇒ + a =1 ⇒ a + = 1.
3 b b
2. (B) As BP = × (2 a )
2
7. Let the coordinates of P be (x, y).
= a 3
1
and OP = OA = a
2
AP 3 AB 7
∴ Coordinates of B are (a, 3 a). ⇒ = ⇒ =
AB 7 AP 3
3. (A) Let the required point be (h, k).
AB 7 PB 4
⇒ –1 = –1 ⇒ =
AP 3 AP 3

Then AP 3
⇒ = .
PB 4
3×(– 4) + 2 × 6 3× 5 + 2× 3 Using section formula, we have
h= and k =
3+2 3+2
3×2+ 4 ( – 2 ) 3×(– 4)+ 4× ( – 2 )
21 x= and y =
i.e., h = 0 and k = . 3+ 4 3+ 4
5
2 20
 21  i.e., x = – and y = –
So, the required point is  0,  . 7 7
 5
 2 20 
4. (A) Thus, the coordinates of P are  – , – .
 7 7 
Hint: Use AB = BC = AC
Take AB2 = BC2 8. Let the coordinates of P be (x, y).
and BC2 = AC2. PA = PB
5. True. ⇒ PA2 = PB2 (Squaring)
Let O(0, 0), A(5, 5) and B(– 5, 5) be the three ⇒ (x – 3) + (y – 4)2 = (x – 5)2 + (y + 2)2
2

points. ⇒ x2 – 6x + 9 + y2 – 8y + 16
= x2 – 10x + 25 + y2 + 4y + 4
∴ OA = 5 2 = OB ⇒ 4x – 12y – 4 = 0
and AB2 = 100 = OA2 + OB2. ⇒ x – 3y – 1 = 0 ...(i)

244 M A T H E M A T I C S – X
Area of ∆PAB = 10 Dividing equation (i) by equation (ii), we
1 7
⇒ { }
x ( 4 + 2 ) + 3 ( –2 – y ) + 5 ( y – 4 ) = 10 ar(∆PQR)
2 have = 4
ar(∆ABC) 7
6x – 6 – 3y + 5y – 20 = ± 20
6x + 2y – 26 = ± 20 1
⇒ ar(∆PQR) = ar(∆ABC).
⇒ 3x + y – 3 = 0 ...(ii) 4
or 3x + y – 23 = 0 ...(iii) 9. Since, A is on
the x-axis, so its
Now, we have to solve equations (i) and
coordinates will
(ii) as well as equations (i) and (iii).
be of the form
Solving equations (i) and (ii), we get (x, 0). Similarly,
x = 1, y = 0 the coordinates
Solving equations (i) and (iii), we get of B will be of
x = 7, y = 2 the form (0, y).
Since P is the mid-point of AB.
Hence, the coordinates of P are (1, 0) or
(7, 2). x+0 0+ y
∴ –2= and 3 =
OR 2 2
∵ P is mid-point of AB ∴ x = – 4 and y = 6
∴ Coordinates of A are (– 4, 0) and coordi-
 1+ 3 5 – 7  nates of B are (0, 6).
∴P≡  , ,
 2 2  Now,
i.e., P ≡ (2, – 1) PO = (– 2)2 + (3)2 = 4 + 9 = 13
∵ Q is mid-point of BC
PA = (– 4 + 2)2 + (0 – 3)2 = 4 + 9 = 13
 3+0 –7+4  3 3
∴Q≡  ,  , i.e., Q ≡  , – 
 2 2  2 2 Clearly, PA = PB = PO
⇒ P is equidistant from A, B and the
∵ R is mid-point of CA
origin O.
 0+1 4+ 5  1 9
∴R≡  ,  , i.e., R ≡  , 
 2 2  2 2 WORKSHEET – 82
Now, ar(∆PQR) 1. (A) (– 5, 1), (1, p) and (4, – 2) are collinear.
⇒ – 5(p + 2) + 1 (– 2 –1) + 4(1 – p) = 0
1  3 9 39  1 3 
= 2  – –  +  + 1  +  –1 +   ⇒ – 5p – 10 – 3 + 4 – 4p = 0 ⇒ 9p = – 9
2  2 2 22  2 2 
⇒ p = –1.
1 33 1  –7 7 2. (C)
=  –12 + +  = = ...(i)
2 4 4 4 4 1
Area = {1(4 – 6) – 2(6 – 3) + 0}
ar(∆ABC) 2

1 1
= (– 2 – 6) = 4 sq. units.
= {1(– 7 – 4) + 3(4 – 5) + 0(5 + 7)} 2
2
3. See Worksheet − 78, Sol. 4.
1 4. False, because P does not lie on the line
= (–11– 3 + 0) = – 7 = 7 ...(ii)
2 segment AB.

C O O R D I N A T E G E O M E T R Y 245
5. See Worksheet – 80, Sol. 5. AB – AD AC – AE 4–1
6. As P is equidistant from A and B, ⇒ = =
AD AE 1
PA = PB ⇒ PA2 = PB2 (Squaring)
⇒ (a + b – x)2 + (b – a – y)2 BD CE 3
⇒ = =
= (a – b – x)2 + (a + b – y)2 AD AE 1
⇒ (a + b – x) – (a – b – x)2
2 ⇒ AD : BD = AE : EC = 1 : 3
= (a + b – y)2 – (b – a – y)2 Let the coordinates of D and E be (x1, y1)
⇒ (a + b – x + a – b – x) (a + b – x – a + b + x) and (x2, y2) respectively.
= (a + b – y + b – a – y) Let us use section formulae.
(a + b – y – b + a + y) 1 × 1+ 3 × 4 1 × 5+3 × 6
x1 = , y1 =
⇒ 2(a – x) × 2b = 2(b – y) × 2a 1+ 3 1+ 3
⇒ ab – bx = ab – ay ⇒ bx = ay
Hence proved. 1 × 7+3 × 4 1 × 2+3 × 6
and x2 = , y2 =
OR 1+ 3 1+ 3
Let the third vertex 23
13
be C(x, y) of the i.e., x1 = , y1 =
4 4
given ∆ ABC such
that A(0, 0) and 19
and x2 = , y2 = 5
B (3, 3). 4

Using AC = AB  13 23 
So, the coordinates of D are  ,  and
i.e. AC2 = AB2 (Squaring)  4 4 
19
⇒ x2 + y2 = 9+3 of E are  , 5  .
⇒ x2 + y2 = 12 ...(i)  4 
Also, using BC = AC ar(∆ADE)
i.e., BC2 = AC2 (Squaring)
1   23  13 19  23  
⇒ (x – 3)2 + (y – 2
3) = x2 + y2 = 4  – 5  + (5 – 6) +  6 – 
2  4  4 4  4 
⇒ x2 – 6x + 9 + y2 – 2 3y + 3 = x2 + y2
1 3 13 19 1 
⇒ 3x + 3 y – 6 = 0 ...(ii) = 4 × – + × 
2 4 4 4 4
Solving equations (i) and (ii), we obtain
x = 0, y = 2 3 or x = 3, y = – 3 15
= sq. units.
Hence, the third vertex is (0, 2 3 ) or 32
(3, – 3). Again, ar(∆ABC)
1
AD AE 1 = {4(5 – 2) + 1(2 – 6) + 7(6 – 5)}
7. = = 2
AB AC 4
1 15
= (12 – 4 + 7) = sq. units
AB AC 4 2 2
⇒ = =
AD AE 1
15
AB AC 4 32
⇒ – 1= – 1 = –1 ∴ ar(∆ADE) : ar(∆ABC) = = 1 : 16.
AD AE 1 15
(Subtracting 1 throughout) 2

246 M A T H E M A T I C S – X
 a − 2a  3. D is mid-point of BC
8. P(at2, 2at), Q  2 ,  , S(a, 0)
t t   1– 3 5 –1
∴ D≡  , ;
∴ SP =  2 2 
( at2 − a)2 + (2 at)2
i.e., D ≡ ( – 1, 2)
= a 2 t 4 + a 2 − 2a 2 t 2 + 4 a 2 t 2
AD = (5 + 1)2 + (1– 2)2
= a2 t 4 + a2 + 2a2 t 2
= 36 + 1
= 2
( at + a) = at + a
2 2
= 37 units.
2 2 4. False, because area of the triangle formed
SQ =  a   −2a  by joining the given points is not zero as,
 2 − a +  
t   t 
1
area = 4(6 – 3) + 7(3 – 5) + 6(5 – 6)
a2 2 2 a2 4 a 2 2
= + a − + 2
t4 t2 t 1
= 12 – 14 – 6 = 4 ≠ 0.
2 a 2
=  a  = +a
 + a t2 5. As P(x, y) is the mid-point of A(3, 4) and
 t2 
B(k, 6),
1 1 1 t2
∴ + = 2 + 3+k 4+6
SP SQ at + a a + at 2 x= and y =
2 2
t2 + 1 1 (Using mid-point formula)
= 2
=
a(t + 1) a
3+k
which is independent of t. i.e. x = and y = 5.
2
Substituting these values of x and y in
ASSESSMENT SHEET – 13 x + y – 10 = 0, we get
2×7 + 3×3 23 3+k 3+k
1. (B) x = = + 5 – 10 = 0 ⇒ =5
2+3 5 2 2
⇒ 3 + k = 10 ⇒ k = 7.
6. Using the distance formula

2×9+ 3×5 33
y= =
2+3 5
23 33 
Hence, the coordinates of P are  , .
 5 5 
2. (B) Let the centroid be P(x, y).
2+ 5+3 1+ 2+ 4
x= ;y= ; Distance = ( x2 – x1 )2 + ( y2 – y1 )2 ,
3 3
10 7 we get
i.e., x = ;y=
3 3
AB = {– 4 – (– 5)}2 + (– 2 – 6)2
 10 7 
Thus P ≡  ,  .
 3 3 = 1 + 64 = 65

C O O R D I N A T E G E O M E T R Y 247
Since, P(h, k) lies on the line
BC = {7 – (– 4)}2 + {5 – (– 2)}2 2x + 3y – 5 = 0, therefore, x = h and y = k
must satisfy it.
= 121 + 49 = 170

2
and CA = (– 5 – 7 ) 2
+ (6 – 5 )

= 144 + 1 = 145
Since the lengths of all the sides of ∆ABC
are different, therefore, ∆ABC is a scalene
triangle.
7. Let the mid-points of sides AB and AC be
D and E respectively. Such that D(2, – 1)
and E(0, – 1).
1 1
AD = AB and AE = AC ...(i) Therefore, 2h + 3k – 5 = 0
2 2
According to the converse of Basic  2λ + 8   λ–9
⇒ 2  + 3 – 5 = 0
Proportionality Theorem, DE || BC, and  λ +1   λ +1 
therefore, ∆ADE ~ ∆ABC.
⇒ 4λ + 16 + 3λ – 27 – 5λ – 5 = 0
ar( ABC) AB2 (2AD)2
Now, = = ⇒ 2λ = 16 ⇒ λ = 8 ⇒ λ : 1 = 8 : 1
2
ar( ADE) AD AD2
2× 8 + 8 8–9
[Using (i)] ∴ h= and k =
8+1 8+1
⇒ ar(∆ABC) = 4 × ar(∆ADE) ...(ii) 8 1
i.e., h= and k = –
Now, ar(∆ADE) 3 9
1 8 1
= x (y – y3) + x2(y3 – y1) + x3(y1 – y2) ∴ P(h, k) = P  , –  .
2 1 2 3 9
1 Hence, the given line divides the given line
= 1(– 1 + 1) + 2(– 1 + 4) + 0(– 4 +1)
2 segment in the ratio 8 : 1 at the point
1 8 1
= 0 + 6 + 0 = 3 square units  , – .
2  3 9
Substituting this value of ar(∆ADE) in
equation (ii), we have ASSESSMENT SHEET – 14
ar(∆ABC) = 4 × 3 = 12 square units.
1. (C) Let the coordinates of P and Q be (x, 0)
8. Let the given line divides the given line
and (0, y) respectively.
segment at P(h, k) in the ratio λ : 1.
x+0 0+y
Using section formula, we have ∴ = 3 and =–7
2 2
λ × 2 + 1× 8 λ× 1 + 1× (– 9)
h= and k = i.e., x = 6 and y = – 14
λ +1 λ +1 Here, P ≡ (6, 0) and Q ≡ (0, – 14).
2λ + 8 λ–9 2. (C) Area of triangle formed by A, B and the
i.e., h= and k =
λ +1 λ +1 origin = 0

248 M A T H E M A T I C S – X
1 mx2 + nx1 my2 + ny1
⇒ 1(b – 0) + a(0 – 2) + 0(2 – b) = 0 x= and y =
2 m+n m+n
⇒ ± (b – 2a) = 0 ⇒ 2a = b.
3. Let the required point be P(h, k).
Then PO = PA = PB In this question,
∴ h2 + k2 = (h – 2x)2 + k2 – 2λ – 5 3λ – 4
and h2 + k2 = h2 + (k – 2y)2 –3 = and k =
λ +1 λ +1
⇒ h2 = h2 + 4x2 – 4xh
3λ – 4
and k2 = k2 – 4yk + 4y2 ⇒ – 3λ – 3 = – 2λ – 5 and k =
λ +1
⇒ h = x and k = y
∴ P is (x, y). 3× 2 – 4
⇒ λ = 2 and i.e., k =
2+1
4. False, because AB ≠ CD and BC ≠ AD as
2
AB = 2 2 + 12 = 5; ⇒ λ : 1 = 2 : 1 and k =
3
2
BC = (–1)2 + (–10)2 = 101 ; Hence, the ratio is 2 : 1 and k = .
3
2
CD = (– 8) + 112 = 185 ; 7. First, we find the length of each side of
quadrilateral ABCD.
2
AD = (– 7 ) + 22 = 53 .
AB = (1 – 5)2 + (5 – 6)2 = (– 4)2 + (–1)2
Diameter
5. Radius = r = = 16 + 1 = 17
2
2
10 2 BC = (2 – 1)2 + (1– 5)2 = 12 + ( – 4 )
= = 5 2 units
2 = 1 + 16 = 17
Coordinates of centre O are (2a, a – 7).
Coordinates of a point P on the circumference CD = (6 – 2)2 + (2 – 1)2 = 42 + 12
are (11, – 9)
= 16 + 1 = 17
∴ r = OP
AD = (6 – 5)2 + (2 – 6)2 = 12 + (– 4)2
⇒ 5 2 = (11– 2a)2 + (– 9 – a + 7)2
= 1 + 16 =
( ) 17
2
⇒ 5 2 = (11 – 2a)2 + (– a – 2)2
Clearly, AB = BC = CD = AD
(On squaring) All the sides of quadrilateral ABCD are
2 2
⇒ 121 + 4a – 44a + a + 4a + 4 = 25 × 2 equal.
⇒ 5a2 – 40a + 75 = 0 Therefore, ABCD is a rhombus. It may be a
(Dividing throughout by 5) square if diagonals are equal. To confirm it,
⇒ (a – 5) (a – 3) = 0 ⇒ a = 5 or 3 we have to find out the lengths of diagonal
Hence, a = 5, 3. AC and BD.

6. Let the required ratio be λ : 1. AC = (2 – 5)2 + (1 – 6)2 = (– 3)2 + (– 5)2


Here, we will use section formula as given
below. = 9 + 25 = 34

C O O R D I N A T E G E O M E T R Y 249
Now, ar(quadrilateral ABCD)
BD = (6 – 1)2 + (2 – 5)2 = 52 + (– 3)2
= ar(∆ABC) + ar(∆ACD)
= 25 + 9 = 34 35 109 144
= + =
Clearly, AC = BD. 2 2 2
Hence, quadrilateral ABCD is a square. = 72 square units.
8. To find area of quadrilateral ABCD, we Alternative Method:
divide it into two parts by either diagonal ar(quadrilateral ABCD)
(see graph).
Area of a triangle ABC 1
= [(x – x ) (y2 – y4) + (x2 – x4) (y3 – y1)]
2 1 3
1
= x (y – y ) + x2(y3 – y1) + x3(y1 – y2) 1
2 1 2 3 = [(– 5 + 1) (– 5 – 5) + (– 4 – 4) (– 6 – 7)]
2
1
= – 5 (– 5 + 6) – 4(– 6 – 7) – 1(7 + 5) 1
2 = [40 + 104] = 72 sq. units.
2
1 1
= – 5 + 52 – 12 = × 35
2 2
CHAPTER TEST
35
= sq. units
2 1. (D) Let the point of division be (x, y)
1× 3 + 2×7 1 × 4 + 2(– 6)
x= , y=
1+2 1+ 2

17 –8
⇒ x= ,y=
3 3

 17 8
⇒  , –  lies in the IVth quadrant.
 3 3

2. (A) Mid-point of hypotenuse AB is


equidistant from the vertices A, B and O.
Therefore, the required point is
 0 + 2x 2y + 0 
 2 , 2  , i.e., (x, y).
 

3. Let the required point be P(0, y) such that


PA = PB

1 ⇒ (0 – 1)2 + ( y – 5)2 = (0 – 4)2 + ( y – 6)2


ar(∆ACD) = – 5(– 6 – 5) – 1 (5 – 7) + 4(7 + 6)
2 ⇒ 1 + y2 – 10y + 25 = 16 + y2 – 12y + 36
1 1 (Squaring)
= 55 + 2 + 52 = × 109
2 2 ⇒ y = 13
109 Hence the coordinates of the point P be
= sq. units
2 (0, 13).

250 M A T H E M A T I C S – X
4. False, because Q lies outside the circle as 8. Let the coordinates of the vertices be
OQ > radius of circle. A(x1, y1), B(x2, y2) and C(x3, y3)
5. Observe the adjoining figure.

3 × 8 a + 1× (3 a + 1)
9a – 2 =
3 +1

3 × 5 + 1(– 3)
and –b=
3 +1
⇒ 36a – 8 = 24a + 3a + 1 x1 + x2 y1 + y2
3= and 4 =
and – 3b – b = 15 – 3 2 2
⇒ 9a = 9 and 4b = – 12 ∴ x1 + x2 = 6 ...(i)
Thus, a = 1 and b = – 3. and y1 + y2 = 8 ...(ii)
Similarly,
6. We know that area of a triangle is four
times the area of a triangle formed by x2 + x3 = 16 ...(iii)
joining the mid-points of it. y2 + y3 = 18 ...(iv)
x3 + x1 = 12 ...(v)
 1 5 7 7 y3 + y1 = 14 ...(vi)
D  – ,  , E(7, 3), F  , 
 2 2 2 2
Add equations (i), (iii) and (v) to get
ar(∆DEF) x1 + x2 + x3 = 17 ...(vii)
1 1 7  7 5 7 5  Add equations (ii), (iv) and (vi) to get
= –  3 –  + 7  –  +  – 3   y1 + y2 + y3 = 20 ...(viii)
2 2 2  2 2 2 2 
Subtract equation (iii) from equation (vii)
11 7 11 to get
=  + 7 –  = 4 sq. units.
24 4 x1 = 1
Since, D, E and F are the mid-points of Similarly, x2 = 5 and x3 = 11
sides of ∆ABC, Subtract equation (iv) from equation (viii)
∴ ar(∆ABC) = 4 × ar(∆DEF) to get
y1 = 2
11
=4× = 11 sq. units. Similarly, y2 = 6 and y3 = 12
4
Hence, the coordinates of the vertices of
7. See Worksheet – 72, Sol. 8. the ∆ABC are (1, 2), (5, 6) and (11, 12).
❑❑

C O O R D I N A T E G E O M E T R Y 251
Chapter

8 AREAS RELATED TO CIRCLES

WORKSHEET – 85 ∴ Area of major segment


= Area of circle − Area of minor segment.
π r 2 θ π × 8 × 8 × 135°
1. (A) A = = 8. (i) Length of wire used
360° 360°
2 = 2πr + 5 × (2r)
= 24 π cm .
22 35 35
2. (B) Perimeter = AB + BC + CD + length of = 2× × + 10 ×
7 2 2
q
arc AED = 110 + 175
= 20 + 14 + 20 + π × (7) = 285 mm.
= 76 cm.
(ii) Area of each sector
3. Perimeter = Outer arc length + Inner
π r2 θ
arc length + 2 × (14). =
360°
π × 30°
= (21 + 7) + 28 22 35 35
180° × × × 36°
7 2 2
=
22 28 44 360°
= × + 28 = + 28
7 6 3
385
= mm2.
44 + 84 128 2 4
= = = 42 cm.
3 3 3
4. No. WORKSHEET– 86
The correct statement is: "Area of a segment
1. (B) πR12 + πR22 = πR2
of a circle = Area of the corresponding
sector – Area of the corresponding triangle." ⇒ π(R12 + R22) = πR2
5. 154 m2 ⇒ R12 + R22 = R2.
Hint: Find area of shaded part. 2. (C)
Hint: Area of shaded part
π × 45°
=  212 − 7 2 
360°  
= 154 cm 2.
49 3. 228.57 cm2
6. (i) 77 cm2 (ii) cm2 Hint: Shaded portion
8 8
= Area of quadrant – Area of square ABCD
π r2
Hint: Area of quadrant = Radius OB = 20 × 2
4
7. 686.18 cm2, 20.32 cm2 = Diagonal of square.
Hint: Area of minor segment 4. No, because radius of the largest circle must
 πθ
= r2  −
sin θ  b πb 2
 be cm such that the area will be cm2.
 360° 2  2 4

252 M A T H E M A T I C S – X
5. Area of rectangle ABCD = AB × BC 7. 98.4 cm2
= 50 × 28 Hint: Inner radius = r
= 1400 cm2 ∆
∴ r= ;
BC 28 S
Radius of each semicircle (r) = = where ∆ = area of ∆ABC
2 2
= 14 cm. S = semiperimeter of ∆ABC.
1 ∴ Shaded part = ar(∆ABC) − ar(circle)
Area of each semicircle = πr2
2 8. (i) Perimeter of sector = 2r + Arc length
1 22 πrθ
= × × 14 × 14 = 308 cm2. = 2 × 5.7 +
2 7 180°
Now, area of remaining paper πr θ
∴ 27.2 = 11.4 +
= Area of rectangle ABCD – 2 × area 180°
of either semicircle πrθ
⇒ = 27.2 – 11.4
= 1400 – 2 × 308 180°
= 784 cm2 = 15.8 m
Further, observing the ∴ Arc length = 1580 cm.
adjoining figure, we (ii) 450300 cm2
have length of curve 1
APD or curve BQC. Hint: Area of sector = l r.
2
22
πr = × 14
7 WORKSHEET– 87
= 44 cm.
Now, perimeter of remaining paper 1. (A)
= AB + CD + curve APD + curve BQC Hint: 2πr = πr2
= 50 + 50 + 44 + 44 = 188 cm. ⇒ r = 2 units.
6. Radius = 6 cm 2. (A)
∴ Diameter PS Hint: 2 π r = 2 π r 1 + 2 π r2
= 2 × 6 = 12 cm ⇒ r = r1 + r2
Also, PQ = QR = RS = 19 + 9 = 28 cm.
12 3. False, because radii are different so their
= = 4 cm.
3 areas will also be different if their
corresponding arc lengths are equal.
p + length of QS
Perimeter = length of PQ p
4. 22 cm
o
+ length of PS
πrθ
Hint: Arc length = .
22 180°
= π [2 + 4 + 6] = 12 × = 37.71 cm.
7 5. Shaded part = Area of square − 4 × Area
π 2 π π of circle
Area = (6 ) − ( 4 )2 + ( 2 )2 = (14)2 − 4(πr 2)
2 2 2
π 22 22 7 7
= (36 − 16 + 4) = × 24 = 196 − 4 × × ×
2 7×2 7 2 2
528 264 = 196 − 154
= = = 37.71 cm2.
2×7 7 = 42 cm2.

A R E A S R E L A T E D T O C I R C L E S 253
OR 1 2
∴ Area of quadrant = πr
2.8 4
Length of arc AEB = π × = 1.4π cm.
2 1 11 11
= ×π × ×
4 π π
1.4
Length of arc BFC = π × = 0.7π cm.
2 121
= cm2.
2.8 + 1.4 4π
Length of arc ADC = π = 2.1π cm π r2 θ
2 3. Area of sector =
Now, perimeter of shaded region 360°
= Sum of lengths of the arcs AEB, BFC 3.14 × 4 × 4 × 30°
=
and ADC 360°
= 1.4π + 0.7π + 2.1π = 4.19 cm2
22 Area of major sector = ar(circle) – 4.19
= 4.2π = 4.2 ×
7 = 3.14 × (16) – 4.19
= 13.2 cm. = 46.05 cm2
4. True.
154
6. cm 2 Hint: ∠AOB = 60°
3
3
Hint: r = 14 cm ar(∆OAB) = × (side)2.
4
Angle swept in 1 min = 6°
1
Angle swept in 5 min = 30° 5. 51 cm 2
3
π r2 θ Hint: Use, area of shaded part
∴ Area = .
360° π
= × 40° R 2 − r 2 
7. 88.44 cm2 360°
R = Outer radius
π r2 θ 1 2
Hint: Use − r sin θ r = Inner radius.
360° 2
6. (i) The horse can graze in the shape of a
3 quadrant of a circle with radius 5 m
sin 120° = sin 60° = .
2 1
8. 66.5 cm2 ∴ Required area = π × (radius)2
4
Hint: See solved example 7.
1
= × 3.14 × 5 × 5
4
WORKSHEET – 88
= 19.625 m2.
1. (C) Circumference = 2πr (ii) Radius of quadrant of circle
22 = Length of the rope = 10 m
= 2× × 42
7 1
Area of the sector = × π × 102
= 264 cm. 4
1
792 × 100 = × 3.14 × 100
∴ No. of revolutions = = 300. 4
264
= 78.50 m2
2. (B) 2π r = 22 So, the increase in the grazing area
22 11 = (78.50 – 19.625) m2
⇒ r= =
2π π = 58.875 m2.

254 M A T H E M A T I C S – X
7. Let us mark the four unshaded parts as WORKSHEET – 89
I, II, III, IV in figure.
360°
1. (A) Angle = × 35 = 210° .
60
2. (C) Perimeter of square
= Circumference of circle
22
= 2πr = 2 × × 21 = 132 cm
7
132
∴ Side of the square = = 33 cm.
∴ Area of I + area of III = area of ABCD − 4
area of two semicircles of radius 5 cm each. 3. Let O be the centre of circle.
= 100 − 3.14 × 25 OD
In ∆OBD, cos 60° =
= 21.5 cm2 OB
Similarly, area of II + area of IV = 21.5 cm2 BD
and sin 60° =
Area of shaded part OB
= ar(ABCD) − ar(I + II + III + IV) OD 1 3 BD
⇒ = and =
= 100 − 2 × 21.5 32 2 2 32
= 57 cm2. ⇒ OD = 16 and BD = 16 3
8. Radius of the circle having ABC as quadrant
⇒ BC = 2BD = 32 3
= AB = AC = 14 cm ... Area of design
1 = Area of circle – Area of ∆ABC
Area of this quadrant ABC = × π × 142
4
 3 2
= 49π cm2 =  π × 322 −
4
( )
× 32 3  cm2
 
1
Area of isosceles ∆ABC = × AC × AB  3 × 1.73 
2 = 322 ×  3.14 –  cm
2
 4 
1
= × 14 × 14 = 98 cm2
2  12.56 – 5.19  2
= 1024 ×   cm
 4 
BC = AB 2 + AC 2
= 256 × 7.37 = 1886.72 cm2.
= 142 + 142 = 14 2 cm. 4. 161.31 cm2
Area of semicircle having diameter as BC 1 2
Hint: Required area = πr – ar(∆PRQ).
2
2
1  
= π ×  14 2  5.
2  2 
1
π × (7 2 ) = 49π cm2
2
=
2
Now, area of shaded region = Area of ∆ABC
+ Area of the semicircle with BC as (i) The distance around the track along the
diameter – Area of quadrant ABC inner edge = 106 + 106 + (π × 30 + π × 30)
= 98 + 49π – 49π = 98 cm2. (∵ Inner radius = 30 m)

A R E A S R E L A T E D T O C I R C L E S 255
22 7. Inner area of ring I = π × (Radius)2
= 212 + × 60
7 2
= π ×  32 
1320 2804 4  2
= 212 + = m = 400 m.
7 7 7 = 256π cm2
(ii) The area of track 2
π Outer area of ring I = π ×  34  = 289π cm2
= (106 × 80 − 106 × 60) + 2 ×  40 2 − 302   2
2 
22 So, area of ring I = Outer area – Inner
= 2120 + 700 ×
7 area
= 2120 + 2200 = 4320 m2. = 289π – 256π
6. AB = 56 m = 33π cm2
Area of lawn ABCD = AB2 = 562 = 56 × 56 2
= 3136 m2 ... (i) Inner area of ring II = π ×  19 
 2
= 90.25π cm2
2
21
Outer area of ring II = π  
 2

= 110.25π cm2
So, area of ring II = 110.25π – 90.25π
= 20π cm2
Draw OM ⊥ AB.
Therefore, total area of these two rings
AC = 2 AB = 56 2 m = (33π + 20π) cm2
AC 56 2 = 53π cm2.
AO = = = 28 2 m
2 2 8. Sum of areas of 4 quadrants
⇒ Radius (r) of circular flower bed
1 
= 28 2 m = 4 ×  π × Radius 2 
4 
BC 56
OM = = = 28 m = π × 12 = π cm2
2 2
Area of each circular flower bed Area of the circle in the middle
= Area of sector AOBP – Area of ∆AOB = π × (Radius)2
1 1 2
= πr2 – × AB × OM
4 2 = π ×  2  = π cm2
 2
1 22 1
= × × 28 2 × 28 2 – Area of the square = (Side)2
4 7 2
× 56 × 28 = 42 = 16 cm2.
= 1232 – 784 Now, area of the remaining portion
= 448 m2 ...(ii) = 16 – π – π = 16 – 2π
Using equations (i) and (ii), we get the
22
required area = 16 – 2 ×
7
= 3136 + 2 × 448
= 9.71 cm2.
= 4032 m2.

256 M A T H E M A T I C S – X
ASSESSMENT SHEET – 15
2 2
16 12
1. (A) πR2 = π   + π  
 2   2 
⇒ R2 = 64 + 36 = 100
⇒ R = 10 m.
120°
2. (C) Area = π(21)2 × ∴ ar(semicircle SPM) + ar(semicircle RQN)
360°
1 1 22 88 2
22 1 = πr2 + πr2 = πr2 = ×4= m
= × 21 × 21 × 2 2 7 7
7 3
SP = RQ = 2r = 4 m
= 462 cm2.
PQ = SR = AB – 3 – 3 – r – r
3. Let central angle be θ. = 26 – 3 – 3 – 2 – 2 = 16 m
θ Now, ar(rectangle SPQR) = SP × PQ
Area = πr2 ×
360 ° = 4 × 16 = 64 m2
θ
⇒ 54π = π × 362 × Area of the shaded region
360°
88
54π ×360° = 312 – − 64
⇒ θ= = 15° 7
36×36 × π = 312 – 12.57 – 64
Now, length of arc = 235.43 m2.
22 6. ACB is the minor segment of a circle with
πrθ × 36 × 15°
(l) = = 7 centre O and radius OA = OB = r = 14 m
180° 180° Join AB.
22 66 In ∆OAB,
= ×3 = = 9.43 cm.
7 7 OA = OB (Radii of same circle)
Alternative Method: ⇒ ∠2 = ∠1 ... (i)
(Angles opposite to equal sides)
1
Area of sector = lr. ∴ ∠1 + ∠2 + 60° = 180°
2
(Angle sum property for a triangle)
1
⇒ 54π = × l × 36 ⇒ ∠1 + ∠1 + 60° = 180° [Using (i)]
2
⇒ ∠1 = ∠2 = 60° [Using (i)]
54 × 22 × 2 66 ⇒ ∆AOB is equilateral
∴ l= =
7 × 36 7
= 9.43 cm.
4. False, as the corresponding radii are
unequal so the area is different.
5. Join SP and QR.
ar(rectangle ABCD) = AB × CD = 26 × 12
= 312 m2
Semicircles SPM and RQN are congruent. 3 2 3
∴ ar(∆AOB) = r = × 142
Radius(r) of the semicircle SPM 4 4
1
= (12 − 4 − 4) = 2 m = 49 3 m 2
2

A R E A S R E L A T E D T O C I R C L E S 257
60° ∴ ar(∆ABC) = 24(24 − 15)(24 − 16)(24 − 17)
ar(sector AOBC) = πr2 ×
360° 2
=24 × 9 × 8 × 7 = 24 21 m
22 1
= × 14 × 14 × Now, area of shaded region
7 6
308 2 = ( 24 21 − 77 ) m2
= m Hence, area of ungrazed region
3
Now, ar(segment ACB) = ( 24 21 − 77 ) m2.
= ar(sector AOBC)
– ar(∆AOB) 1
8. ar(∆ABC) = × Base × Height
2
308
=  
− 49 3  m 2 . (∵ ∠A = 90°)
 3  1
= × 10 × 10 = 50 cm2.
7. Sides of ∆ABC are AB = 15 m, BC = 16 m 2
and CA = 17 m. Areas grazed by three 90°
ar(sector APR) = π(7)2 ×
animals tied at A, B and C are respectively 360°
22 1
APU, BQR and CST. = ×7×7×
7 4
The ungrazed area is shaded, i.e., PQRSTU
= 38.5 cm2.
(see figure) which to be required.
ar(PBCR) = ar(∆ABC) – ar(sector APR)
In ∆ABC,
θ1 + θ2 + θ3 = 180° ... (i) = 50 – 38.5 = 11.5 cm2.
Cost of the silver plating on PBCR
(Angle sum property of a triangle)
= ar(PBCR) × Rate
ar(sector APU) + ar(sector BQR)
= 11.5 × 20
+ ar(sector CST)
θ θ θ = ` 230.00.
= πr2 × 1 + πr2 × 2 + πr2 × 3
360° 360° 360°
ASSESSMENT SHEET – 16
πr 2
= (θ + θ2 + θ3)
360° 1 1.54
1. (B) πr2 = 1.54 ⇒ r2 = × 7 ⇒ r = 0.7 m
22 72 22
= × × 180° [Using (i)]
7 360° 176
Number of revolutions = = 40.
22
= 77 m2 2 × × 0.7
7
2. (A) This rhombus must be a square with
diagonals as the diameters of the circle.
πr2 = 1256
1256 × 7
⇒ r2 = ⇒ r = 20 cm (approx.)
22
∴ Diameters = d1 = d2 = 40 cm
1
∴ Area of the rhombus = × d1 × d2
2
For ∆ABC, semiperimeter 1
= × 40 × 40
15 + 16 + 17 2
s= = 24 m
2 = 800 cm2.

258 M A T H E M A T I C S – X
3. Minutes elapsed by minute hand ∴ Area of corresponding segment
= 6.40 – 6.05 = 35 = Area of APB
∵ Angle covered by minute hand in 2
πr θ
60 minutes = 360° A= – ar(∆AOB)
360°
∴ Angle covered by minute hand in
As ∆AOB is equilateral
360° 2
35 minutes = × 35 = 210° π(10) × 60 3
60° ∴ A= – × 10 × 10
360° 4
210°
∴ Area = πr2 ×
360°  157 
= − 25 3  cm2.
22 7 275  3 
= × 52 × =
7 12 6 7. Area of square ABCD = BC2
5 = 142 = 196 cm2
= 45 cm2.
6 EFG, GHI, IJK and KLE are four semicircles
4. True, because when areas are equal, the 14 − 3 − 3
radii are equal and so circumferences are of radius = 2 cm each.
equal. 4
EGIK is a square of side as the diameter of
5. In ∆AOB, AB2 = 202 = 400
either circle, that is, 4 cm.
AO2 + BO2 = (10 2 ) + (10 2 )
2 2
And
= 400
i.e., AO2 + BO2 = AB2 ⇒ ∠AOB = 90°
1
∴ ar(∆AOB) = × AO × BO
2
1
= × 10 2 × 10 2
2
= 100 cm2
90°
ar(sector AOB) = π × (AO)2 ×
360°
= 50π cm2
Sum of areas of the four semicircles
ar(rectangle ABCD) = AB × BC = 4 × area of one semicircle
= 20 × 10 = 200 cm2 1
Now, area of the shaded region = 4 × π × 22 = 8π cm2
2
= ar(rectangle ABCD)
Area of square EGIK = (Side)2 = 42 = 16 cm2
– ar(sector AOB) + ar (∆AOB)
Now, area of shaded region
= 200 – 50π + 100
= Area of square ABCD
= 300 – 50π = 50(6 – π) cm2.
– Sum of areas of four semi-
6. r = 10 cm circles – Area of square EGIK
Let AB is the chord = 196 – 8π – 16
which subtend an = (180 – 8π) cm2.
angle of 60° at centre
of circle. 8. In right ∆ABC,
AC2 = AB2 + BC2
(Pythagoras theorem)

A R E A S R E L A T E D T O C I R C L E S 259
⇒ AC2 = 142 + 142 2. (B)
⇒ AC = 14 2 cm Hint: Diameter of circle = 16 cm
∴ Diagonal of a square = 2 × side.
1
ar(∆ABC) = × Base × Height 2 × side = 16
2
1 side = 8 2
= × 14 × 14 ∴ Area of square = (8 2)2
2
= 98 cm2 = 128 cm2.
Radius of quadrant ABCP = AB = BC = 14 cm 3. Let radius of given circle and side of given
ar(quadrant ABCP) square be r and a respectively.
1 Perimeter of the circle = perimeter of the
= × Area of corresponding square
4
circle 2a
⇒ 2πr = 4a ⇒ r =
1 1 22 π
= × π(AB)2 = × × 14 × 14
4 4 7 πr 2 π 4a 2
Required ratio = = ×
= 154 cm2 a 2
a 2
π2
2
1 AC  4 4 14
ar(semicircle ACQ) = × π ×   = = =
2  2  π 22 11
7
1 22 14 2 14 2
= × × × i.e., 14 : 11.
2 7 2 2
4. True.
[∵ AC = 14 2 ]
= 154 cm2
Now, area of shaded portion
= ar(semicircle ACQ)
– ar(quadrant ABCP) 5
+ ar(∆ABC) Let radius of smaller circle = r =
2
= 154 – 154 + 98 = 98 cm2. 5
Let radius of larger circle = R = 2.  
Hence, area of shaded portion is 98 cm2. 2
2
5
CHAPTER TEST 2 
πR 2 2
∴ Ratio of areas = = = 2 : 1.
1. (A) Two vertices of the πr 2 5
2

triangle should coincide  


2
with the two extremities 5. Length of arc = Length of the wire = 20 cm
of the diameter of the
60° × π π
semicircle and the third Angle = 60° = radian = radian
180° 3
one lies on the curve.
Radius of the curve ABC
1
ar(∆ABC) = × BC × AO Length of arc
2 =
Angle
1
= × 2r × r
2 20 60
⇒ r= π = cm.
= r2 sq. units. π
3
260 M A T H E M A T I C S – X
π 1
ar(∆ABC) = × AB × BC
Area of sector OABC = 3 × πr2 2

1
1 60 × 60 = × 14 × 48
= ×π× 2
6 π×π
(A1) = 336 cm2 ... (i)
600
= cm2.
π
6. Let the angle subtended by arc at the centre
be θ.
Area of sector = 54π
θ
⇒ × π × (Radius)2 = 54π
360°
θ
⇒ × π × (36)2 = 54π Now area of sectors drawn at each corner
360° of triangle ABC
54
⇒ θ = 360° × = 15° πr 2θ1 πr 2 θ2 πr 2θ3
36 × 36 (A2) = + +
360° 360° 360°
Now, length of the arc = Radius × Angle in
radian πr 2
= (θ + θ2 + θ3)
15° 360° 1
= 36 × ×π
180° π×5×5
= 3π cm. = × 180°
360°
7. In ∆ABC, (Using Angle sum property)
BC = 48 cm, AC = 50 cm, AB = 14 cm 25π 25 × 3.14
Here, AB2 + BC2 = 142 + 482 = =
2 2
= 196 + 2304
= 25 × 1.57 = 39.25 cm2
= 2500 = (50)2
∴ Required area = A1 – A2
= AC2
= 336 – 39.25 = 296.75 cm2.
So, ∆ABC is a right-angled triangle with
8. See Assessment Sheet – 16, Sol. 7.
∠ABC = 90°
❑❑

A R E A S R E L A T E D T O C I R C L E S 261
Chapter

9 SURFACE AREAS AND VOLUMES

WORKSHEET – 93 1
So, volume of x lead shots = × capacity
1. (C) Edge of cube = 4 cm 4
∴ Length of resulting cuboid = 4 + 4 = 8 cm of the cone
Breadth of resulting cuboid = 4 cm 4 1
⇒ x× π × (0.5)3 = × 64π
Height of resulting cuboid = 4 cm 3 4
∴ Surface area = 2(lb + bh + hl) 16 × 3 48
⇒ x= = = 96
= 160 cm2. 4 × 0.125 0.5
2. (B) Volume of big sphere Hence, the required number of shots is 96.
4 6. Volume of earth dug out = πr2h
V1 = π (3)3
3
22
Volume of 1 small sphere = × 7 × 7 × 20 = 154 × 20 = 3080 m3
7
4 Area of platform = 22 × 14 − π r2
V2 =π (0.3)3
3 22
= 22 × 14 − × 72
∴ Number of balls obtained 7
V1 27 = 154 m2
= = = 1000.
V2 0.027 Now, height of the platform
3. 3 : 1. Volume of earth dug out
=
Hint: Volume of cylinder = V1 = πr2h Area of platform
1 2 3080
Volume of cone = V2 = πr h = = 20 m.
3 154
1
∴ V1 : V2 = 1 : = 3 : 1. 14 7
3 7. Radius of the pipe = r = = 7 cm = m
2 100
4. 176 mm2
Rate of flowing of water
Hint: = 15 km/h
15 × 1000 25
= m/s = m/s
3600 6
Surface area of capsule Let the required time be t.
= Lateral surface area of cylinder Water flowing through the pipe in 1 second
+ 2 × Lateral surface area of hemisphere = πr2 × Rate of water flowing
5. Volume of water filled in the cone 22 7 7 25 3 77
= × × × m = m3
1 1 7 100 100 6 1200
= π r2h = π × 4 × 4 × 12
3 3 ∴ Water flowing into the tank in t seconds
= 64π cm3 77
Let the number of lead shots be x. = t m3
1200

262 M A T H E M A T I C S – X
This volume of the water is the same as ∴ Volume of sphere = Volume of wire
the volume of the water in the tank at the 4 4
height of 21 cm. ⇒ × 27 = l
3 100
77 21 ⇒ l = 900 cm.
∴ × t = 50 × 44 ×
1200 100 2. (B)
50 × 44 × 21 × 1200 h
⇒ t = Hint: V =  A1 + A2 + A1 A2 
77 × 100 3 

= 7200 seconds = 5(20 + 8) = 140 cm3.


⇒ t = 2 hrs. 3. Let the base radii be 3x and 5x respectively
and let the same height be h.
8. r = 8 cm; R = 20 cm; h = 16 cm
1 1 2
2
V1 = π (3x)2 × h ; V2 = π (5x) × h
∴ Slant height = l = 2
h + (R – r ) 3 3
V 9x 2
= 16 2 +12 2 ∴ 1 = , i.e., V1 : V2 = 9 : 25.
V2 25x2
= 400 = 20 cm. 4. 53.625 cm2
Hint:
H = height of cone
= 7.75 – 1.75
= 6 cm

l = slant height of
∴ Total surface area of container cone
= πl (R + r) + πr2 = r2 + H2
= π [l (R + r) + r2]
= 3.14 [20 × 28 + 64] ∴ T.S.A.
= 1959.36 cm2 = C.S.A. of cone + C.S.A. of hemisphere
= π r l + 2π r 2
∴ Cost of metal sheet used
= π r [l + 2 r].
15
= 1959.36 × 5. T.S.A. = C.S.A. + 2 × (C.S.A. of hemisphere)
100
= ` 293.90. = 2π r h + 2(2π r 2) = 2π r h + 4π r 2
22
= 2π r [h + 2 r] = 2 × × 3.5 [10 + 7]
WORKSHEET – 94 7
= 2 × 22 × 0.5 × 17
4 = 374 cm2.
1. (A) Volume of sphere = π(3)3
3 6. 4158 cm 3
Let length of wire = l. Hint:
4 2 Volume of solid
Radius of wire (r) = = 2 mm = cm.
2 10 = Volume of cone
+ Volume of hemisphere
 2 2
∴ Volume of wire = πr2l = π   × l 1 2 πr
2
 10  = πr h + 2 πr3 = (h + 2r).
3 3 3

S U R F A C E A R E A S A N D V O L U M E S 263
7. Volume of material of hollow spherical shell Now,
= External volume – Internal volume Cost = Surface area × Rate
4 4 = 17.85 × 1.20 = 21.42
= π (5)3 – π (3)3
3 3 Hence, the volume is 8792 cm3 and cost is
` 21.42.
4 4 × 98
= π (53 – 3 3 ) = π cm3
3 3 WORKSHEET – 95
Let the radius of the cylinder be r.
1. (A) Let the increase in level be h.
8
Volume of the cylinder = πr2 × cm 3 Increase in volume of water = Volume of
3
the sphere
As volume of the material will remain
4
same. ⇒ π × 82 × h = × π × 63
3
8 4 × 98
∴ πr2 × = π ⇒ h = 4.5 cm.
3 3
2. (D) Let radius of the cone = r
∴ r2 = 49 ⇒ r = 7 ⇒ 2r = 14
1 2 4
Hence, the diameter of the cylinder is πr x = πx 3
3 3
14 cm.
⇒ r = 2x cm.
20
8. Radius of bottom = r1 = = 10 cm 3. Let the required number of cubes be n.
2
Volume of n cubes = Volume of the ball
40
Radius of top = r2 = = 20 cm 4 22 21 21 21
2 ⇒ n × 13 = × × × ×
Height = h = 12 cm 3 7 2 2 2
⇒ n = 4851.
πh 2
Volume of the bucket = (r + r22 + r1 r2)
3 1 4. T.S.A. of remaining solid
= C.S.A. of cylinder
3.14 × 12
= (102 + 202 + 10 × 20) + Area of circular base
3 + C.S.A. of cone.
= 12.56 × 700 = 8792 cm3 = 2πrh + πr 2 + πr l
Slant height of the bucket = πr [2h + r + l];
2 22
l= h2 + (r1 – r2 ) =
7
× 0.7 [4.8 + 0.7 + 2.5]

2 ∴ l = ( 2.4 )2 + (0.7 )2 = 2.5 cm 


= 122 + ( 20 – 10 )  
= 244 = 2 61 = 2 × 7.81 = 17.6 ≈ 18 cm2.

= 15.62 cm 5. Volume of 1 conical depression


1
Surface area of the bucket V1 = π r 2 h
3
= π (r1 + r2) l + πr12
1 22 2
= 3.14 × (10 + 20) × 15.62 + 3.14 × 100 = × × (0.5 ) × 1.4
3 7
= 1785.40 cm2 22 × 0.25 × 0.2
= = 0.37
= 17.85 dm2 3

264 M A T H E M A T I C S – X
∴ Volume of 4 conical depressions WORKSHEET – 96
V2 = 4 × V1
1. (A) ∆ABC ~ ∆ADE
⇒ V2 = 1.48 cm3
r2 2h
Volume of wood in stand = ⇒ r2 = 2r1
r1 h
= 15 × 10 × 3.5 – 1.48
Volume of upper part
= 523.52 cm3.
1
6. Let n cones are required. V1 = π r12h
3
Volume of ice cream in n cones Volume of lower part
= Volume of ice cream in 4 cylinders
1 7 2
V2 = πh (r12 + 4r12 + 2r12) = πr1 h
1 2 3 3 3
7 2 7 
⇒ n ×  π ×   × 12 + π ×    ∴ V1 : V2 = 1 : 7.
 3 2 3  2  

21
2 2. (C) Let required height = H.
= 4 × π ×   × 38
 2 1 2 1
πr2H + πr h = 3 × πr 2 h
2 3 3
1 7  7
⇒ n× × π ×   × 2 6 +  1 2h
3 2
   2 ⇒ H+ h=h⇒H= .
3 3
21 × 21
=4×π× × 38 3. Volume of whole solid
4 = Volume of cone + Volume of cylinder
n × 49 × 19 + Volume of hemisphere.
⇒ = 21 × 21 × 38
12 1 πr2 H + πr 2 h + 2 πr3
=
⇒ n = 3 × 3 × 2 × 12 3 3
⇒ n = 216. 1 2
= π r [H + 3h + 2r]
3
7. 157.5 cm.
1 22 7 7
Hint: Volume of water flows out of pipe in = × × × × ( 2.8 + 19.5 + 7 )
1 hour (V1) = π (1)2 × 70 × 3600 cm3 3 7 2 2

Let level of water rise in tank = h 22 × 7


= × 29.3 = 376.016 cm3.
Volume of tank (V2) = π (40)2 ×h 3× 4
∴ V 1 = V2 4. False, because the volume of the ball
70 × 3600 a 4  a 3 4
⇒ h = having radius as = π  = π a3 .
40 × 40 2 3 2 24

= 157.5 cm. 5. Height of the cylinder = h = 20 cm.


Radius of the cylinder or of the hemisphere
4
8. 782 cm2
7 10.5
=r= = 5.25 cm
Hint: Area of sheet required = Surface area 2
of cylinder + Surface area of frustum
= 2π r1 h + π(r1 + r2) l;
where r1 = radius of smaller base
and r2 = radius of larger base.

S U R F A C E A R E A S A N D V O L U M E S 265
Total surface area ∴ Time taken to fill the tank
= Curved surface area of the cylinder 1100
+ 2 × curved surface area of
either hemisphere = 7 = 100 × 60 seconds
11
= 2πrh + 2 × 2πr2 = 2πr (h + 2r) 420
22
=2× × 5.25 (20 + 10.5) = 1 hr 40 min.
7
8. h = 16 cm, r1 = 8 cm, r2 = 20 cm
= 33 × 30.5 = 1006.5 cm2.
Capacity of container (frustum)
6. Let the radius and slant height of the
conical heap be r and l respectively.
Volume of sand filled in the bucket
= π × (18)2 × 32
Volume of sand in conical form
1
π × r 2 × 24
=
3 πh 2
As volume of sand will remain same.
=
3
(
r1 + r2 2 + r1r2 )
1 2 3.14 × 16
∴ πr × 24 = π × (18)2 × 32 = (82 + 202 + 8 × 20)
3 3
3 × 18 × 18 × 32 50.24
⇒ r2 = = 1296 = (64 + 400 + 160)
24 3
⇒ r = 36 cm 50.24
= × 624 = 50.24 × 208
2 3
Further, l= ( Radius )2 + ( Height )
10449.92
= 10449.92 cm3 = l
= 36 2 + 242 = 1872 1000
= 10.44992 l
= 12 13 cm Cost of milk
Thus, radius = 36 cm and slant height = Capacity in litres × Rate per litre
= 12 13 cm. = 10.44992 × 20
5 m/sce = ` 208.9984  ` 209.
7. Rate of water flowing =
6 If l be the slant height of the frustum, then
5
∴In 1 second water flows = m l= 2
h 2 + (r2 – r1 ) = 162 + 122
6
1 = 400 = 20 cm
Internal diameter = m
5
Total surface area of the frustum
∴ Volume of water flows out from pipe
= π (r1 + r2) l + πr12
in 1 second = π r2 h
= 3.14 [(8 + 20) × 20 + 82]
2
22  1  5 11 = 3.14 × 624 = 1959.36 cm2
= ×  × = m3 .
7  10  6 420 Cost of metal sheet used
Volume of water in tank = π r2 h 1959 . 36
= × 8 = 156.7488 ≈ ` 156.75
2 100
22  10  1100 3
= ×  ×2 = m Thus, cost of milk is ` 209 and cost of metal
7  2  7 sheet is ` 156.75.

266 M A T H E M A T I C S – X
WORKSHEET– 97 5. r = 14 m
h = height of cone
1. (A) Surface area of cone
= 13.5 – 3 = 10.5 m
= Surface area of hemisphere
H = height of cylinder
⇒ πr r 2 + h2 = 2πr2
=3m
⇒ r2 + h2 = 4r2
⇒ 3r2 = h2
r2 1
⇒ 2
=
h 3
⇒ r : h= 1 : 3.
3
4  4.2  = 66 × 42 × 21
2. (D) N × π× 
3  2  ∴ l= 14 2 + 10.5 2 = 196 + 110.25
66 × 42 × 21 × 3 × 7 =
⇒ N= = 1500. 306.25 = 17.5 m
4 × 22 × 2.1 × 2.1 × 2.1
Area to be painted = C.S.A. of cone + C.S.A.
3. Let H = 120 cm = height of cone of cylinder.
h = 180 cm = height of cylinder
= πr l + 2 π r H = πr [l + 2H]
r = 60 cm = radius of cone, cylinder
and hemisphere 22
= × 14 × 23.5 = 1034 m2
7
∴ Cost = 1034 × 2 = ` 2068.

6.

∴ Volume of water left in cylinder


= Volume of cylinder – Volume of cone 40
r = radius of cone = = 20 cm
– Volume of hemisphere 2

2 1 2 2 h = height of cone = 24 cm
= πr h − π r H − π r3
3 3 1 2
∴ Volume of cone (V1) = π r h cm3
3
2  H 2 
= π r h − − r Also, volume of water flows out of pipe in
 3 3 
1 min
22 V 2 = π R2 H
= × 60 × 60 [180 – 40 – 40]
7
2
5
=
22
× 60 × 60 × 100 cm3 = 1.131 m3. = π   × 1000 cm 3
7  20 
4 3 4 R Let conical vessel fills in 't' min.
4. False, because πR = 8 × πr 3 ⇒ r = .
3 3 2 ∴ V 1 = t × V2

S U R F A C E A R E A S A N D V O L U M E S 267
1 2 Volume of circular drum
π ( 20 ) × 24 3. Number of bags =
V1 3 Volume of each bag
⇒ t= = 2
V2  5 
π   × 1000 3.14 ×(4.2)2 × 3.5
 20  = ≈ 92.
2.1
400 × 8 × 400
= 51.2 min. 4. L = 15 cm, B = 10 cm and H = 5 cm
25 × 1000
= 51 min 12 sec. Volume of block = L × B × H
= 15 × 10 × 5
7. r1 = radius of small cylinder = 8 cm
= 750 cm3.
h1 = height of small cylinder = 60 cm
The hole is only possible throughout the
r2 = radius of big cylinder = 12 cm surface having area 15 cm × 10 cm.
h2 = height of big cylinder = 220 cm Volume of circular hole
2
Volume of pole 22  7 
= π r2 h= ×  × 5
= Volume of small cylinder 7 2
+ Volume of big cylinder. 22 × 7
= ×5
4
= π r 1 2 h1 + π r 2 2 h2
154 × 5
=
= 3.14 × [64 × 60 + 144 4
× 200] = 192.5 cm3
= 3.14 × [3840 + 31680] ∴ Volume of remaining solid
= 111532.8 cm3 = 750 − 192.5
= 557.5 cm3
∴ Mass = 111532.8 × 8 g
5. 6.521 kg (approx.)
= 892.26 kg.
Hint: Volume of metal in pipe = πh (R2 – r2)
8. 340π m2 where, R = outer radius and r = inner radius.
Also, mass = volume × density.
Hint:
6. Canal is in the form of cuboid.
So, width = 30 dm = 3 m
height = 12 dm = 1.2 m
length = distance travelled by water
in 30 min.
= 5 km = 5000 m
∴ Volume of water in canal = Volume of
cuboid
= 3 × 1.2 × 5000
Quantity of canvas required = 18000 m3
= C.S.A. of cone + C.S.A. of frustum. Let the required area for irrigation be
x square metre. Then
WORKSHEET – 98 8
x× = 18000
100
1. (A)
{... Area × height = volume}
2. (C)
⇒ x = 225000 m2.

268 M A T H E M A T I C S – X
7. Let BO = r Similarly, ST = 2r
∴ OD = r 1 2
Now, VAPR = πr h ...(i)
3
AC = 152 + 20 2
πh 2
VPSUR = [r + (2r)2 + r × 2r]
= 225 + 400 3
= 625 = 25 cm. 7 2
= πr h ...(ii)
1 3
Area of ∆ABC = × BC × AB
2 πh
VSBCU = [(2r)2 + (3r)2 + 2r × 3r]
1 3
= × 15 × 20 = 150 cm2 ...(i)
2 19 2
= πr h ...(iii)
Again area of ∆ABC 3
1 1 Using equations (i), (ii) and (iii), we get
= × AC × BO = × 25 × r
2 2 VAPR : VPSUR : VSBCU := 1 : 7 : 19
...(ii) Hence proved.
From (i) and (ii), we obtain OR
r = 12 cm.
1.05 l; 1961.14 cm2
From ∆AOB,
Hint: Capacity = Volume
AO = 202 − 122 = 16 cm. πh 2
Now, volume of double cone
=
3
(
r1 + r22 + r1 r2 )
= Volume of cone BADOB + Volume Surface area = π r12 + π ( r1 + r2 ) l , r1 < r2
of cone BCDOB
1 2 1 2
= π × (12 ) × 16 + π × (12 ) × 9 ASSESSMENT SHEET – 17
3 3
1 22 1. (C) π × 4x × 7x = 792
= × × 144 (16 + 9 ) = 3771.43 cm3
3 7 792 × 7
Surface area of double cone ⇒ x2 = ⇒ x=3
22 × 4 × 7
= π × 12 × 20 + π ×12 ×15
⇒ Radius = 4x = 12 cm.
22
= × 12 ( 20 + 15 ) = 1320 cm2. 2. (B) ∆AMN ~ ∆AOC
7
8. Let height and radius of the original cone h′ r
⇒ = 2
ABC be AO = 3h and BO = 3r respectively. h + h′ r1
As planes PQR and STU trisect the cone ⇒ h′r1 = hr2 + h′r2
ABC, ⇒ h′(r1 – r2) = hr2
AQ = QT = TO = h
hr2
In ∆ABO, PQ & BO, ⇒ h′ =
r′ – r2
PQ AQ
∴ = hr1
BO AO ⇒ h′ + h = .
r1 – r2
PQ h
⇒ = 3. h = 12 m
3r 3h
⇒ PQ = r π r12 = 9 and π r22 = 4

S U R F A C E A R E A S A N D V O L U M E S 269
9 4 2
7
⇒ r12 = π and r22 = i.e., π × 1.5 (7 + x) x = π ×   × 22.5
π  2
πh
Volume = × ( r12 + r22 + rl r2) 15 × 7 × 7
3 ⇒ 7x + x2 =
2×2
π × 12  9 4 9 4 ⇒ 2
4x + 28x – 735 = 0
=  + + × 
3 π π π π This is a quadratic equation in x. Now, we
1 have to solve it.
= 4π × (9 + 4 + 6) = 76 m3 . 2
π ∴ (2x + 7 ) = 735 + 49 = 282
a ⇒ 2x + 7 = ± 28
4. False, volume of the ball with radius
2 ⇒ x = 10.5, – 17.5
4  a
3
1 Since the width cannot be negative.
= π   = πa3 . Therefore, x = 10.5
3 2 6
Hence, width of the embankment is 10.5 m.
5. Edge of the cube = l
Radius of the hemisphere 14
7. Radius of the pipe = r = = 7 cm
l 2
=r= Rate of flowing the water = 15 km/hr
2
= 1500000 cm/hr
After cutting out the
Volume of water flowing per hour from the
hemispherical depres-
pipe = πr2h
sion, the surface area of
the remaining solid 22
= × 7 2 × 1500000
= Surface area of the cube + Inner 7
curved surface area of hemisphere = 22 × 7 × 1500000 cm3
– Base area of the hemisphere Volume of water required in the tanks
= 6l2 + 2πr2 – πr2 = 6l2 + πr2 = Length × Width × Height
l
2
1 = 50 m × 44 m × 21 cm
= 6l2 + π   = l 2 (24 + π) sq. units. = 5000 × 4400 × 21 cm3
2 4
Required time
6. Let the width of the embankment be x. Volume of water required
Let radius of well (r) =
Volume of water flowing per hour
7
= m 5000 × 4400 × 21
2 = = 2 hours.
22 × 7 × 1500000
Volume of the earth
dug out = πr2h 8. In the adjoining figure,
2 in ∆ABE and ∆ACD,
= π ×  7  × 22.5 BE CD, CD ⊥ AC and
 2
BE ⊥ AC
Volume of the embankment
⇒ ∆ABE ~ ∆ACD
= π (r + x)2H – πr2H
= πH (r + x + r) (r + x – r) AB BE
⇒ =
= π × 1.5 × (7 + x) x AC CD
Since, the embankment is formed by using h r R
⇒ = ⇒ r= ⇒ r = 5 cm ... (i)
the earth dug out. Therefore, their volumes 2h R 2
should be equal. (∵ R = 10 cm as given)

270 M A T H E M A T I C S – X
1 2 5. Let the edge of the cube be a and the radius
V1 = Volume of upper part = πr h of the sphere be r.
3
1 Surface area of the sphere
= π × 25 h ...(ii)
3 = Surface area of the cube
[Using (i)]
6 a2 3
V2 = Volume of lower part ⇒ 4πr2 = 6a2 ⇒ r2 = ⇒ r= a
4π 2π
1
= πh (r 2 + R 2 + rR) 4 3
3 Volume of the sphere (V1) = πr
3
1 3
= πh ( 25 + 100 + 50 ) 4
3  3 2
= πa3  
1 3  2π 
= πh × 175 ... (iii) Volume of the cube (V2) = a 3
3
1 V1
π × 25h ∴ Ratio in their volumes
V2
∴ Required ratio V1 = 3 3
V2 1
πh × 175 4  3 2
3 = πa3   : a3
3  2π 
[Using(ii) and (iii)]
–3 –3
1 22 × 2 2 × π × π 2
= = a3 : a3
7 –3
Hence, the ratio of the volumes of the two 3×3 2
parts is 1 : 7.
1

ASSESSMENT SHEET – 18 π 6
= a3 : a3 = : 1.
1 π
1. (B) C.S.A. = Inner C.S.A. + Outer C.S.A.
3
2
( 2
= 2π r12 + 2π r22 = 2π r1 + r2 . )
π
3 ∴ V1 : V2 = 1 : .
4 6 1 2 6
2. (A) n × π ×   = π × (12 ) × 24
3  2 3
5
12 × 12 × 8 × 3 6. h = 7 cm; r = mm = 0.25 cm
⇒ n= 2
4×3×3×3
Volume of the barrel = πr2h
n = 32.
22
= × 0.25 × 0.25 × 7 = 1.375 cm3
3. πr2h = 448 π ⇒ r2 × 7 = 448 7
⇒ r = 64 ⇒ r = 8 cm Volume of ink in the bottle
22 1 1
Curved surface area = 2πrh = 2 × ×8×7 = litre = × 1000 cm 3 = 200 cm3
7 5 5
= 352 cm2.
Number of barrels filled by the ink of bottle
4. False, sides become a, a and 2a and so surface
200 200000 1600
area will become = = =
1.375 1375 11
2(a × a + a × 2a + 2a × a) = 2(a2 + 2a2 + 2a2)
= 10a2. ∵ Number of words written by 1 barrel = 330

S U R F A C E A R E A S A N D V O L U M E S 271
∴ Number of words written by
1600
barrels 8 h12 + 8h1 h2 – 9h1 h2
⇒ =1
11 h2 ( h1 + h2 )
1600
= 330 × = 48000. ⇒ 8 h12 – h1 h2 = h1 h2 + h22
11
7. The given hollow ⇒ 8 h12 – h22 = 2h1h2
cone of base radius r2
h1 h2
is cut at a distance h1 ⇒ 8 – =2
from the vertex of it. h2 h1
The upper part is also (Dividing throughout by h1h2)
a cone of height h1 , h1
base radius r1 and the ⇒ Put = x to get
h2
slant height l 1 . The
remaining part is a frustum of height h2, base 1
⇒ 8x – =2
radii r1, r2 and slant height l2. Thus, the height x
and slant height of the whole cone will be h1 ⇒ 8x2 – 2x – 1 = 0
+ h2 and l1 + l2 respectively (see figure).
⇒ (2x – 1) (4x + 1) = 0
∆ABC ~ ∆ADE (AA criterion of similarity)
1 1
⇒ x=or x = –
AD DE AE 2 4
⇒ = =
AB BC AC Since ratio of heights can't be negative,
h1 + h2 r l +l 1
⇒ = 2= 1 2 ... (i) so x ≠ –
h1 r1 l1 4
1 h 1
h l ∴ x= ⇒ 1 = ⇒ h1 : h2 = 1 : 2
⇒ 1+ 2 = 1 + 2 2 h2 2
h1 l1
Hence the plane divides the altitude of the
h1 l cone in the ratio 1 : 2.
⇒ = 1 ... (ii)
h2 l2 21
8. Radius of cylindrical vessel = R = cm and
2
8 height of it = H = 38 cm
Curved surface of the frustum = × curved
9 Volume of each cylindrical vessel = πR2H
surface of the whole cone Volume of ice cream in 4 vessels = 4πR2H
8 7
⇒ π (r1 + r2) l2 = πr (l + l ) Radius of cone = r = cm
9 2 1 2 2
⇒ 9r1l2 + 9r2l2 = 8r2l1 + 8r2l2 Height of cone = h = 12 cm
⇒ 8r2l1 – 9r1l2 = r2 l2 Volume of ice cream in the

8 l1 9 r1 1 2
⇒ – =1 each conical shape = πr h
l2 r2 3
(Dividing throughout by r2 l2) 7
Radius of hemisphere = r = cm
2
h1  h1 
⇒ 8 – 9  =1 Volume of ice cream in each hemispherical
h2  h1 + h2  2 3
[Using (i) and (ii)] shape = πr
3

272 M A T H E M A T I C S – X
∴ Volume of ice cream in one cone 32706.67
1 2 = 32706.67cm3 = l
= πr 2 h + πr 3 1000
3 3 = 32.7 l.
1 2
= πr ( h + 2r ) 4. True, because capacity
3
2
Number of required cones = πr 2 h – πr 3
3
Volume of ice cream in 4
cylindrical vessels πr 2
= = (3h – 2r ) .
Volume of ice cream in 1 cone 3
with hemispherical top 6
5. Radius of each cone = r = = 3 cm
4 πR 2 H 12R 2 H 2
= = Let the heights of the cone be h1 and h2
1 2
πr ( h + 2 r ) r 2 ( h + 2r ) respectively.
3 ∴ h1 + h2 = 21 cm ...(i)
21 21
×12 ×
× 38 12 × 3 × 3 × 38 V1 2
2 2 Given: =
= = V2 1
7 7 19
× × (12 + 7 )
2 2 1 2
π r h1
= 216. 3 2
∴ = ⇒ h1 = 2h2
1 2 1
πr h2
3
CHAPTER TEST Substitute h1 = 2h2 in equation (i) to get
h2 = 7 cm ∴ h1 = 14 cm
4 3
πr1 r1
V1 64 43 4 1 1 22
⇒ 3
2
1. (D) = = 3 ⇒ = Now, V1= πr 2 h1 = × × 3 × 14 = 132 cm3
V2 27 4 3 r 3 3 3 7
πr2 3 2
3
1 2 1 22
and V2 = πr h2 = × × 32 ×7 = 66 cm3
2 2 3 3 7
S1 4πr12 r  4 16
= 2
=  1  =   = Volume of remaining portion
S2 4πr2 r
 2  3  9
= Volume of the cylinder – (V1 + V2)
∴ S1 : S2 = 16 : 9. = πr2 × 21 – (132 + 66)
4 22 3 22 2
2. (A) 49 × 33 × 24 = × ×r = × 3 × 21 – (132 + 66 )
3 7 7
⇒ r= = 594 – 198 = 396 cm3.
3
9261 ⇒ r = 21 cm.
6. Let the edge of the new formed cube be a.
44 Note that volume of a cube = (Edge)3
3. r1 = = 22 cm,
2 Sum of the volumes of three cubes
24 = Volume of the new formed cube
r2 = = 12 cm, h = 35 cm ⇒ 33 + 43 + 53 = a3
2
⇒ a3 = 27 + 64 + 125
πh 2
Capacity = (r + r22 + r1r2 ) 1
3 1 ⇒ a = ( 216 ) 3
22 35 ⇒ a = 6 cm
= × × ( 484 +144 + 264 )
7 3 Hence, the required edge is 6 cm.

S U R F A C E A R E A S A N D V O L U M E S 273
7. See Worksheet – 96, Sol. 6. 2 3 1408
8. Let height of the building be h. ⇒ πr2h – πr3 + πr =
3 21
∴ Radius of dome or cylinder
1 2 1408
2 1 1 ⇒ πr (3h – r) =
=r= h× = h 3 21
3 2 3
1 22 1 2  1  1408
⇒ × × h  3h – h  = 21
3 7 9  3 
1 3
Capacity of the building = 67 m 1408 × 21 × 9 × 3
21 ⇒ h3 = ⇒h= 3
216
22 × 8 × 21
2 3 1408 ⇒ h=6m
⇒ πr2 (h – r) + πr =
3 21
Hence, height of the building is 6 m.
❑❑

274 M A T H E M A T I C S – X
PRACTICE PAPERS
Practice Paper–1 4 6
3
4
2
8. (C) N × × π ×   = π ×   × 45
3  2 2
SECTION-A
⇒ N × 36 × π = π × 4 × 45
1. (C) D = b2 – 4ac = (– 4)2 –4 ( )(
2 – 2 ) ⇒ N=
180
= 5.
= 16 + 8 = 24. 36

1 9. (A) In a single throw of die, the even


2. (B) a = – 5, d = number may be 2, 4 or 6.
2
∴ an = a + (n – 1) d 3 1
∴ P(getting an even number) = = .
1
6 2
n 1
⇒ an = – 5 + (n – 1) =–5+ –
2 2 2 10. (D) Let the required angle be θ.
– 11 n We know that angle between tangents
1
= + = ( n – 11) . and the angle between corresponding
2 2 2 radii are supplementary
3. (C) M is the nearest point from P because ∴ θ + 35° = 180°
⇒ θ = 145°.
PMO is a straight line.
PO2 = OQ2 + PQ2 SECTION-B
(∵ ∠Q = 90°)
1 2
⇒ (PM + 6)2 = 62 + 82 11. V1 = Volume of cone = πr h
3
⇒ PM + 6 = 10 1 2
⇒ PM = 4 cm. = × π × ( 2.1) × 8.4
3
AB 2 1 4 3
4. (C) ∵ tan P = = = = tan 30° V2 = Volume of sphere = πR
BP 2 3 3 3
∴ P = 30°. As V 1 = V2
5. (D)As we know AB + DC= AD + BC 4 3 1 2
∴ πR = π × ( 2.1) ×8.4
⇒ 18 + x = 16 + 10 3 3
⇒ 18 + x = 26 ⇒ R3 = (2.1)3
⇒ x= 8 cm. ⇒ R= 2.1 cm.
πrθ 12. As ∠APB = 80°
6. (C) ∵ Arc length =
180° ⇒ ∠APO = 40°
π (6) θ 3π × 180° (... OP bisect ∠APB)
⇒ 3π = ⇒ =θ
180° π×6 Also ∠PAO = 90°
∴ θ = 90°. ∴ In ∆OAP,
7. (D) ∵ ∠OPQ = ∠OPT – ∠QPT ∠A + ∠P + ∠O = 180°
= 90° – 60° ⇒ ∠O = 50°
= 30°. ⇒ ∠POA = 50°.

P R A C T I C E P A P E R S 275
13. We have kx2 – 5x + k = 0 18. Let OAB be the given sector.
∴ Comparing it with ax2 + bx + c = 0, Then perimeter of OAB = 22 cm
we get a = k, b = – 5 and c = k. ⇒ OA + OB + l = 22 cm
For real and equal roots, ⇒ 6 + 6 + l = 22 cm
b2 – 4ac = 0 ⇒ 25 – 4k2 = 0
⇒ l = 10 cm
5
⇒ 4k2 = 25 ⇒ k = ± . 1
2 ∴ Area of sector = lr
2
14. Let the common difference of the two
1
A.P.’s be d. Then, their nth terms are: = × 10 × 6 = 30 cm2.
an = 3 + (n – 1).d
2
and a′n = 8 + (n – 1).d OR
∴ an – a′n = 3 + (n – 1).d – 8 – (n – 1).d CD 12
ND = = = 6 cm
= – 5 for all n∈N 2 2
∴ a2 – a′2 = – 5. AB 16
MB = = = 8 cm
15. The coordinates of centroid are: 2 2

 x1 + x2 + x3 y1 + y2 + y3  In ∆ODN, ON = OD2 – ND 2
 , 
 3 3 
∴ ON = 102 – 6 2 = 64 = 8 cm
 0 + 8 + 6 8 + 12 + 0  14 20 
=  ,  =  , .
 3 3   3 3 In ∆ OBM, OM = OB2 – MB2
16. Since A(x, y), B(1, 2), C(7, 0) are collinear. ∴ OM = 102 – 82 = 36 = 6 cm
So ar(∆ABC) = 0
1 ∴ Required distance,
⇒ x(2 – 0) + 1(0 – y) + 7(y – 2) = 0
2 i.e., MN = ON – OM = 8 – 6 = 2 cm.
⇒ 2x – y + 7y – 14 = 0
⇒ x + 3y – 7 = 0. SECTION-C
17. Total number of cards = 1000
15 AB
The cards each bearing a perfect square 19. tan θ = =
number greater than 500 are: 529, 576, 8 BC
625, 676, 729, 784, 841, 900 and 961. ⇒ AB = 15k and BC = 8k
∴ Number of such cards = 9
(i) P(the first player wins a prize) ⇒ AC = 225k 2 + 64k 2
9
= = 17k.
1000
∴ 17k = 90
(ii) After winning the first player:
Number of total cards = 999 90
⇒ k=
and number of card each bearing a 17
perfect square number greater than
15 × 90
500 = 8 ∴ AB = 15k = = 79.41 m.
∴ P(the second player wins a prize, if 17
8 ⇒ Height of kite above the ground is
the first has won) = . 79.41 m.
999

276 M A T H E M A T I C S – X
20. Given equation is 4x2 + 4bx – (a2 – b2) = 0
mn  2 (mn – 1) 
Comparing with Ax2 + Bx + C = 0, we get =  + 
2  mn mn 
A = 4, B = 4b, C = – (a2 – b2)
∴ D = B2 – 4AC 1
Smn = (mn + 1) .
⇒ D = 16b2 + 16(a2 – b2) 2
= 16a2 > 0 Hence proved.
∴ Two distinct real roots are given by 22. We have

( at2 – a)
2 2
–B± D – 4b ± 16a2 SP = + ( 2at – 0 )
x= =
2A 8

(t2 – 1)
2
– 4b ± 4a = a + 4t 2
=
8
a–b a+b = a t 4 + 1 – 2t 2 + 4t 2
∴ x= or – .
2 2
21. Let first term of an A.P. = a = a t 4 + 2t 2 + 1
and common difference = d
(t2 + 1) ( )
2 2
1 = a = a t +1
∴ am =
n
1
⇒ a + (m – 1)d = ...(i)  a   2a 
2 2
n
and SQ =  2 – a +  – 0
1 t   t 
and an =
m
( )
2
1 a2 1 – t 2 4 a2
⇒ a + (n – 1)d = ...(ii) = +
m
t4 t2
Subtract equation (ii) from (i),
1 1 a
(1 – t 2 )
2
(m – n)d = – ⇒ SQ = + 4t2
n m t2
m–n a a
⇒ (m – n)d =
(1 + t2 )
2
mn = = (1 + t2)
t2 t2
1
⇒ d= 1 1 1 t2
mn ∴ + = +
Using d in (i), we get
SP SQ (
a t2 + 1 a t2 + 1 ) ( )
1 1
a + (m –1) = 1 + t2 1
mn n = = ;
1 1 1
( 2
a t +1 ) a
⇒ a+ – =
n mn n which is independent of t.
1 Hence proved.
⇒ a=
mn 23. As we know that opposite sides of the
circumscribing quadrilateral sub-tend
mn supplementary angles at the centre of the
∴ Smn = {2a + (mn – 1)d}
2 circle.

P R A C T I C E P A P E R S 277
OR

∴ ∠DOC + ∠AOB = 180°


⇒ 95° + ∠AOB = 180°
⇒ ∠AOB = 180° – 95° = 85°
Similarity, ∠AOD + ∠BOC = 180°
⇒ ∠AOD + 92° = 180°
⇒ ∠AOD = 180° – 92° = 88°.
24. Let A ≡ (3, – 4) and B ≡ (1, 2).

As AP = PQ = QB.
So AP : PB = 1 : 2
∴ Using section formula,
1.(1) + 2(3) 7
p= =
1+ 2 3
again Q is mid-point of PB.
AP and AQ are required tangents.
∴ Using mid-point formula,
–2+2 26. OA = 7 cm = R (say)
q= = 0.
2 ∴ Area of outer circle
7 = πR2 = π (7)2
Thus, p = ; q = 0.
3
22
25. = × 49 = 154 cm2
7
Also OD = OA = 7 cm
7
∴ r= cm
2
∴ Area of smaller circle
= πr2
22 7 7 77
= × × =
7 2 2 2
= 38.5 cm2.
∴ Required area = 154 – 38.5
= 115.5 cm2.
OR
The sectors made by drawing arcs are
∴ A’BC’ is required triangle. equal in area.

278 M A T H E M A T I C S – X
Sum of areas of the three sectors 28. Hint: Total events = 36
60° Even sum: 2; (1, 1)
= 3× × π × 52 .
360° 4; (2, 2), (1, 3), (3, 1)
⇒ Area of shaded region 6; (1, 5), (5, 1), (2, 4), (4, 2), (3, 3)
1 8; (2, 6), (6, 2), (3, 5), (5, 3), (4, 4)
= 3 × × π × 25
6 10; (4, 6), (6, 4), (5, 5)
25 12; (6, 6)
= π cm2 ∴Number of favourable events = 18
2
Area of equilateral triangle ABC 18 1
∴ Required probability = = .
36 2
3 OR
= × 102 = 25 3 cm 2
4
Total number of balls = 10 + 5 + 7 = 22
Now, area of unshaded region
(i) Probability of drawing a red ball
= Area of ∆ABC – Area of shaded
region Number of red balls 10 5
= = =
Total number of balls 22 11
25
= 25 3 – π
(ii) Probability of drawing a green ball
2
25 Number of green balls 7
= 25 × 1.732 – × 3.14 = =
2 Total number of balls 22
= 43.3 – 39.25
(iii) Probability of drawing a blue ball
= 4.05 cm2.
Number of blue balls 5
27. Since each design = =
Total number of balls 22
occupy an area equal to
∴ Probability of drawing not a blue ball
a segment of a circle of
radius 28 cm and having = 1 – Probability of
central angle 60°. drawing a blue ball
5 17
∴ Area of 6 designs =1– = .
22 22
θ θ θ
= 6 ×  × πr 2 – r 2 sin cos  cm 2 SECTION-D
 360° 2 2
60° 22 29. Height of the cylinder h = 2.4 cm
= 6 ×  × × 28 × 28 – 28 × 28
 360° 7 1.4
and radius r = = 0.7 cm
 2
× sin 30°. cos 30°  cm2 Also height of the conical cavity

= 2.4 cm
1 22 1 3 and radius = 0.7 cm
= 6 × 28 × 28  × – × 
 6 7 2 2  ∴ Slant height of the cavity l
= 2464 – 1999.2 = 464.8 cm2 = l 2 + h2
∴ Cost of making the designs
= (0.7)2 + (2.4)2
= 464.8 × 3.50
= ` 1626.80. = 6.25 = 2.5 cm.

P R A C T I C E P A P E R S 279
Now, T.S.A. of remai- OR
ning solid = C.S.A. of Let Nisha’s present age be x years, so at
cylinder + C.S.A. of present Asha’s age be (x2 + 2) years when
cone + Area of base Nisha grows to her mother’s present age,
of cylinder. i.e., (x2 + 2) years then Asha’s age would
⇒ 2πrh + πrl + πr2 be (10x – 1) years.
⇒ πr (2h + l + r) That means Nisha grows (x2 + 2x – x) year
22 and hence Asha also grows the same
= × 0.7
7 years. That means Asha’s age would be
(4.8 + 2.5 + 0.7) {(x2 + 2) + (x2 + 2 – x)} years.
= 17.6 cm2 ≅ 18 cm2. Now equating Asha’s both of above
mentioned ages, we get:
30. Let V = Volume of pool
(x2 + 2) + (x2 + 2 – x) = 10x – 1
Let x = Number of hours required by ⇒ 2x2 – x + 4 = 10x – 1
second pipe to fill the pool ⇒ 2
2x – 11x + 5 = 0
∴ x + 5 = Number of hours taken by first ⇒ 2x2 – 10x – x + 5 = 0
pipe to fill the pool ⇒ 2x(x – 5) – 1 (x – 5) = 0
x – 4 = Number of hours taken by third 1
pipe to fill the pool. ⇒ (x – 5) (2x – 1) = 0 ∴ x = 5,
2
The parts of pool 1
But x = does not satisfy all the given
filling in one hour 2
by first, second and conditions.
third pipe would be Therefore, Nisha’s present age = 5 years
V V V and Asha’s present age = x2 + 2 = 27 years.
, and
x+5 x x –4 31. Join OA, OC, OB;
respectively. and also join OE,
∴ According to question, OF; E and F being
point of contacts.
V V V Now, radius of
+ =
x x+5 x–4 circle
1 1 1 = OD = OE
⇒ + = = OF = 4 cm
x x+5 x–4
and BC = CD + DB = 14 cm
⇒ (2x + 5)(x – 4) = x2 + 5x Also CD = CF = 6 cm
⇒ x2 – 8x – 20 = 0 and BD = BE = 8 cm.
⇒ x2 – 10x + 2x – 20 = 0 Let AF = AE = x
⇒ (x – 10)(x + 2) = 0 ⇒ AC = 6x; AB = 8 + x
⇒ x = 10 or x = – 2 ∴ a =14, b = 6 + x, c = 8 + x
∴ x = 10. Now, using Heron’s formula
(Value with negative sign is rejected) ar(∆ ABC) = s( s – a)( s – b)( s – c )
Hence times required by first, second and
third pipe to fill the pool individually are a + b + c 14 + 6 + x + 8 + x
where, s= =
15 hours, 10 hours and 6 hours 2 2
respectively. = 14 + x

280 M A T H E M A T I C S – X
Let AE = x so PA = x.
∴ar(∆ ABC) = (14 + x) × x × 8 × 6
⇒ TA = PT – PA = 12 – x
= 48 x(14 + x) ... (i) As OE ⊥ AB, in right-angled ∆AET,
Also, AE2 + ET2 = AT2
ar(∆ABC) = ar(∆OBC) + ar(∆OAB)
⇒ x2 + (8)2 = (12 – x)2
+ ar(∆OAC)
⇒ x2 + 64 = 144 – 24x + x2
1 1
= × BC × OD + AB × OE 80 10
2 2 ⇒ 24x = 80 ∴x= = cm
24 3
1
+ × AC × OF 10
2 Similarly, BE = cm
1 3
= × 4 × (BC + AB + AC) 20
2 Therefore, AB = AE + BE = cm.
= 2(14 + 6 + x + 8 + x) 3
32. r1 = Radius of top
= 56 + 4x ... (ii)
= 28 cm
∴ From (i) and (ii),
r2 = Radius of bottom
48x (14 + x) = 56 + 4x = 7 cm

4 3x (14 + x) = 56 + 4x V = Capacity
= 21560 cm3
3x (14 + x) = 14 + x
h = Height of bucket
Squaring both sides, we get
⇒ 3x (14 + x) = (14 + x)2 1
∴ V= π h(r 12 + r 22 + r1r2 )
⇒ 3x = 14 + x 3
⇒ 2x = 14 1 22
⇒ 21560 = × h (784 + 49 + 196)
⇒ x= 7 3 7
∴ AB = 8 + x = 15 cm ⇒ 20580 = h × 1029
And AC = 6 + x = 13 cm. ⇒ h = 20 cm.
OR Now, whole surface area (S)
We have, radius (OP) = 5 cm = OE and = πr22 + π(r1 + r2)l
OT = 13 cm.
OP ⊥ PT so in right-angled ∆OPT, Here, l = h 2 + (r1 – r2 )2

= 400 + 441 = 841 = 29 cm

22 22
∴ S = × (7)2 + (28 + 7) × 29
7 7
22
= × (49 + 35 × 29)
7
OP2 + PT2 = OT2 (Pythagoras Theorem) = 3344 cm2.
⇒ 52 + PT2 = 132 33. Cash prizes are:
⇒ PT = 169 – 25 = 12 cm. ` 320, ` 280, ` 240, ` 200, ` 160, ` 120, ` 80.

P R A C T I C E P A P E R S 281
Hint: S7 = 1400; d = – 40; n = 7 So, the height of the multi-storeyed

∴ 1400 =
7
2
[2a – 6 × 40] building = 4 {( ) } (
3 +1 +8 m=4 3+ 3 m )
n and the distance between the two buildings
(∵ Sn = {2a + (n – 1)d})
2 (
is also 4 3 + 3 m. )
⇒ 200 = a – 120
⇒ a = 320. Practice Paper –2
34. In Figure, PC SECTION-A
denotes the multi-
storeyed building 1. (A) D > 0
and AB denotes ⇒ 62 – 4 × 1 × k > 0
the 8 m tall ⇒ k < 9.
building. We are
interested to deter- 2. (D) 5th term from the end
mine the height of the multi-storeyed = 201 + (5 – 1) × (– 2)
building, i.e., PC and the distance between = 201 – 8
the two buildings, i.e., AC. = 193.
Look at the figure carefully. Observe that 3. (B) 7
PB is a transversal to the parallel lines PQ Hint: To draw a triangle similar to a given
and BD. Therefore, ∠QPB and ∠PBD are
p
alternate angles, and so are equal. So triangle with a scalar factor , p > 0, q > 0,
q
∠PBD = 30°. Similarly, ∠PAC = 45°.
In right-angled ∆PBD, we have we should locate the number of points
which is greater of p and q.
PD 1
= tan 30° = 4. (C) Sum of opposite sides of a
BD 3
quadrilateral having a circle inscribed it
or BD = PD 3 are equal.
In right-angled ∆PAC, we have
PC
= tan 45° = 1
AC
i.e., P C = AC
Also, P C = PD + DC.
Therefore, PD + DC = AC.
∴ PQ + RS = PS + QR.
Since, AC = BD and DC = AB = 8 m,
we get 5. (B) Join AC and PC.
PC bisects ∠APB
PD + 8 = BD = PD 3
⇒ ∠APC = 45°
This gives
⇒ ∠ACP = 45°

PD =
8
=
8 ( 3 +1 ) In ∆APC,
3 –1 ( )( 3 – 1)
3 +1 ∠APC = ∠ACP
⇒ AC = AP = 4 cm
= 4 ( 3 + 1) m. ⇒ Length of each tangent = 4 cm.

282 M A T H E M A T I C S – X
4 SECTION-B
6. (A) π R3 = 49 × 33 × 24
3 8 4
11. r1 = = 4 m , r2 = = 2 m; h = 6 m
4 22 2 2
⇒ × × R 3 = 49 × 33 × 24 Volume of frustum
3 7
1
49 × 33 × 24 × 3 × 7 = πh(r12 + r22 + r1 r2 )
⇒ R3 = 3
4 × 22
1 22
= 49 × 7 × 3 = 73 × 33 = (21)3 = × × 6(16 + 4 + 8)
3 7
∴ R = 21 units.
44
7. (B) Average = × 28 = 176 m3.
7
3+ 5+5+7 +7 +7 +9+ 9+ 9+ 9 Thus, the capacity of the reservoir is 176 m3
= =7 or 176 kl.
10
3 12. Let E be the event of getting the sum as a
P(selected number is 7) = . perfect square.
10
∴ E = {(1, 3), (2, 2), (3, 1), (3, 6), (4, 5),
8. (B) In quadrilateral POQT, (5, 4), (6, 3)}
∠P + ∠O + ∠Q + ∠T = 360° ∴ n(E) = 7,
n(E) 7
Now, P(E) = = . [∵ n(S) = 36]
n(S) 36
13. Since –4 is a root of x2 + px – 4 = 0
⇒ (– 4)2 + p(– 4) – 4 = 0
⇒ 16 – 4p – 4 = 0
⇒ 90° + 110° + 90° + ∠T= 360° ⇒ 4p = 12 ⇒ p = 3.
⇒ ∠P T Q = 70°. Also as the equation x2 + px + k = 0 has
equal roots.
4.2 ∴ D=0 ⇒ b2 – 4ac = 0
9. (D) r= = 2.1 cm;
2 ⇒ p2 – 4k = 0
h = 4.2 cm ⇒ 9 – 4k = 0
(∵ p = 3)
1 2
Volume= πr h 9
3 ⇒ k= .
4
1 22 OR
h = × × 2.1× 2.1 × 4.2
3 7 False, because consider the equation
2x2 + 3x – 1 = 0; its coefficients are all rational
= 19.404 cm3 ≈ 19.4 cm3.
–3 ± 9+8 – 3 ± 17
10. (A) In right ∆ABC, but its roots are x = =
2× 2 4
6 both of them are irrational.
sin 30° =
AC
14. The numbers are: 3, 9, 15, 21,.......,99.
1 6 ∴ It is an A.P.
⇒ =
2 AC ⇒ Let a = 3, d = 6
an = 99
⇒ AC = 12 m.
⇒ a + (n – 1)d = 99
⇒ 3 + (n – 1) × 6 = 99 ⇒ n = 17
P R A C T I C E P A P E R S 283
n We will use section formula:
∴ Required sum = Sn = (a + an)
2 m x + m2 x1
x= 1 2
17 m1 + m2
S17 = (3 + 99) = 867.
2 3 1
15. True. Here, x = , x = , x = 2, m1 = λ, m2 = 1
4 1 2 2
Let PQ be the tangent at A to the circum- 1
circle of isosceles ∆ABC. λ × 2 + 1×
3 2
Given: AB = AC. ∴ =
4 λ +1
∴ ∠ABC = ∠ACB
⇒ 8λ + 2 = 3λ + 3
1
⇒ 5λ = 1 ⇒ λ=
5
1
∴ λ : 1= : 1 = 1 : 5.
5
Thus, required ratio is 1 : 5.

Construction: Draw AD; perpendicular 18. Area of the triangle


bisector of BC as AB = AC. 1
= {x1(y2 – y3) + x2(y3 – y1) + x3(y1 – y2)}
∴ Perpendicular bisector of BC will pass 2
through A as well as centre of circle O. 1
= {9(7 + 4) + 4(– 4 – 2) + 7(2 – 7)}
∠1 + ∠2 = 90° and ∠3 + ∠4 = 90° 2
⇒ ∠2 = ∠3 1
= (99 – 24 – 35)
⇒ ∠CAQ = ∠ACB. 2
So ∠ACB = ∠QAC 1
= × 40 = 20 sq. units.
2
Therefore, PQ || BC
(∵ Alternate interior angle pair is equal). SECTION-C
16. Here, r1 = 6 cm, 19. Let the point on the y-axis be P(0, y) and
r2 = 12 cm on the x-axis be Q(x, 0).
For point P:
and h = 8 cm
∵ PA = PB
⇒ PA2 = PB2
⇒ (0 – 6) + (y – 5)2 = (0 + 4)2 + (y – 3)2
2

⇒ 36 + y2 – 10y + 25 = 16 + y2 – 6y + 9
Slant height (l) ⇒ 4y = 36 ⇒ y = 9.
For point Q:
= (r2 – r1 )2 + h 2 QA = QB
⇒ QA2 = QB2
= (12 – 6)2 + 8 2
⇒ (x – 6)2 + (0 – 5)2 = (x + 4)2 + (0 – 3)2
= 36 + 64 = 10 cm. ⇒ x2 – 12x + 36 + 25 = x2 + 8x + 16 + 9
⇒ 20x = 36
17. Let the required ratio be λ : 1. 9
⇒ x= .
5
9
∴ Required points are (0, 9) and  , 0  .
 5 
284 M A T H E M A T I C S – X
OR ⇒ 8x = 8 ⇒ x = 1
Let in ∆ABC; Also ∠ B = 90°.
D(1, 2), E(0, – 1) So, AB2 + BC2 = AC2
and F(2, – 1) be ⇒ (x + 1)2 + (y – 2)2 + (x – 3)2 + (y – 2)2
the mid-points = (4)2 + 02
of sides BC, AC Put x = 1,
and AB respec- ⇒ 4 + (y2 + 4 – 4y) + 4 + (y2 + 4 – 4y) = 16
tively. ⇒ 2(y2 + 4 – 4y) = 8
Let A(x1, y1), B(x2 , y2), C(x3, y3) be vertices ⇒ y2 + 4 – 4y = 4
of ∆ABC. ⇒ y2 – 4y = 0 ⇒ y(y – 4) = 0
⇒ y = 0 or y = 4
Now, using mid-point formula
∴ Coordinates of B and D may be taken as
x1 + x2
= 2 ⇒ x1 + x 2 = 4 ...(i) (1, 0) and (1, 4).
2
x2 + x3 21. a = 5, l = 45, Sn = 400
= 1 ⇒ x2 + x3 = 2 ...(ii) n n
2 Now, Sn = {a + l} ⇒ 400 = {5 + 45}
x3 + x1 2 2
= 0 ⇒ x3 + x1 = 0 ...(iii)
2 ⇒ 50n = 800 ⇒ n = 16
Similarly, Now an = 45
y1 + y2 = – 2 ...(iv) ⇒ a + (n – 1).d = 45
y2 + y3 = 4 ...(v) ⇒ 5 + (16 – 1).d = 45 ⇒ 15d = 40
y1 + y3 = – 2 ...(vi) 40 8
⇒ =
d=
Adding equations (i), (ii) and (iii), we get 15 3
x1 + x2 + x3 = 3 ...(vii) Hence, number of terms is 16 and common
And from equations (iv), (v) and (vi), we get 8
difference is .
y1 + y2 + y3 = 0 ...(viii) 3
By solving equation (vii) with equations (i), OR
(ii) and (iii) respectively pairwise, we have Consider:
x3 = –1, x1 = 1, x2 = 3 a – b 3 a – 2b 5 a – 3b ............
+ + + 11 terms
Similarly, we can find that a+b a+b a+b
y3 = 2, y1 = – 4, y2 = 2
 a b   3a 2b 
Thus, A = (1, – 4), B = (3, 2), C = (– 1, 2) =  – + – 
∴ Coordinates of the vertices are  a+b a + b   a+b a + b 
(1, – 4); (3, 2) and (– 1, 2).  5a 3b  .............
+ – +
20. Let ABCD be  a+b a + b 
the square in a  1 + 3 + 5 + .........11terms  b
which A(–1, 2) =   –
a+b  A.P.  a+b
and C(3, 2) are
given vertices.  1 + 2 + 3 + ......... + 11 terms 
Let coordinates  
 A.P. 
of B are (x, y).
a  11  b
As AB = BC ⇒ AB2 = BC2 =  {2 × 1 + (11 – 1). 2} –
a+b  2  a+b
⇒ (x + 1) + (y – 2)2 = (x – 3)2 + (y – 2)2
2

⇒ (x + 1)2 = (x – 3)2  (11 + 1) 


11 
⇒ x + 2x + 1 = x2 + 9 – 6x
2
 2 

P R A C T I C E P A P E R S 285
a b AE 20.5
= (121) – ( 66 ) So, tan θ = ⇒ tan θ = =1
a+b a+b CE 20.5
121a – 66b 11(11a – 6b ) ⇒ tan θ = tan 45° ∴ θ = 45°.
= = .
a+b a+b 23. Join AB.
22. Let AB be the tower of height 22 m and As OA = r; OP = diameter = 2r
CD be an observer of height 1.5 m. As AP is tangent,
In ∆ACE, CE = DB = 20.5 m, ⇒ OA ⊥ AP
AE = AB – BE (i.e., CD) ⇒ ∠OAP = 90°
= 22 – 1.5 = 20.5 m ∴ In right-angled ∆ OAP,
and ∠ACE = θ OA r 1
sin (∠1) = = =
OP 2r 2
⇒ ∠1 = 30°
Similarly, ∠2 = 30° ⇒ ∠APB = 60°
As AP = PB ⇒ ∠PAB = ∠PBA
Using Angle sum property in ∆ APB,
∠PAB = ∠PBA = 60° = ∠APB
⇒ ∆APB is equilateral. Hence proved.

24. The ratio between sides is 3 : 2


Let ∆ ABC; AB = AC and AM ⊥ BC;
AM = 4 cm
∴ A′BC′ is required triangle such that
∆ A′BC′ ~ ∆ ABC.

286 M A T H E M A T I C S – X
25. As AB = 28 cm;
⇒ 3x2 + 6 5 x – 5 x – 10 = 0
BC = 21 cm;
∠B = 90° ⇒ 3x(x + 2 5 ) – 5 (x + 2 5 ) = 0
⇒ AC2 = AB2 + BC2 ⇒ (x + 2 5 ) (3x – 5 ) = 0
⇒ AC2 = 282 + 212
Therefore, x + 2 5 = 0 or 3x – 5 =0
= 784 + 441
= 1225 5
∴ x = – 2 5 or 3x = 5 ⇒ x=
3
⇒ AC = 1225 = 35 cm
∴ Area of semicircle at AC as diameter 5
∴ The two real roots are – 2 5 and .
1  35 
2 3
A1 = π  27. The total surface area of the cube = 6 × (edge)2
2  2 
= 6 × 5 × 5 cm2 = 150 cm2.
1 22 35 × 35 Note that the part of the cube where the
= × ×
2 7 2×2 hemisphere is attached is not included in the
55 × 35 surface area.
= = 481.25 cm2
4 So, the surface area of the block
Area of ∆ABC = T.S.A. of cube – base area of hemisphere
+ C.S.A. of hemisphere
1
A2 = × BC × AB = 150 – πr2 + 2πr2 = (150 + πr2) cm2
2
1  22 4.2 4.2 
= × 21 × 28 = 294 cm2 = 150 cm 2 +  × ×  cm
2
2  7 2 2 
Area of quadrant PCB = (150 + 13.86) cm2 = 163.86 cm2.
1 OR
A3 = π (21)2
4
∆AOB ~ ∆AO’B’
1 22
= × × 21 × 21 AO' O'B'
4 7 ⇒ =
= 346.50 cm2. AO OB
∴ Required area 6 r
⇒ =
= Area of ∆ ABC + Area of 12 8
semicircle – Area of quadrant
⇒ r = 4 cm
= A2 + A1 – A3
= 294 + 481.25 – 346.50 Volume of the whole cone (V )
= 428.75 cm2. 1
= π × 82 × 12
3
26. The given equation is
= 256π cm3
3x2 + 5 5 x – 10 = 0
Volume of the upper part (small cone)
The discriminant = b2 – 4ac
1
= (5 5 )2 – 4 × 3 × (– 10) V1 = π × r2 × 6
3
= 125 + 120 = 245 > 0
1
So, the given equation has two distinct = π × 42 × 6
real roots. 3
= 32π cm3
Now, 3x2 + 5 5 x – 10 = 0

P R A C T I C E P A P E R S 287
Now, 30. Let unit’s digit = x
volume of upper part V1 and ten’s digit = y
= Then, original number = 10y + x
volume of lower part V – V1
Now, xy = 15 ... (i) (Given)
32π 1
= = . According to question,
256π – 32 π 7
(10y + x) + 18 = 10x + y
Hence, the required ratio is 1 : 7.
⇒ – 9x + 9y = – 18
28. Total number of coins = 100 + 50 + 20 + 10 ⇒ y–x=–2
= 180 ⇒ y=x–2
(i) Total number of 50 paise coins = 100 Using it in equation (i), we get
∴ Probability of getting a 50 paise coin x(x – 2) = 15
100 5 ⇒ 2
x – 2x – 15 = 0
= = .
180 9 ⇒ x2 – 5x + 3x – 15 = 0
(ii) Total number of 5 rupee coins = 10 x(x – 5) + 3(x – 5) = 0
∴ Total number of coins other than 5 rupee (x + 3)(x – 5) = 0
= 180 – 10 = 170 ⇒ x = – 3 or x = 5
∴ Probability of getting a coin other than As negative digits are not acceptable here,
170 17 . ∴ x=5
of 5 rupee = =
180 18 ∴ Unit’s digit = 5
SECTION-D And ten’s digit = 3
29. First instalment = ` 1000 Hence, number is 35.
Second instalment = ` 1100 31. Let C(O, r) be a circle
Third instalment = ` 1200 with centre at O and
......................................................... radius r. The circle
......................................................... touches the sides AB,
......................................................... BC, CD and DA of a
30th instalment = ? (to be calculated) quadrilateral ABCD
Here, 1000, 1100, 1200,........ forms an AP at the points P, Q, R
with a = 1000 and S respectively.
d = 100
To show:
n = 30
Using an = a + (n – 1)d, ∠AOB + ∠COD = 180°
a30 = a + 29d ∠AOD + ∠BOC = 180°
= 1000 + 29 × 100 Construction: Join OP, OQ, OR and OS.
= 1000 + 2900 Proof: Since two tangents drawn from an
external point to a circle subtend equal
= 3900
angles at the centre
∴ 30th instalment will be of ` 3900
n ∴ ∠1 = ∠2; ∠3 = ∠4;
Now, using Sn = (a + an)
2 ∠5 = ∠6; ∠7 = ∠8
30 Now∠1 + ∠2 + ∠3 + ∠4 + ∠5 + ∠6
Also S30 = (1000 + 3900) = 15 (4900)
2 + ∠7 + ∠8 = 360°
= ` 73500 ⇒ 2 (∠2 + ∠3 + ∠6 + ∠7) = 360°
∴ Total amount left to be paid is and 2 (∠1 + ∠8 + ∠4 + ∠5) = 360°
118000 – 73500 = ` 44500. ⇒ (∠2 + ∠3) + (∠6 + ∠7) = 180°

288 M A T H E M A T I C S – X
and (∠1 + ∠8) + (∠4 + ∠5) = 180°
22 1100 3
⇒ ∠AOB + ∠COD = 180° ⇒ V1 = × (5)2 × 2 = m
7 7
and ∠AOD + ∠BOC = 180°
Hence proved. Now let time required to fill the tank = t hours
OR ∴ Volume of water delivered by pipe in
We are given a circle t hours
with centre O, an
external point T and
two tangents TP and
TQ to the circle,
where P, Q are the points of contact (see
figure) . We need to prove that
∠PTQ = 2∠OPQ
Let ∠PTQ = θ
As we know that tangent segments
drawn from an external point to the circle V2 = t × π (0.1)2(3000)
are equal. So, TP = TQ
22
⇒ TPQ is an isosceles triangle. = 30 × ×t
7
1 660
Therefore, ∠TPQ = ∠TQP = (180° – θ) ⇒ V2 = t
2 7
1
= 90° – θ Consider: V1 = V2
2
Also, OP ⊥ PT, 1100 660
⇒ = t
i.e., ∠OPT = 90° 7 7
So, ∠OPQ = ∠OPT – ∠TPQ 1100 5
⇒ t = = h
660 3
 1 
= 90° –  90° – θ  5
 2  = × 60 min = 100 min
3
1 1 or 1 h 40 min.
= θ = ∠PTQ
2 2
33. 2πr1 = 18 ⇒ πr1 = 9
This gives ∠PTQ = 2 ∠OPQ
2πr2 = 6 ⇒ πr2 = 3
Hence proved.
∴ π(r1 + r2) = πr1 + πr2
32. Speed of water = 3 km/h = 9 + 3 = 12
Let r1 = radius of pipe (cylindrical) Curved surface area
= π(r1 + r2) l = 12 × 4 = 48 cm2.
20
= = 10 cm = 0.1 m. 34. Let AB = DE = h = Height of each pole
2
C = Position of point on street
10
And r2 = radius of tank = =5m BE = 30 m
2
∠ACB = 45°, ∠DCE = 30°
Also, H = height of tank = 2 m. AB
∴ In ∆ABC, tan 45° =
∴ Volume of cylindrical tank V1= πr22 H BC

P R A C T I C E P A P E R S 289
h Let C = Position of cloud
⇒ 1= AB = 60 m, ∠CAD = 30°
BC
F = Position of reflection of cloud in the
⇒ BC = h ... (i)
lake
∠ DAF = 60°
Let C E = h = Height of cloud above the lake
∴ C E= EF = h
Now in right-angled ∆CDA,
CD
tan 30° =
DE AD
In ∆CED, tan 30° =
EC 1 h − 60
⇒ =
1 h 3 AD
=
3 EC ⇒ AD = (h − 60) × 3 ... (i)
⇒ EC = h 3 ... (ii) Now in right-angled ∆FDA,
∴ On adding (i) and (ii), h + 60
tan 60° =
⇒ BC + CE = h + h 3 AD
h + 60
⇒ (
30 = h 1 + 3 ) ⇒ 3 =
AD
(... BC + CE = BE = 30 m) h + 60
⇒ AD = ... (ii)
30 1− 3 3
⇒ h= ×
1+ 3 1− 3 ∴ Equations (i) and (ii) give
h + 60
⇒ h=
30 ( ) = 30 × 0.73
3 −1 (h − 60) 3 =
3
2 2
⇒ 3h – 180 = h + 60
= 15 × 0.73
⇒ h = 10.95 m ⇒ 3h – h = 60 + 180
∴ BC = 10.95 m ⇒ 2h = 240
and EC = 30 – 10.95 = 19.05 m. ⇒ h = 120 m.
OR
Practice Paper –3
SECTION-A
1. (C) 64x2 – 1 = 0 ⇒ (8x)2 = 1
1
⇒ 8x = ± 1 ⇒ x=±
8
1
Out of which positive real root is x = .
8
2. (A) Given A.P. becomes: 5, 8, 11, ....., 113.
an = a + (n – 1) d
⇒ 113 = 5 + (n – 1)3
113 − 5
⇒ n–1= = 36 ⇒ n = 37.
3

290 M A T H E M A T I C S – X
3. (A) In ∆OAD, 9. (C) From figure,
OA = 5 cm, OD = 3 cm AB = AP + BP
∠ODA= 90° = AQ + BS
∴ AD = OA2 – OD2 = AQ + (BC – CS)
= AQ + (BC – CR)
= 52 – 32
= 5 cm + (7 cm – 3 cm)
= 4 cm
= 9 cm. ∴ x = 9 cm.
∴ AB = 2AD = 2 × 4 cm = 8 cm.
10. (D) Let height of pole be x m
4. (C) R2 + 82 = 102 In ∆ABC,
⇒ R2 = 100 – 64
AB
= 36 sin 30° =
AC
∴ R = 6 cm.
1 AB
5. (D) ∠PBA + ∠PAB + ∠APB = 180° (ASP) ⇒ =
2 AC
⇒ ∠PBA + ∠PBA + 52° = 180° 1 x
(∵ PA = PB) ⇒ = ⇒ x = 10 m.
2 20
180° − 52°
⇒ ∠PBA = = 64°. SECTION-B
2
6. (B) ar(APQB) = ar(sector AOB) –
ar(sector POQ) 11.
2 60° 2 60°
= π × (21) × – π × (14) × Using distance formula,
360° 360°
60 (10 − 2)2 + ( y + 3)2 = 10
= π× [212 – 142]
360
⇒ 64 + (y + 3)2 = 100
22 1
= × × 35 × 7 ⇒ (y + 3)2 = 100 – 64 = 36
7 6
⇒ y + 3 = ± 6 ⇒ y = 3 or – 9.
385
= cm2.
3 12. Let number of blue balls = x
7. (D) A non-leap year having 365 days ∴ Total balls = x + 6
has 52 weeks and one day out of 7 days. x
∴ P(drawing a blue ball) =
That one day could be Sunday, Monday, x+6
Tuesday, Wednesday, Thursday, Friday or 6
Saturday. and P(drawing a red ball) =
x+6
∴ Total probabilities = 7. According to questions,
Favourable outcomes = 1
x 6 
∴ Required probability = 3 
x+6  x + 6 
1
(i.e., having 53 sundays) = . ⇒ x = 18.
7
OR
8. (C) Let radius = 4x; slant height = 7x
No, the sample space has 8 outcomes. Out
Consider, π × 4x × 7x = 792
of them only one outcome say, TTT,
⇒ x2 = 9 represents no heads, so the probability of
⇒ x=3 1
∴ Radius = 4x = 12 cm. no heads is .
8

P R A C T I C E P A P E R S 291
13. For real roots D ≥ 0 ⇒ k2 – 4 ≥ 0 1
18. As A= lr
⇒ (k – 2)(k + 2) ≥ 0 2
⇒ k ≤ –2 or k ≥ 2. 1
⇒ 20π = × 5π × r
∴ Required value of k is 2. 2
14. No. ⇒ r = 8 cm.
Each side of the outer square
SECTION-C
= diameter of the circle
=d 19. The possible outcomes of the experiment
So, area of square = d2 are listed in the given table the first number
in each ordered pair is the number
Each diagonal of the
inner square =d appearing on the blue die and the second
number is that on the grey die.
i.e., 2 × side = d
d d2
∴ Side = ⇒ Area = .
2 2
Hence area of inner square
1
= × area of outer square.
2
15. an = 2n + 1 ⇒ a1 = 3
n n
∴ Sn = (a1 + an) ⇒ (3 + 2n + 1)
2 2
= n(n + 2).
16. Let third vertex be C(x, y).
2
We have A(2, 3), B(– 2, 1) and G(1, ) So, the number of possible outcomes
3
Using Centroid formula, = 6 × 6 = 36.
2– 2+x 2 3 +1+ y (i) The outcomes favourable to the event
1= ; = ‘the sum of the two numbers is 8’
3 3 3
denoted by E, are: (2, 6), (3, 5), (4, 4),
⇒ x = 3, y = – 2
(5, 3), (6, 2) (see figure).
⇒ Vertex C is (3, – 2).
i.e., the number of outcomes
17. As XP = XQ ... (i) favourable to E = 5.
AP = AR ... (ii) 5
Hence, P(E) =
BQ = BR ... (iii) 36
(ii) As you can see from figure, there is
no outcome favourable to the event
F, ‘the sum of two numbers is 13’.
0
So, P(F) = =0
36
From equation (i),
(iii) As you can see from figure, all the
⇒ XA + AP = XB + BQ
outcomes are favourable to the event
{... XP = XA + AP; XQ = XB + BQ} G, ‘sum of two numbers ≤ 12’.
⇒ XA + AR = XB + BR [Using (ii ) and (iii)] 36
So, P(G) = = 1.
Hence proved. 36

292 M A T H E M A T I C S – X
{ }
20. Comparing given equation with
15 8 × 19
ax2 + bx + c = 0, we get = 8 2+ = = 76.
2 2
a = p2, b = p2 – q2, c = – q2
1 1
∴ D = b2 – 4ac Case II. If d = – , then a = 3 – 4  −  = 5
2  2
⇒ D = (p2 – q2)2 – 4(p2)(– q2)
= p4 + q4 + 2p2q2 16   1 
∴ S16 =  2 × 5 + (16 – 1)×  –  
= (p2 + q2)2 > 0 2   2 

{ }
∴ Given equation has two real roots given by: 15 8×5
2 2 2 2
= 8 10 – = = 20.
–b± D – (p – q ) ± (p + q ) 2 2
x= =
2a 2 p2 22. From figure, ar (∆ABD)
q2 1
⇒ x= or –1. = [– 5(– 5 – 5) – 4(5 – 7) + 4(7 + 5)]
2
p2
OR 1 106
= [50 + 8 + 48] = = 53 sq. units.
3 2x2 − 5x − 2 = 0 2 2
We need to divide –5x into two parts such
that sum of them is –5x and product of
them is 3 2 x2 ×(– 2) = – 6x2. Such parts
are – 6x and x.
∴ 3 2x2 − 6x + x − 2 = 0
or 3 2 x ( x – 2) + 1 ( x − 2) = 0
Also, ar (∆BCD)
or ( x − 2)(3 2x + 1) = 0
i.e., x– 1
2 = 0 or 3 2x + 1 = 0 = [– 4(– 6 – 5) – 1(5 + 5) + 4(– 5 + 6)]
1 2 2
i.e., x = 2 or x = − = – 1 38
3 2 6 = [44 – 10 + 4] = = 19 sq. units.
2 2 2
Hence, the required roots are 2 and − . So area of quadrilateral ABCD
6
21. As a3 + a7 = 6 and a3. a7 = 8 = ar (∆ABD) + ar (∆BCD)
⇒ (a + 2d) + (a + 6d) = 6 ... (i) = 53 + 19 = 72 sq. units.
and (a + 2d)(a + 6d) = 8 ... (ii) 23. Let ABCD is a parallelogram.
From (i), 2a + 8d = 6 ⇒ AB = DC and AD = BC
⇒ a + 4d = 3 ⇒ a = 3 – 4d ∴ Consider
∴ Using it in (ii), we get AD = AS + DS
(3 – 4d + 2d)(3 – 4d + 6d) = 8 = AP + DR
⇒ (3 – 2d)(3 + 2d) = 8 {∵ AS = AP, DS = DR}
⇒ 9 – 4d2 = 8 ⇒ 4d2 = 1 = (AB – PB) + (DC – RC)
1 1 = (AB – BQ) + (AB – CQ)
⇒ d2 = ⇒ d=± {∵ DC = AB and PB = BQ and RC = CQ}
4 2
1 = 2 AB – (BQ + CQ)
 1
Case I. If d = , then a = 3 – 4   = 1. = 2 AB – BC
2  2
AD = 2 AB – AD {∵ BC = AD}
16 1 ⇒ 2 AD = 2 AB ⇒ AD = AB
∴ S16 = {2 × 1 + (16 – 1) × }
2 2 ∴ ABCD is a rhombus. Hence proved.

P R A C T I C E P A P E R S 293
24.

∆A’BC’ is required triangle.


OR

AP = AQ and BR = BS are required tangents.


294 M A T H E M A T I C S – X
25. Let PQ be the tower of height h m. 7 k − 33
or = −2
In right ∆APQ, k +1
h ⇒ 7k – 33 = 2k + 2 or 7k – 33 = – 2k – 2
tan θ = ... (i)
4 ⇒ 5k = 35 or 9k = 31
In right ∆BPQ, 31
⇒ k = 7 or k = .
h 9
tan (90° – θ) =
9 31
Therefore, required values of k = 7, .
h 9
cot θ = ... (ii)
9 27. Total area = Area of sector OAB + Area of
Multiplying corresponding sides of sector ODC + Area of ∆OAD
equations (i) and (ii), we get + Area of ∆OBC.
h h 90 22 90 22
tan θ × cot θ = × = × × 28 × 56 + × × 28 × 56
4 9 360 7 360 7
h2 +
1 1
× 56 × 56 + × 56 × 56 m2
⇒ 1 = ⇒ h2 = 36
36 4 4
∴ h = 6 m. = 22 × 56 + 22 × 56 + 14 × 56 + 14 × 56
26. It is given that the point A divides the join = 56(22 + 22 + 14 + 14) m2
of P(–5, 1) and Q(3, 5) in the ratio k : 1. So = 56 × 72 m2 = 4032 m2.
the coordinate of A are: OR
 3k − 5 5k + 1 We can divide the given figure into two
 , 
k +1 k +1  parts, one part as a rectangle with length
∴ Area of ∆ABC a = 8 m, breadth b = 4 m and the other one
1 as a semicircle with radius r = 2 m.
= x (y – y ) + x2(y3 – y1) + x3(x1 – y2)
2 1 2 3
1 3k − 5   5k + 1  
⇒∆ = ⋅ (5 + 2) +  − 2 −   + 7
2 k +1   k + 1 
 5k + 1 
 − 5
 k +1 
1 21k − 35 7 k + 3 − 28
= − +
2 k +1 k +1 k +1 Now, required area
1 14 k − 66 7 k − 33 = ar(rectangle) + ar(semicircle)
= =
2 k +1 k +1 1
= a × b + πr2
2
7 k − 33
∴ =2 [... ar(∆ABC = 2] 1 22
k +1 = 8×4+ × × 22
2 7
7 k − 33 = 32 + 6.29 = 38.29 m2.
⇒ =±2
k +1
28. Let BPC be the hemisphere and ABC
7 k − 33 be the cone standing on the base of the
⇒ =2
k+1 hemisphere (see figure). The radius BO of
the hemisphere (as well as of the cone)

P R A C T I C E P A P E R S 295
1 a = y (cot α – cot θ) ... (iv)
= × 4 cm = 2 cm. Again eliminating x from equations (i) and
2
(iii), we get
b = y (cot β – cot θ) ... (v)
Eliminating y from equations (iv) and (v),
we get
a cot α − cot θ
=
b cot β − cot θ
⇒ b cot α – b cot θ = a cot β – a cot θ
2 3 1 2 ⇒ a cot θ – b cot θ = a cot β – b cot α
So, volume of the toy = πr + πr h
3 3 ⇒ cot θ (b – a) = b cot α – a cot β
2 1  b cot α − a cot β
=  × 3.14 × (2)3 + × 3.14 × (2)2 × 2  ⇒ b–a=
3 3  cot θ
2 1  Hence proved.
= 23 ×  × 3.14 + × 3.14 = 8 × 3.14
3 3  OR
= 25.12 cm3. Let AB = height of building = 30 m
Now, let the right circular cylinder EFGH C D = height of boy = 1.5 m
circumscribe the given solid. The radius of ∴ AG = AB – GB
the base of the right circular cylinder = HP
= AB – CD {∵ GB = CD}
= BO = 2 cm, and its height is
= 30 – 1.5 = 28.5 m
EH = AO + OP = (2 + 2) cm = 4 cm
Let the distance travelled by the boy towards
So, the volume required
the building be x m.
= Volume of the right circular cylinder
– Volume of the toy
= (3.14 × 2 × 4 – 25.12) cm3
2

= 25.12 cm3
Hence, the required difference of the two
volumes = 25.12 cm3.

SECTION-D
29. Let PQ be the leaning tower.
Let PM = x and QM = y,
In right ∆PMQ,
x
cot θ = In ∆AEG,
y
AG
⇒ x = y cot θ .... (i) tan 60° =
GE
Similarly, right triangles AMQ and BMQ
respectively provides. AG
⇒ 3=
a + x = y cot α ... (ii) GE
and b + x = y cot β ... (iii)
AG 28.5
Eliminating x from equation (i) and (ii), we ⇒ GE = = ... (i)
get 3 3

296 M A T H E M A T I C S – X
Again in ∆AGC, ⇒ x2 + 6x – 216 = 0
⇒ 2
x + 18x – 12x – 216 = 0
AG AG
tan 30° = = ⇒ x(x + 18) – 12(x + 18) = 0
GC GE + x
⇒ (x + 18) (x – 12) = 0
1 28.5 ⇒ x = – 18 or x = 12
⇒ =
3 GE + x As side can’t be negative, so take
x = 12 m
⇒ GE + x = 28.5 × 3
∴ y = x + 6 = 18 m.
From equation (i), OR
28.5 During 2 pm to 3 pm the minutes hand
⇒ + x = 28.5 × 3
3 complete one round, i.e., 60 minutes.
At t minutes past 2 pm, the minutes hand
⇒ 28.5 + 3 x = 28.5 × 3
⇒ 3 x = 85.5 – 28.5  t2 
needed  – 3  minutes to show 3 pm.
57 3  4 
⇒ x= ×
3 3
 t2 
57 i.e.,  – 3  + t = 60
= 3 = 19 3 m. 4 
3
30. Let side of 1st square = x m t 2 – 12
⇒ + t = 60
and side of 2nd square = y m 4
(Assuming y > x) ⇒ t2 – 12 + 4t = 240
∴ Area of 1st square = x 2
⇒ t2 + 4t – 252 = 0
And area of 2nd square = y2
⇒ t2 + 18t – 14t – 252 = 0
According to question,
⇒ t(t + 18) – 14(t + 18) = 0
x2 + y2 = 468 ... (i)
⇒ (t – 14) (t + 18) = 0
Also, perimeter of 1st square = 4x
And perimeter of 2nd square = 4y ⇒ t – 14 = 0 or t + 18 = 0
⇒ t = 14 or t = – 18
(Rejected because time cannot be negative)
∴ t = 14.
31. 1st Part
Let AP and AQ are tangents drawn from an
external point A to the circle with centre O.
Again according to question, Join OP, OQ and OA.
4y – 4x = 24
⇒ y–x=6
⇒ y=6+x ... (ii)
Using (ii) in (i),
x2 + (6 + x)2 = 468 Now in ∆OPA and ∆OQA,
⇒ x + 36 + x2+ 12x = 468
2
OP = OQ = r (Radius of circle)
⇒ 2x2 + 12x – 432 = 0
OA = OA (Common)
P R A C T I C E P A P E R S 297
∠OPA = ∠OQA = 90° So three middle most terms are:
⇒ ∆OPA ≅ ∆OQA (RHS)  n+1
th
  n+1  n+1 
th th
 – 1 ,   , + 1
⇒ AP = AQ (CPCT)  2   2   2 
Hence proved.
th th th
2nd Part i.e.,  37 + 1 – 1  ,  37 + 1  ,  37 + 1 + 1 
Using above theorem in figure,  2   2   2 
AD = AF ... (i) i.e., a18, a19 , a20
BD = BE ... (ii) As sum of three middle most term = 225
CE = CF ... (iii) ⇒ a18 + a19 + a20 = 225
As given, AB = AC ⇒ (a + 17d) + (a + 18d) + (a + 19d) = 225
⇒ AB – AD = AC – AD (d is common difference)
⇒ AB – AD = AC – AF ⇒ 3a + 54d = 225
[Use equation (i)] i.e., a + 18d = 75 ...(i)
⇒ BD = CF Again, sum of last three term = 429
⇒ BE = CE ⇒ a35 + a36 + a37 = 429
[Use equations (ii) and (iii)] (... n = 37)
Hence proved. ⇒ (a + 34d) + (a + 35d)(a + 36d) = 429
32. Volume of frustum ⇒ 3a + 105d = 429
1 i.e., a + 35d = 143 ...(ii)
= πh (r12 + r22 + r1r2) Solving (i) and (ii), we get a = 3, d = 4
3
1 22 Therefore, A.P. is 3, 7, 11, 15, ........
= × × 45 × (282 + 72 + 28 × 7)
3 7 9
34. m
22 × 15 8
= × 7 × (28 × 4 + 7 + 28) 2
7 3
Hint: Volume of earth dug out = π   × 14
= 330 × 147 = 48510 cm3 2
Curved surface area Volume of embankment = π[(5.5)2 – (1.5)2] × h
= π (r1 + r2)l, i.e., π(r1 + r2) h2 + (r1 – r2 )2 9
⇒ × 14 = 7× 4 × h
4
22
= (28 + 7) 45 2 + (28 – 7)2 9 14 9
7 ⇒ h= × = m.
4 7×4 8
= 110 2025 + 441 = 110 × 49.65 cm2
= 5461.5 cm2. Practice Paper – 4
Now, the total surface area of the frustum
= Curved surface area + πr12 + πr22 SECTION-A
1. (D) b2 – 4ac = 0
22 22 2 2
= 5461.5 cm2 + (28)2 cm2 + (7) cm
7 7 b2
⇒ c= .
= 5461.5 cm2 + 2464 cm2 + 154 cm2 4a
= 8079.5 cm2. 2. (C) an = a + (n – 1)d
33. Let an A.P. is a, a + d, a + 2d, .... 10 1
⇒ = – 1 + (n – 1) ×
Here, n = 37 which is odd. 3 6

298 M A T H E M A T I C S – X
n–1 10 13 10. (B) OQ ⊥ PR and AB || PR.
⇒ = +1 ⇒ n – 1 = 6 × So, QO perpendicular bisector of AB
6 3 3
⇒ n = 27. ⇒ ∆AQB is an isosceles triangle.
⇒ QO is also angle bisector of ∠AQB.
3. (A) In ∆OPR,
∴ ∠AQB = 2∠BQO
∠POR = 55°, ∠ORP = 90°
= 2 × (90° – 70°) = 40°.
∴ ∠OPR = 180° – (55° + 90°) = 35°
∴ ∠QPR = 2∠OPR = 2 × 35° = 70°. SECTION-B

4. (B) 11. From figure,


∠OSQ = ∠OQS (As OS = OQ)
= ∠OQL – ∠SQL
= 90° – 50° = 40°.

x= 252 – 7 2 = ( 25 + 7 )( 25 – 7 )
= 576 = 24 cm.
5. (A) ar(segment APB)
= ar(sector APBO) – ar(∆AOB)
60° 3 2
= × πr2 – r
360° 4
and ∠OSR = ∠ORS (As OS = OR)
π 3 2 = ∠ORM – ∠SRM
=  –  r sq. units.
6 4  = 90° – 60° = 30°
Now, ∠QSR = ∠OSQ + ∠OSR
6. (C).
= 40° + 30° = 70°.
7. (A)Volume of n balls= Volume of the cone OR
4 1
⇒ n× × π × 33 = π × 122 × 24 False.
3 3
Let a number of
⇒ n × 36π = 144 × 8π
circles with centres
⇒ n= 32. O1, O2, O3 ....... touch
8. (C) Number of cards which are neither the line PQ at A. Join
red nor queen = 52 – 26 – 4 + 2 = 24 A to O1, O2, O3, ...... .
P(a drawn card is neither red nor Therefore, O1A ⊥ PQ,
24 6 O2A ⊥ PQ, O3A ⊥ PQ, ..... at same point A.
queen) = = .
52 13 So; all the centres O1, O2, O3, ...... lie on a
h 15 3 line perpendicular to PQ but the line does
9. (C) tan 30° = ⇒ h= × not bisect PQ.
15 3 3
12.
15 × 1.732
⇒ h=
3
Let coordinates of B are (x, y)
∴ h = 8.66 m.
P R A C T I C E P A P E R S 299
∴ Using section formula:

 3 x + 8 3 y + 20 
 ,  = (–1, 2)
 7 7 

3x + 8 3 y + 20
⇒ = – 1 and =2
7 7
⇒ 3x = – 15 and 3y = – 6
⇒ x = – 5 and y= –2 Required area OAPB
∴ B(– 5, – 2) is required point. = Area of the circle – Area of minor
sector OAQB
13. For equal roots,
45°
D= 0 = π × 202 – π × 202 ×
⇒ b2 – 4ac = 0 360°
⇒ 4(1 + 3k)2 – 28(3 + 2k) = 0 = 400π – 50π = 350 × 3.14
⇒ 1 + 9k2 + 6k – 21 – 14k = 0 = 1099 cm2.
⇒ 9k2 – 8k – 20 = 0
17. As 2πR = 48 and 2πr = 36
⇒ 9k2 – 18k + 10k – 20 = 0
24 18
⇒ 9k(k – 2) + 10(k – 2) = 0 ⇒ R= and r =
π π
⇒ (k – 2) (9k + 10) = 0
⇒ k – 2 = 0 or 9k + 10 = 0 1
∴ V= πh(R2 + r2 + Rr)
3
– 10
∴ k = 2 or .
9 1  24 2 18 2 24 × 18 
= π × 11 ×  2 + 2 + 
14. nth term, i.e., an 3  π π π2 
= Sn – Sn – 1
11 1332 7
⇒ = n2 + 8n – {(n – 1)2 + 8(n – 1)} = π× 2
= 11 × 444 ×
3 π 22
= n2 + 8n – (n2 – 2n + 1 + 8n – 8)
= 2n + 7 = 1554 cm3.
∴ a15 = 2 × 15 + 7 = 30 + 7 = 37. 18. Total number of cards

15. True, because the coordinates of the mid- n(S) = 50 – 2 = 48


points of both the diagonals coincide, that Let E be the event that the card is not
is, the diagonals bisect each other at primes. Prime numbers are: 3, 5, 7, 11, 13,
17, 19, 23, 29, 31, 37, 41, 43, 47.
1 5 ∴ n(E) = 48 – 14 = 34
 , .
2 2
n (E ) 34 17
∴ P(E) = = = .
15. 7  l n (S ) 48 24
16. θ = radian ∵ θ =
20  r 
SECTION-C
15.7 180° 15.7 180°
= × = × = 45° 19. (i) Total outcomes = 36
20 π 20 3.14
E = 6 will not come up either time
 180°  ⇒ E = (1, 1), (1, 2), (1, 3), (1, 4), (1, 5)
∵ 1 radian = π 
  (2, 1), (2, 2), (2, 3), (2, 4), (2, 5)

300 M A T H E M A T I C S – X
(3, 1), (3, 2), (3, 3), (3, 4), (3, 5) S1 = (n + 1) (a + nd)
(4, 1), (4, 2), (4, 3), (4, 4), (4, 5) also S2 = Sum of even terms
(5, 1), (5, 2), (5, 3), (5, 4), (5, 5) = a2 + a4 + ........ + a2n
∴ n(E) = 25 n
= {a + a2n}
25 2 2
∴ Required probability = .
36 n
= {a + d + a + (2n – 1).d}
(ii) P(6 will come up at least one) 2
P(Ē) = 1 – P(E) [∵ Using (i)]
n
25 11 = {2a + 2nd}
=1– = . 2
36 36
= n(a + nd)
20. Multiplying the equation throughout by ∴ S1 : S2 = (n + 1) (a + nd) : n(a + nd)
5, we get =n+1:n
25x2 – 30x – 10 = 0 Hence proved.
This is the same as OR
(5x)2 – 2 × (5x) × 3 + 32 – 32 – 10 = 0 Let the angles be a – d, a and a + d.
i.e. (5x – 3)2 – 9 – 10 = 0 Now,
i.e. (5x – 3)2 – 19 = 0 a – d + a + a + d = 180°
i.e. (5x – 3)2 = 19 (Angles sum property of a triangle)
∴ 3a = 180°
i.e. 5x – 3 = ± 19
or a = 60° ...(i)
i.e. 5x = 3 ± 19 1
Also, a – d = (a + d)
3 ± 19 2
i.e. x= (Given condition)
5
3 + 19 2a – 2d = a + d
Therefore, the roots are and or a = 3d
5
Using (i), we get
3 – 19 3d = 60°
.
5 ⇒ d = 20°
21. Let first term = a ∴ a – d = 60° – 20° = 40°
Common difference = d and a + d = 60° + 20° = 80°
Now, ak = a + (k – 1) . d ...(i) Hence, the required angles are 40°, 60°
and 80°.
Let S1 = Sum of odd terms
22. 24 sq. units.
⇒ S1 = a1 + a3 + a5 + ........+ a2n + 1
Hint: PQ = 26 ; QR = 26 ;
n+ 1
⇒ S1 = {a1 + a2n + 1}
2 RS = 26 ; SP = 26
n+ 1 ⇒ PQRS is a rhombus.
= {a + a + (2n + 1 – 1)d}
2 As PR = 4 2 ; QS = 6 2 .
[∵ Using (i)] ∴ PR ≠ QS
n+ 1 ⇒ diagonals are unequal.
= {2 a + 2nd}
2 ⇒ PQRS is not a square.

P R A C T I C E P A P E R S 301
1 We know that pair of tangents drawn from
Area of PQRS = × (Product of diagonals) an external point to a circle are equal in
2
length.
= 24 sq. units.
∴ CA = CB
23. We are given a circle In ∆CAB, ∠CAB = ∠CBA ...(i)
with centre O, an (∵ CA = CB)
external point T and Now, ∠CAB + ∠PAB = 180° (Linear pair)
two tangents TP and ⇒ ∠PAB = 180° – ∠CAB ...(ii)
TQ to the circle, Similarly, ∠QBA = 180° – ∠CBA
where P, Q are the points of contact (see
⇒ ∠QBA = 180° – ∠CAB ...(iii)
figure). We need to prove that
[Using (i)]
∠PTQ = 2∠OPQ
From (i), (ii) and (iii), we have
Let ∠PTQ = θ
∠CAB = ∠CBA and ∠PAB = ∠QBA
As we know that tangent segments
Hence proved.
drawn from an external point to the circle
are equal. So, TP = TQ 24. Let A be the point on the bridge. From A,
the angles of depression of the banks C
⇒ TPQ is an isosceles triangle.
and D are 30° and 45° respectively.
1
Therefore, ∠TPQ = ∠TQP = (180° – θ) Also let AB be the height of bridge which
2 is 30 m.
1 Let us find the width CD of the river.
= 90° – θ
2 In ∆ABC,
Also, OP ⊥ PT, AB 1 30
tan 30° = ⇒ =
i.e., ∠OPT = 90° BC 3 BC
So, ∠OPQ = ∠OPT – ∠TPQ ∴ BC = 30 3 m ... (i)
1
= 90° –  90° – θ 
 2 
1 1
= θ = ∠PTQ
2 2
This gives ∠PTQ = 2 ∠OPQ
Hence proved.
OR Again in ∆ABD,
We are given a chord AB of a circle and two AB 30
tan 45° = ⇒ 1=
tangents CP and CQ drawn at the points A BD BD
and B respectively, which intersect at C.
∴ BD = 30 m ... (ii)
Adding (i) and (ii), we get
BC + BD = 30 3 + 30
∴ CD = 30( 3 + 1)
= 30(1.732 + 1)
We need to prove that = 30 × 2.732
∠CAB = ∠CBA and ∠PAB = ∠QBA = 81.96 m.

302 M A T H E M A T I C S – X
25.

As PT = PT′ and QS = QS′ are required tangents.


26. Let us mark the four ⇒ x2 – 6x+9 + y2 – 8y +16 = x2 – 10x + 25
unshaded regions as I, + y2 + 4y + 4
II, III and IV (see figure) ⇒ x – 3y – 1 = 0 ...(i)
Area of I + Area of III Now, area of ∆PAB = 10
1
⇒ x1(y2 – y3) + x2 (y3 – y1) + x3(y1 – y2)
2
= 10
= Area of ABCD – Area of two 1
⇒ x ( 4 + 2 ) + 3 ( –2 – y ) + 5 ( y – 4 ) = 10
semicircles of each of radius 5 cm 2
⇒ 6x – 6 – 3y + 5y – 20 = ± 20
 1 
=  10 × 10 – 2 × × π × 5 2  cm2 ⇒ 6x + 2y – 26 = ± 20
 2 
⇒ 6x + 2y – 46 = 0 or 6x + 2y – 6 = 0
= (100 – 3.14 × 25) cm2 = 21.5 cm2 ⇒ 3x + y – 23 = 0 ...(ii)
Similarly, Area of II + Area of IV = 21.5 cm2 or 3x + y – 3 = 0 ...(iii)
So, area of the shaded design Solving (i) and (ii), we get x = 7, y = 2
= Area of ABCD – Area of (I + II + III + IV) Solving (i) and (iii), we get x = 1, y = 0
= (100 – 2 × 21.5) cm2 ∴ Required point is (7, 2) or (1, 0).
= (100 – 43) cm2 = 57 cm2. 28. Radius of the hemispherical tank
3
27. Let coordinate of P be (x, y) = m
2
Given: Points A and B are (3, 4) and (5, – 2). 3
2 22  3  3
Also, PA = PB Volume of the tank = × ×  m
3 7 2
⇒ PA2 = PB2
99 3
⇒ (x – 3)2 + (y – 4)2 = (x – 5)2 + (y + 2)2 = m
14

P R A C T I C E P A P E R S 303
So, the volume of the water to be emptied Also, tn = t1 + (n – 1)d
1 99 3 ⇒ c = a + (n – 1) (b – a)
= × m
2 14 ⇒ c–a = (n – 1) (b – a)
c–a
99 99000 ⇒ = n–1
= × 1000 litres = litres b–a
28 28
c–a c – a+b – a
⇒ n= +1 =
b–a b–a
b + c – 2a
∴ n= .
b–a
n
As we know Sn = (a + l) where a is first
25 2
Since, litres of water is emptied in 1 term and l is last term.
7
second. ( b + c – 2c )
99000 b–a
So, litres of water will be emptied i.e., Sn = . (a + c)
28 2
99000 7
in × seconds, i.e., in 16.5 minutes. ( a + c )(b + c – 2 a)
28 25 ∴ Sn =
2(b – a)
OR
Volume of the wall= 24 m × 0.4 m × 6 m Hence proved.
= 2400 cm × 40 cm × 600 cm 30. Let the speed of the stream be x km/h.
= 2400 × 40 × 600 cm3 Therefore, the speed of the boat
upstream = (18 – x) km/h and the speed
Volume of the mortar
of the boat downstream = (18 + x) km/h.
1
= × Volume of the wall The time taken to go upstream
10
distance 24
= 240 × 40 × 600 cm3 = = hours.
speed 18 – x
Let the number of bricks used in the
construction be N. Similarly, the time taken to return downstream
Now, Volume of the wall = Volume of the 24
mortar + Volume of N bricks = hours.
18 + x
⇒ 2400 × 40 × 600
According to the question,
= 240 × 40 × 600 + N × 25 × 16 × 10
24 24
240 × 40 × 600 × (10 – 1) – =1
⇒N= = 1440 × 9 18 – x 18 + x
25 × 16 × 10
i.e., 24(18 + x) – 24(18 – x) = (18 – x) (18 + x)
⇒ N = 12960
i.e., x2 + 48x – 324 = 0
Hence, 12960 bricks are used.
Using the quadratic formula, we get
SECTION-D
– 48 ± 48 2 + 1296 – 48 ± 3600
29. Given: In an A.P., t1 = a, t2 = b and tn = c. x= =
2 2
( a + c )(b + c – 2 a)
We have to show that Sn =
2(b – a) – 48 ± 60
= = 6 or – 54
Common difference d = t2 – t1 = b – a. 2

304 M A T H E M A T I C S – X
Since x is the speed of the stream, it = (AP + BR) + (DP + CR)
cannot be negative. So, we ignore the root (Using above theorem)
x = – 54. Therefore, x = 6 which gives the = (AP + DP) + (BR + CR)
speed of the stream as 6 km/h.
∴ AB + CD = AD + BC. Hence proved.
OR
32. Let r1 = radius of top of frustum
Hint:
x2 + (x + 2)2 = 290 = radius of base of cone = 3 m
r2 = radius of base of frustum = 10 m
⇒ 2x2 + 4x – 286 = 0
FG = h = height of frustum = 24 m
⇒ x2 + 2x – 143 = 0
AF = H = height of cone = 28 – 24 = 4 m
(x + 13) (x – 11) = 0
∴ L = slant height of cone
⇒ x = 11
∴ Numbers are 11, 13. = r 12 + H2 = 32 + 42 = 5 m
31. Proof: We are given a circle with centre l = slant height of frustum
O, a point P lying outside the circle and
two tangents PQ and PR on the circle = (r2 − r1 )2 + h2
from P(see figure). We are required to
prove that PQ = PR. = 7 2 + 242 = 49 + 576 = 25 m

For this, we join OP, OQ and OR. Then


∠OQP and ∠ORP are right angles,
because these angles are between the radii
and tangents. Now in right triangles OQP
and ORP,
OQ = OR Quantity of canvas required
(Radii of the same circle)
= C.S.A. of cone + C.S.A. of frustum
OP = OP (Common)
= πr1 L + π(r1 + r2) l
Therefore, ∆OQP ≅ ∆ORP (RHS)
= π(3 × 5 + 13 × 25)
This gives PQ = PR (CPCT)
22 22
= × (15 + 325) = × 340
7 7
= 1068.57 m2 (approx.)
33. Let AB = 40 m = Height of tower
CD = h = Height of lighthouse
∠CAE = 30°; ∠CBD = 60°
CD
In ∆CBD, tan 60° =
DB
Consider: AB + CD CE + DE
⇒ 3 =
= (AS + BS) + (DQ + CQ) DB
P R A C T I C E P A P E R S 305
CE + 40 120 3
⇒ 3 = ⇒ CB = ×
DB 3 3
(∵ AB = DE = 40 m) ⇒ CB = 40 3 m.
CE 40 OR
3 = + ...(i)
DB DB In figure, O is the centre of the balloon.
OP = R, ∠PAQ = θ, ∠OAB = φ.

Also in ∆AEC,
Let the height of the centre of the balloon
CE
tan 30° = be h. Thus OB = h
AE
In right-angled triangle AOP,
1 CE OP
⇒ = ...(ii) (∵ AE = DB) sin ∠OAP =
3 DB AO
Using (ii) in (i), we have θ R
⇒ sin =
1 40 2 AO
3 = +
3 DB θ
⇒ AO = R cosec ...(i)
2
40 1 2
⇒ = 3− = Also, in right-angled triangle AOB,
DB 3 3
OB h
⇒ DB = 20 3 m sin φ = ⇒ sin φ =
AO AO
From equation (ii),
⇒ AO = h cosec φ ...(ii)
1 CE
= From equation (i) and (ii), we get
3 20 3
θ
h cosec φ = R cosec
⇒ CE = 20 m 2
∴ h = CE + DE θ
∴ h = R sin φ cosec
= 20 + 40 2
⇒ h = 60 m. Hence proved.
Also distance of foot of tower from top of 34. Since the inner diameter of the glass = 5 cm
lighthouse = CB. and height = 10 cm
CD So, the apparent capacity of the glass
∴ sin 60° =
CB = πr2h
3 60 = 3.14 × 2.5 × 2.5 × 10 cm3
⇒ =
2 CB = 196.25 cm3.

306 M A T H E M A T I C S – X
But the actual capacity of the 4. (A) Area of a triangle
glass is less by the volume of drawn in a semicircle
the hemisphere at the base of is directly proportional
the glass. to its height.
2 3 1
i.e., it is less by πr ∴ar(largest ∆ABC) = × 2r × r
3 2
2 = r2 sq. units.
= × 3.14 × 2.5 × 2.5 × 2.5 cm3
3
4 3
πr
= 32.71 cm3 V1 64 3 1 43
5. (D) = ⇒ = 3
So, the actual capacity of the glass V2 27 4 3 3
πr
3 2
= apparent capacity of glass
– volume of the hemisphere r1 4 r12 16
⇒ = ⇒ 2
=
= (196.25 – 32.71) cm3 r2 3 r2 9
= 163.54 cm3. S1 4πr12 r12 16
∴ = ⇒ =
S2 4πr22 r22 9
Practice Paper–5
i.e., S1 : S2 = 16 : 9.
SECTION-A
6. (D) Let O be the centre of the circle and PA
2
1 1 5 and PB are two tangents inclined at 60°.
1. (A)   + k   – =0
2 2 4 In right-angled ∆OAP,

1 5 1 ∠OPA = 30°
⇒ k= –
2 4 4  ∠APB 
∵ ∠OPA = ∠OPB = 
⇒ k = 2.  2 
2. (B) The given A.P. is – 11, – 8, – 5, ........, 49. OA
∴ tan 30° =
To find 4th term from last, a = 49, d = – 3. AP
∴ Required term = 49 + (4 – 1) × (– 3) 1 3
= 49 – 9 = 40. ⇒ =
3 AP
3. (A) In ∆OPQ,
PQ2 = 132 – 52 ⇒ AP = 3 3 cm.
⇒ PQ = 12 cm 7. (D) Each pair of opposite sides of a
quadrilateral circumscribing a circle
subtends complementary angles at the
centre of the circle.
∴ ∠AOB + ∠COD = 180°
⇒ ∠COD = 180° – 125°
= 55°.
∴ ar( PQOR) = 2 × ar(∆POQ)
8. (C) Let the girl bought n tickets.
1 n
= 2 × × 5 × 12 ∴ = 0.08
2 6000
= 60 cm2. ⇒ n = 480.

P R A C T I C E P A P E R S 307
9. (B) Let the sun’s elevation = θ. we get a = 6, b = – 2 , c = – 2
AB D = b2 – 4ac = 2 + 48 = 50
⇒ tan θ =
BC Quadratic formula:
6
= = 3 – b ± b 2 – 4 ac
2 3 x =
2a
= tan 60°
2 ± 50 2±5 2
⇒ θ = 60°. ∴ x = =
12 12
10. (D) As XY ⊥ AB and CD || XY so CD ⊥ AB. 6 2 –4 2
∴ x = or x =
⇒ CD is bisected by AB at M. 12 12
⇒ CD = 2 DM.
2 2
In ∆OMD, i.e., x = or x = –
2 3
DM = OD2 – OM2 2 2
Hence, required roots are and – .
2 3
= 52 – 32 OR
(... r = 5 cm, OM = AM – AO = 8 – 5 = 3 cm) Consider the equation,
∴ DM = 4 cm 2x2 – 3x – 5 = 0
Therefore, CD = 2 × 4 = 8 cm. ⇒ 2
2x – (5 – 2)x – 5 = 0
(Spliting middle term)
SECTION-B 2
⇒ 2x – 5x + 2x – 5 = 0
11. Side of square = a = 7.5 × 2 = 15 cm ⇒ x(2x – 5) + 1 (2x – 5) = 0
Area of shaded part = Area of square ⇒ (2x – 5) (x + 1) = 0
– Area of circle ⇒ 2x – 5 = 0 or x + 1 = 0
= a2 – πr2 5
= 152 – 3.14 × (7.5)2 i.e., x= or – 1
2
= 225 – 176.625
5
= 48.375 cm2. Thus, and – 1 are the required roots.
2
12. Let first term = a 1 and common
14. We are given three points are A(– 4, 0),
difference = d.
B(4, 0) and C(0, 3).
⇒ 7a7 = 11a11
Using distance formula, we find the
⇒ 7(a1 + 6d) = 11(a1 + 10d)
lengths of segments (sides) as
⇒ 7a1 – 11a1 = 110d – 42d
⇒ a1 = – 17d AB = 82 + 02 = 8;
⇒ a1 + 17d = 0
BC = 42 + 32 = 5
∴ a18 = 0.

13. 6x2 – 2x – 2 = 0
CA = 42 + 32 = 5
Comparing the coefficients of like powers ⇒ BC = CA ≠ AB
of the given equation with ax2 + bx + c = 0, ⇒ ∆ABC is an isosceles triangle.

308 M A T H E M A T I C S – X
2 20 SECTION-C
15.  – , – 
 7 7  19. Number of cards with numbers from 2
Hint: Use Section formula to 101
= 101 – 2 + 1 = 100
lx2 + mx1 ly + my1
x= :y = 2 ∴ n(S) = 100
l+m l+m
(i) Let E1 be the event of drawing a card
where l : m = 3 : 4.
having an even number.
16. Sample space (all possible outcomes) Even numbers are : 2, 4, 6, ....., 100,
when rolling a pair of dice is given by: which are 50.
S = {(1, 1), (1, 2), (1, 3), (1, 4), (1, 5), (1, 6), ∴ n(E1) = 50
(2, 1), (2, 2), (2, 3), (2, 4) (2, 5), (2, 6), n(E1 ) 50 1
(3, 1), (3, 2), (3, 3), (3, 4), (3, 5), (3, 6), Now, P(E1) = = = .
n(S) 100 2
(4, 1), (4, 2), (4, 3), (4, 4), (4, 5), (4, 6),
(ii) Let E2 be the event of drawing a card
(5, 1), (5, 2), (5, 3), (5, 4), (5, 5), (5, 6),
having a square number. Square numbers
(6, 1), (6, 2), (6, 3), (6, 4), (6, 5), (6, 6)}.
are: 4, 9, 16, 25, 36, 49, 64, 81, 100.
∴ n(S) = 36
n(E 2 ) 9
Favourable outcomes are: (1, 3), (2, 2), ∴ n(E2) = 9 ∴ P(E2) = = .
n(S) 100
(2, 6), (3, 1), (3, 5), (4, 4), (5, 3), (6, 2),
(6, 6). These are 9 in counting. OR
Hence, the required probability Number of non-defective bulbs
= 24 – 6 = 18
9 1
= = . P(First bulbs is not defective)
36 4
Number of non-defective bulbs
17. Volume of the remaining solid =
Total number of bulbs
= Volume of the cube 18 3
– Volume of the cone. = =
24 4
1 2 If the first selected bulb is defective, then
= l3 – πr h
3 the number of remaining defective bulbs
= 6 – 1 = 5 and the remaing total number
1 22 of bulbs = 24 – 1 = 23.
= 73 – × × 32 × 7
3 7
5
= 343 – 66 = 277 cm3. ∴ P(Second bulb is defective) = .
23
18. 20. Given equation:
abx2 + (b2 – ac)x – bc = 0
Comparing it with
Ax2 + Bx + C = 0, we get
A = ab; B = b2 – ac; C = – bc
Extend AB and CD to meet at P. Quadratic formula:
Since the pair of tangents drawn from an
external point to a circle are equal, –B ± B 2 – 4 AC
x=
i.e., BP = DP and AP = CP. 2A
So, AP – BP = CP – DP – (b 2 – ac) ± (b 2 – ac)2 – 4ab( – bc)
i.e., AB = CD. Hence proved. =
2 ab
P R A C T I C E P A P E R S 309
OR
ac – b 2 ± b 4 + 2 ab 2 c + a 2 c 2
= Common difference = a3 – a2 = a2 – a1
2 ab
⇒ 3k 2 + 4k + 4 – (2k 2 + 3k + 6)
ac – b 2 ± (b 2 + ac )2 ac – b 2 ± (b 2 + ac) = 2k2 + 3k + 6 – (k2 + 4k)
= =
2 ab 2ab ∴ k + k – 2 = k2 – k + 6
2

2ac – 2 b 2 ⇒ 2k = 8 ⇒ k = 4
⇒ x = or x = ∴ Common difference = 42 + 4 – 2 = 18
2ab 2ab
Hence, the given terms are:
c b
⇒ x = or x = – .
a 42 + 4 × 4, 42 + 4 × 4 + 18 and 42 + 4 × 4 + 2 × 18
b
i.e., 32, 50 and 68.
3n 2 5n 22. We are given
21. Sn = +
2 2 that AB = 8 cm;
Substituting n = n – 1, we get BC = 10 cm and
3 5 CA = 12 cm
Sn–1 = (n – 1)2 + (n – 1)
2 2
3 2 5
= (n – 2n + 1) + (n – 1)
2 2
3 3 5 5
= n2 – 3n + + n– Let AD = x, then AF = x;
2 2 2 2
BD = AB – AD = 8 – x = BE
3 2 1
=
n – n–1 EC = BC – BE
2 2 = 10 – 8 + x = 2 + x = CF.
th
n term of an A.P. is given by AF = AC – CF
an = Sn – Sn – 1 = 12 – 2 – x = 10 – x
Here, we observe that
3 2 5 3 1
= n + n – n2 + n + 1 AF = x and AF = 10 – x
2 2 2 2 ∴ x = 10 – x ⇒ x = 5 cm
= 3n + 1 Hence, AD = 5 cm; BE = 8 – x = 3 cm;
∴ a31 = 3 × 31 + 1 = 94. CF = 2 + x = 7 cm.

23. Use ASP:


∠A + ∠B + ∠C = 180°
105° + 45° + ∠C = 180°
⇒ ∠C = 180° – 150° = 30°
⇒ ∠C = 30°.

A′B BC′ A′C′ 4


∆A′BC′ ~ ∆ABC such that = = = .
AB BC AC 3
310 M A T H E M A T I C S – X
24. Circumcentre of a triangle is equidistant Hence, the circumcentre is (5, 2) and the
from the vertices of the triangle circumradius is 5 units.
Let O (x, y), be the circumcentre of the given 25. Given chord is AC. Exterior angle
∆ABC. AOC = 270°. ACDA is the major segment.
∴ OA = OB = OC ∴ ar(segment ACDA)
i.e., OA2 = OB2 = OC2 = ar(sector AOCDA) + ar(∆AOC)

Taking OA2 = OC2, we get


(x – 8)2 + (y – 6)2 = (x – 2)2 + (y + 2)2 270° 1
= × π × (20)2 + × 20 × 20
⇒ x2 – 16x + 64 + y2 – 12y + 36 360° 2
= x2 – 4x + 4 + y2 + 4y + 4
3
⇒ – 12x – 16y = 8 – 100 = × 3.14 × 400 + 10 × 20
4
⇒ 12x + 16y = 92
= 942 + 200
⇒ 3x + 4y = 23 ...(i)
= 1142 cm2.
Also taking OB2 = OC2,
(x – 8)2 + (y + 2)2 = (x – 2)2 + (y + 2)2 26. Area of square = (side)2 = (4)2
⇒ (x – 8)2 = (x – 2)2 = 16 cm2
⇒ x – 8 = ± (x – 2) 1 2
Area of each quadrant = πr
But x – 8 = (x – 2) as – 8 ≠ – 2 4
∴ x – 8 = – (x – 2) 1
= × 3.14 × (1)2
i.e., 2x = 10 4
i.e., x=5 3.14
= cm2
Substituting x = 5 in equation (i), we get 4
3 × 5 + 4y = 23
3.14
i.e., 4y = 23 – 15 = 8 ∴ Area of 4 quadrants = 4 ×
4
i.e., y=2
= 3.14 cm2
Circumradius = OA
Also area of the circle drawn in middle of
= ( x – 8 )2 + ( y – 8 )2 the square = πr2
2
2
=
2
(5 – 8 ) + (2 – 6 ) 2
= 3.14 ×  
2
= 9 + 16 = 5 = 3.14 cm2

P R A C T I C E P A P E R S 311
27. In the figure drawn here, let AB be a wall
of height h m and AC be a ladder 15 m
long that makes an angle of 60° with the
wall.

Now, area of shaded part = Area of square


– Sum of areas of 4 quadrants – Area of
circle drawn in centre.
In right-angled ∆ABC,
= 16 – 3.14 – 3.14
AB
= 16 – 6.28 = 9.72 cm2 cos 60° =
AC
OR
Side of ∆ABC = a = 10 cm 1 h
⇒ =
2 15
10
Radius of each circle = r = = 5 cm
2 ⇒ 2h = 15
3 2 3 15
ar(ABC) = a = × 102 ∴ h = = 7.5 m.
4 4 2
= 25 3 = 25 × 1.732 28. Let the required ratio be λ : 1
= 43.30 cm2

We will use section formula,


– 2λ – 5 – 4 + 3λ
–3 = ;p=
λ +1 λ +1

– 4 + 3λ
i.e., – 3λ – 3 = – 2λ – 5 ; p =
λ +1
–4 + 6 2
i.e., λ = 2 ; p = =
Area of each sector situated in ∆ABC 3 3
60° 1 2 2
= × πr 2 = × 3.14 × 52 cm . Required ratio is 2 : 1 and p = .
360° 6 3
Area of the shaded region SECTION-D
= ar(ABC) – 3 × area of any one
sector 29. r = AO = BO = CO = 3.5 cm
1 CD = 15.5 cm
= 43.30 – 3 × × 3.14 × 25
6 OD = CD – CO
= 43.30 – 39.25 = 4.05 cm2. = 15.5 – 3.5 = 12 cm

312 M A T H E M A T I C S – X
1
l = AD = OD 2 + AO 2 = × 50.24 × 624 = 10450 cm3
3
= 144 + 12.25 (approx)
∴ Capacity of container
= 12.5 cm.
10450
= l = 10.45 l
1000
Cost of milk = Volume of the container
× Rate per litre
= 10.45 × ` 15
= ` 156.75.
30. Let the tap of larger diameter takes t hours
to fill the tank. So the smaller one will take
(10 + t) hours on the same work.
∴ The part of the tank filled by larger tap in
1
Now, total surface area 1 hour =
t
= C.S.A. of the cone ABD
And the part of the tank filled by the smaller
+ C.S.A. of the hemisphere ACB
1
= πrl + 2πr2 = πr(l + 2r) tap in 1 hour =
10 + t
22 So, the part of the tank filled by both the taps
= × 3.5 × (12.5 + 7)
7
1 1
= 22 × 0.5 × 19.5 = 214.5 cm2. simultaneously in 1 hour = + ...(i)
t 10 + t
OR But it is given that the taps together fill the
Given: r1 = 20 cm, r2 = 8 cm and h = 16 cm
3 75
tank in 9 = hrs.
8 8
So, the part of the tank filled by both the taps
1 8
simultaneously in 1 hour = = hrs
75 75
8
...(ii)
From the results (i) and (ii), we have
Volume of the container
1 1 8
1 +
V=
3
(
πh r12 + r22 + r1r2 ) =
t 10 + t 75

1
10 + t + t 8
=
3
2
( 2
× 3.14 × 16 20 + 8 + 20 × 8 ) t (t + 10)
=
75

1 ⇒ 750 + 150t = 8t2 + 80t


= × 50.24 × ( 400 + 64 + 160 ) ⇒ 8t2 – 70t – 750 = 0,
3

P R A C T I C E P A P E R S 313
i.e., 4t2 – 35t – 375 = 0 32. Let us take LHS of the given equation
D = (– 35t)2 – 4 × 4 × (– 375) = 1225 + 6000 – 4 + (– 1) + 2 +......+ x
∴ D = ± 7225 = ± 85 Here, 2nd term – 1st term = 3rd term – 2nd term
– 50 = 3. So, this is the sum of A.P. with common
35 ± 85 120
∴ t = = or difference d = 3 and the first term a = – 4.
2× 4 8 8
Let this A.P. contains n terms.
120
∴ t = = 15 hrs Then, using an = a + (n – 1)d, we get
8
x = – 4 + (n – 1) 3
– 50
(Time cannot be negative so t = x+4
8 ⇒ n–1 =
is rejected.) 3
And t + 10 = 25 hrs. x+7
⇒ n =
Thus, time taken by the larger tap = 15 hrs. 3
and time taken by the smaller tap = 25 hrs. x+7
Now, LHS = (– 4 + x)
3
31. DA and DC are two tangents from D to the
circle with centre F. n
[Using Sn = (a + l)]
∴ ∠ADF = ∠CDF ...(i) 2
Similarly, – 4 x + x 2 – 28 + 7 x
=
∴ ∠BEF = ∠CEF ...(ii) 6
As l || m and DE is transversal,
∴ ∠ADE + ∠BED = 180° x 2 + 3 x – 28
=
(Cointerior angles) 6
∴ (∠ADF + ∠EDF) + (∠BEF + ∠DEF) = 180° Hence, the given equation becomes
∴ (∠EDF + ∠EDF) + (∠DEF + ∠DEF) = 180° x 2 + 3 x – 28
= 437
⇒ 2 ∠EDF + 2 ∠DEF = 180° 6
⇒ ∠EDF + ∠DEF = 90° ...(iii) ⇒ x2 + 3x – 2650 = 0
x2 + 53x – 50x – 2650 = 0
⇒ (x + 53) (x – 50) = 0
⇒ x = – 53 or x = 50
But x = – 53 is not possible because the A.P.
has no negative term after the second term.
Therefore, x = 50.
OR
In ∆EDF, Ruchi has 13 flags to be fixed at one side
⇒ ∠EDF + ∠DEF + ∠DFE = 180° of her, each at R 1, R2,......., R3 (see figure)
(Angle sum property of a triangle) and the same no. on the opposite side of
it such that the flags are on the straight
⇒ 90° + ∠DFE = 180°
passage.
[Using (iii)]
∴ ∠DFE = 90°.
Hence proved.

314 M A T H E M A T I C S – X
Let initially Ruchi is at the point R with y
⇒ x= ...(ii)
her books and 27 flags. 3
Ruchi fixed one flag at the point R also
RR1 = R1R2 = ..... = R12R13 = 2 m
Fixing one flag at R1, distance covered by
Ruchi
= RR1 + R1R
= 2+2=4m
Fixing one flag at R2, distance covered by
Ruchi = RR2 + R2R
= 4 + 4 = 8 m and so on.
From equations (i) and (ii), we have
Therefore distance covered by Ruchi to fix
y
all flags on one side of R. y – 40 =
3
= 4 + 8 + 12 + ....... to 13 term
⇒ 3 y – 40 3 = y
= 4(1 + 2 + 3 +.......to 13 terms)
13 × 14 ⇒ ( 3 – 1)y = 40 3
= 4× = 364 m
2 40 3
⇒ y=
Similarly, distance covered by Ruchi to fix 3 –1
all flags on other side of R = 364 m.
40 3 3 +1
Hence, total distance covered ⇒ y= ×
3 –1 3 +1
= 364 + 364 = 728 m
Maximum distance travelled by Ruchi 40(3 + 3)
y=
carrying a flag = RR3 3–1
= 2 × 13 = 26 m.
⇒ y = 20 (3 + 3 ) ...(iii)
33. Let XP = x and PQ = y
i.e. PQ = 20 (3 + 3) metres
∴ YM = x; PM = 40 m; QM = y – 40
In right-angled triangle QYM, Again in right-angled triangle QXP,
QP
QM sin 60° =
tan 45° = XQ
YM

y – 40 3 20 (3 + 3 )
⇒ 1= ⇒ =
x 2 XQ

⇒ x = y – 40 ...(i) 20(3 + 3) × 2
⇒ XQ =
In right-angled triangle QXP, 3
QP ⇒ XQ = 40( 3 + 1)
tan 60° =
XP
Hence, PQ = 20(3 + 3) metres and
y
⇒ 3 =
x XQ = 40( 3 + 1) metres.

P R A C T I C E P A P E R S 315
34. h = 16 cm ∴ Area of sheet used to make bucket
r2 = 8 cm = C.S.A. of frustum + Area of base
r1 = 20 cm = πl(r1 + r2) + πr22
l = h 2 + (r1 – r2 )2 = 16 2 + 12 2 = π[l(r1 + r2) + r22]
= π[20 × 28 + 64]
= 256 + 144 = 400 = 20 cm.
= 3.14 × 624
= 1959.36 cm2

1959.36
∴ Cost = × 15
100
= ` 293.90
‰‰

316 M A T H E M A T I C S – X

You might also like